You are on page 1of 195

MECHANIKA TUHCH TELIES

Nvody na cvienia

NA Milan
LABAOV Eva



















2008







































Ing. Milan Na, CSc., Ing. Eva Labaov, PhD.



Recenzenti : Prof. Ing. Ctirad Kratochvl, DrSc.
Doc. Ing. Milan Michalek, CSc.

Jazykov korektra : Mgr. Valria Krahulcov

Schvlila Vedeck rada Materilovotechnologickej fakulty STU ako
vysokokolsk skript da 14. februra 2007 pre vetky tudijn programy
Materilovotechnologickej fakulty STU v Trnave

ISBN 978-80-8096-050-6
EAN 9788080960506

3
v o d


Skript Mechanika tuhch telies Nvody na cvienia s uren predovetkm
tudentom Materilovotechnologickej fakulty STU so sdlom v Trnave. Hlavnm cieom
skrpt je poskytn tudentom prehadn, zrozumiten a efektvnu tudijn pomcku pri
tdiu predmetu Mechanika tuhch telies.
Po obsahovej strnke vychdzaj skript z prednok a cvien predmetu mechanika
tuhch telies, ktor s realizovan na Katedre aplikovanej mechaniky MTF pre vetky
bakalrske tudijn programy akreditovan na MTF STU v Trnave.
Nplou skrpt s vzorov prklady z oblasti statiky, kinematiky a dynamiky tuhch telies.
V rmci prkladov z dynamiky je zahrnut aj as prkladov tkajcich sa linerneho kmitania
sstav s jednm stupom vonosti. Prklady s koncipovan tak, aby pri ich rieen tudent
pochopil zkladn zkonitosti mechaniky a zrove spsob a pravidl rieenia konkrtnych
problmov mechaniky tuhch telies. Sasou skrpt s okrem rieench prkladov aj
nerieen prklady sliace na samotdium, pre ktor s uveden vsledky.
Prv kapitolu Statika vypracovala Ing. Eva Labaov, PhD. Na vypracovan druhej
kapitoly Kinematika sa rovnakm dielom podieali Ing. Eva Labaov, PhD. a Ing Milan
Na, CSc. Tretiu kapitolu Dynamika vypracoval Ing. Milan Na, CSc.
Prianm autorov je, aby toto skriptum prispelo k interdisciplinrnemu chpaniu vetkch
ast mechaniky vo vzahu k vrobnm technolgim, informatike, materilovm vedm,
priemyslenmu ininierstvu ako aj k environmentalistike.
Autori si povauj za mil povinnos poakova doc. Ing. Jozefovi Mudrikovi, CSc.
garantovi predmetu Mechanika tuhch telies, za jeho vemi cenn pripomienky a morlnu
podporu pri spracovvan skrpt.


Autori




4
1 STATIKA
1.1 SILA A JEJ INKY NA HMOTN OBJEKT
Sila F je vektorov veliina, ktor je definovan vekosou, smerom a orientciou. Vektor
sily me by vyjadren ako sila psobiaca v rovine alebo priestore.
Silu v rovine (obr. 1.1) rozkladme do dvoch vektorovch zloiek F
x
a F
y
, t.j. potom plat
j i F F F
y x y x
F F + = + = , (1.1)
kde F
x
, F
y
s vekosti zloiek sily F v smere sradnicovch os x, y. Zloky sily F
x
a F
y
s
navzjom na seba kolm. Tieto zloky mu by v smere kladnch, resp. zpornch poloos x
alebo y a ich vekosti (mu by aj nulov) s veobecne uren vzahmi
sin cos F F F F
y x
= = , (1.2)
kde je smerov uhol, ktor zviera nositeka sily - n
F
s kladnm smerom osi x.
Vekos sily je uren pomocou Pytagorovej vety

2 2
y x
F F F + = . (1.3)
Zkladnou jednotkou sily je jeden newton [N]. Sila ako vektorov veliina je viazan k svojej
nositeke n
F
. Psobisko sily mono ubovone posun po nositeke sily a inok na objekt
sa nezmen. Sila vzhadom na vetky body na svojej nositeke m posuvn inok.


Obr. 1.1 Obr. 1.2

Silu v priestore (obr. 1.2) rozkladme do troch navzjom kolmch zloiek F
x
, F
y
a F
z

k j i F F F F
z y x z y x
F F F + + = + + = , (1.4)
, cos , cos , cos = = = F F F F F F
z y x
(1.5)
priom , , s smerov uhly nositeky sily k prslunm sradnicovm osiam x, y, z.
Vekos sily je uren zo vzahu

2 2 2
z y x
F F F F + + = . (1.6)
Kosnusy smerovch uhlov s uren zo vzahov (1.5)
F
y
x
y
F
x
i

j

z
k

F
z



F
0
n
F
F
y
x

n
F
y
F
x
F

i

j

5

F F F
z
y
x
F
cos ,
F
cos ,
F
cos = = = , (1.7)
priom plat
1 cos cos cos
2 2 2
= + + .
Ak na objekt psobia dve a viacej sl, zloky ich vslednej sily (vslednice R) s zskan
vektorovm stom prslunch zloiek danch sl
j i R R R
y x y x
R R + = + = , (1.8)
n i F R F R
n
i
yi y
n
i
xi x
, ..... , 2 , 1 , ,
1 1
= = =

= =
. (1.9)
Okrem posuvnho inku m sila aj otav inok, ktor vznik vzhadom na ubovon
bod v rovine alebo v priestore, ktor nele na nositeke sily. Mierou otavho inku je
moment sily. Moment sily je vektorov veliina.
Moment sily M vzhadom na bod A je vyjadren vektorovm sinom
F r M = , (1.10)
kde r je polohov vektor psobiska sily F vzhadom na bod A (obr. 1.3a). Vekos momentu
sily vzhadom na bod A vyjadren ako vekos vektorovho sinu (1.10), t.j. plat
Fd rF M = = sin , (1.11)
kde - uhol, ktor zvieraj smery nositeliek vektorov r a F vychdzajcich z jednho bodu,
d - kolm vzdialenos bodu A od nositeky sily n
F
.
Zkladnou jednotkou momentu sily je jeden newtonmeter [Nm]. Moment sily vzhadom na
bod je viazan vektor, t.j. k rznym bodom m t ist sila in otav inok. Moment sily je
kolm na rovinu vytvoren vektormi r, F. Vektory v porad r, F, M tvoria pravotoiv
systm (obr. 1.3b). Prakticky mono vektor uri pomocou pravidla pravej ruky.
Na rieenie rovinnch loh v statike je zaveden dohoda, e moment sily m kladn
znamienko (+), ak ota objektom proti zmyslu otania hodinovch ruiiek a naopak m
zporn znamienko (), ak ota objektom v zmysle otania hodinovch ruiiek (obr. 1.3c).



Obr. 1.3a Obr. 1.3b Obr. 1.3c
Moment sily vzhadom na os telesa
Moment sily F vzhadom na os o je kolm priemet momentu M
B
do smeru osi o. M
B
je
moment sily F vzhadom na ubovon bod B leiaci na osi o (obr. 1.4). Moment M
o

vyjadruje otav inok sily vzhadom na dan os a je definovan vzahom
A

d

F
M
M = Fd

A

d

F
M
M = + Fd

F
r
M r
F
M
n
F

.
d

A


6
e e e F r e e M M
o o
M = = = ] ) [( ) (
B
, F r M =
B
, (1.12)
kde e je jednotkov vektor v smere osi o.
Na zklade defincie momentu je moment sily vzhadom na os nulov, ak nositeka sily n
F
je
s osou o rovnoben alebo rznoben. Moment sily vzhadom na os je vektor viazan k danej
osi, ie m vdy smer osi o.



Obr. 1.4 Obr. 1.5
Moment silovej dvojice
Silov dvojicu tvoria dve rovnoben, opane orientovan sily rovnakej vekosti F, ktor
leia na rovnobench nositekch (obr. 1.5). Kolm vzdialenos nositeliek je d, nazva sa
tak rameno silovej dvojice. Vslednica takto definovanch sl sa rovn nule, t.j. posuvn
inok je nulov. Otav inok takejto dvojice sl je uren momentom silovej dvojice.
Moment silovej dvojice je von vektor, t.j. m rovnak otav inok vzhadom na
ubovon bod v priestore a jeho vekos je dan vzahom:
M = Fd. (1.14)
Vektor momentu silovej dvojice je kolm na rovinu uren nositekami sl a jeho orientcia je
dan pravidlom pravej ruky.
Varignonova veta (momentov veta)
Moment M
0
vslednice R sstavy sl (obr. 1.6a) so spolonm psobiskom (bod A) vzhadom
na bod 0 sa rovn stu momentov jednotlivch sl vzhadom na ten ist bod 0. Taktie plat,
e moment sily F vzhadom na bod 0 sa rovn stu momentov jednotlivch zloiek tejto sily
v smere sradnicovch os vzhadom na ten ist bod 0 (obr. 1.6b).
r
F
M
B
n
F
.
o

B

e
M
o
d
M
M
F
1
F
2
| F
2
| = | F
1
| = F
F
2
= F
1

7


Obr. 1.6a Obr. 1.6b

Prklad 1.1
Urte vekos a smerov uhol nositeky vslednej sily psobiacej na konzolu poda obrzka.
Dan: F
1
= 100 N, F
2
= 300 N, = 20, = 30



Do bodu A na konzole je zvolen zaiatok sradnicovho systmu. Vslednica sl F
1
a F
2
m
psobisko taktie v bode A. Vekosti zloiek vslednice s zskan zo vzahov (1.9)

N 6 , 225 30 cos . 300 20 sin . 100 cos sin
N 97 , 243 30 sin . 300 20 cos . 100 sin cos
2 1
2 1
= + = = =
= + = + = =

o o
o o
F F F R
F F F R
yi y
xi x


Vekos vslednice:
( ) N 29 , 332 6 , 225 97 , 243
2 2 2 2
= + = + =
y x
R R R .
r
0
F
1
F
2
F
i
A

= =
= = =
n
i
i
n
i
i
1 1
0
; F R F r R r M
r
0
F
x
F

A
F
y
F
z
z y x
F r F r F r F r M + + = =
0


F
1
F
2
A

x

y

F
1x
F
1y
F
2x
F
2y
Poznmka: Pri pouit znaku
xi
F budeme ma na mysli vdy

=
n
i
xi
F
1
, ak by sa v alom
texte suma nevzahovala na vetky sily, bude to pri znaku uveden.


F
1
F
2
A

8
Smerov uhol vslednice
R
urme poda vzahu (1.7)
. 76 , 42
, 7342 , 0
29 , 332
97 , 243
cos
o
=
= = =
R
x
R
R
R

Funkcia kosnus m kladn hodnoty v prvom a tvrtom
kvadrante, preto je potrebn uri aj funkciu snus

. 76 , 42 6789 , 0
29 , 332
6 , 225
sin
o
= =

= =
R
y
R
R
R

Funkcia snus m zporn hodnoty v treom a vo tvrtom kvadrante. Smerov uhol nositeky
n
R
vslednice potom le vo tvrtom kvadrante a jeho vekos je

o o o
24 317 76 42 360 , , = =
R
.
Vslednicu ako vektor mono zapsa:
- v tvare mnoholena: j i j i R 6 , 225 97 , 243 = + =
x x
R R ,
- v tvare vektora: ) 6 , 225 ; 97 , 243 ( ) ; ( = =
y x
R R R .
Prklad 1.2
1. Vertiklna sila F psob na koniec nstrkovho ka. Vypotajte vekos momentu
sily vzhadom na bod 0 (obr.A).
2. Vypotajte vekos momentu, ak na k psob horizontlna sila (obr.B).
3. Vypotajte vekos momentu, ak sila psob kolmo na os ka (obr.C).
Dan: F = 100 N, = 60 , l = 0,18 m


Obr. A

Obr. B

Obr. C
1. Kolm vzdialenos d nositeky sily n
F
vzhadom na bod 0 vyjadrme z obr. A
m 09 , 0 60 cos 18 , 0 cos = = =
o
l d .
Vekos momentu vzhadom na bod 0 (1.11) je
Nm 9 09 , 0 . 100
0
= = = Fd M .
Vektor momentu M
0
je kolm na rovinu nkresu a to v smere hodinovch ruiiek, ie
je zporn. Pomocou vektorovho sinu polohovho vektora r a sily F dostaneme
k k j j i F r M 100 . 60 cos 18 , 0 . cos ) ( ) sin cos (
0
o
= = + = = F l F l l ,
k M 9
0
= .
2. Kolm vzdialenos d nositeky sily n
F
vzhadom na bod 0 vyjadrme z obr. B
d = l sin = 0,18.sin 60 = 0,156 m.
R

R
x
R
y
R


n
R
y

x

R
n
F

F

0

y
d

.

M
0
x

n
F

F

n
0
y
d
.
M
0
x
n
F

F
0

y

d

.
M
0
x

F

0

l

9
Vekos momentu vzhadom na bod 0 je
M
0
= Fd=100.0,156 = 15,6 Nm.
Vektor momentu M
0
je kolm na rovinu nkresu a to proti smeru hodinovch ruiiek,
ie je kladn. Pomocou vektorovho sinu polohovho vektora r a sily F dostaneme
k k i j i F r M 100 . 60 sin 18 , 0 . sin . ) ( ) sin cos (
0
o
+ = + = + = = F l F l l ,
k M 6 , 15
0
+ = .
3. Kolm vzdialenos d nositeky sily n
F
vzhadom na bod 0 sa rovn dke l (obr. C).
Vekos momentu vzhadom na bod 0 je
M
0
= Fd=100 . 0,18 = 18 Nm.
Vektor momentu M
0
je kolm na rovinu nkresu a to v smere hodinovch ruiiek, ie
je zporn. Pomocou vektorovho sinu polohovho vektora r a sily F dostaneme
) cos sin ( ) sin cos (
0
j i j i F r M + = = F F l l ,
k k k k M ) 5 , 13 ( ) 5 , 4 ( ) )( sin )( sin ( ) cos )( cos (
0
+ = + = F l F l ,
k M 18
0
= .

Prklad 1.3
Na stp psob v lane AM sila F poda obrzka.
1. Urte polohov vektor psobiska danej sily.
2. Zapte silu v tvare mnoholena v kartezinskom
sradnicovom systme.
3. Urte moment danej sily vzhadom na zaiatok
sradnicovho systmu.
Dan:
F = 1,2 kN; = 30; l = 1 m; h = 4,5 m; b = 1,5 m


Obr.A

Obr.B

1. Polohov vektor r
A
psobiska sily F vzhadom na bod 0 (zaiatok sradnicovho
systmu) predstavuje telesov uhloprieku hranola s vkou h. Strany podstavy s
y
x
0
z
A
M

h
l
F
r
A
r
Az
r
Ax
r
Ay
H
N
K
x
y
0
z
A
M

h
l
F
r
A
H
n
F
r
Ax
b

N
K
L
B
C D
F
x
F
z
F
y
F
xz

V technickch aplikcich bude vekos momentu sily urovan poda vzahu (1.11) a jeho
orientcia bude urovan poda pravidla pravej ruky.
y
0
z
A
.

h
l
F
b
H
x
M
10
vyjadren na zklade dky ramena HA a uhla , ktor zviera rameno s kolmm
priemetom na os x v rovine rovnobenej s rovinou xy (obr. A).
k j i r ) ( ) ( ) (
0 0 0
z z y y x x
A A A A
+ + = ,
0
0 0 0
= = = z y x ,
k j i k j i r 5 , 0 5 , 4 866 , 0 ) sin ( ) cos ( + + = + + = l h l
A
.
2. Nositeka sily n
F
le na telesovej uhloprieke hranola ABCDKLMN s vkou h (obr. B).
Z danch rozmerov vypotame strany jeho podstavy.
m. 5 , 0 sin AD KN m, 634 , 0 cos MN = = = = = l l b
Na urenie vekosti zloiek sily F
y
a jej kolmho priemetu do roviny xz (F
xz
) je potrebn
vyjadri uhol , ktor je vyjadren z pravouhlho trojuholnka AKM

. , , arctg
, ,
,
, ,
MN KN KM
tg
o
17 10 179 0
179 0
5 4
634 0 5 0
2 2
2 2
= =
=
+
=
+
= =
h h

Potom: = = sin , cos F F F F
xz y
.
Na vyjadrenie zloiek sily F
x
, F
z
potrebujeme vypota uhol, ktor zviera priemet sily
F
xz
s kladnm smerom osi x (resp. so smerom osi z). V obrzku (obr.B) je tento uhol
zaktovan ako uhol v pravouhlom trojuholnku KLM

o
27 , 38 789 , 0
634 , 0
5 , 0
MN
KN
LK
LM
tg = = = = =
Vekosti zloiek sily F
x
, F
z
s vyjadren pomocou vzahov
= = sin , cos
xz z xz x
F F F F .
Zpis sily v tvare mnoholena je
y z x
F F F = F i j k .
Po dosaden predchdzajcich vzahov zskame

. 13 , 0 18 , 1 166 , 0
, 8,27 3 sin ) 0,17 1 sin ( 0,17 1 cos 2 , 1 8,27 3 cos ) 0,17 1 ,2sin 1 (
, sin ) sin ( cos cos ) sin (
k j i F
k j i F
k j i F
=
=
=
o o o o o
F
F F F

3. Moment sily vzhadom na zaiatok sradnicovho systmu je mon vypota priamo
z defincie pre moment sily vzhadom na bod s vyuitm predchdzajcich vsledkov
k j i
k j i
F r M ) ( ) ( ) (
x y y x z x x z y z z y
z y x
z y x
F r F r F r F r F r F r
F F F
r r r + + = = = ,
k j i
k j i
M 769 , 1 196 , 0 005 , 0
0,13 - 1,18 - 0,166
0,5 4,5 0,866 + = = .
11
Vekos momentu je kNm 779 , 1
2 2 2
= + + =
z y x
M M M M .

Prklad 1.4
1. Vypotajte momenty sily F vzhadom na jednotliv sradnicov osi.
2. Vypotajte momenty sily Q vzhadom na jednotliv sradnicov osi.


Obr. A

Obr. B
1. Vpoet momentu sily F vzhadom na os x (obr. A). Najskr je vypotan moment sily
F vzhadom na ubovon bod leiaci na osi x napr. bod 0
j k i k j i F r M bF cF F c b a
F
+ = + + = = ) (
0
.
Moment vzhadom na os x je vyjadren vzahom
0 i i j k i i M M = + = = ] ) [( ) (
0
bF cF
x
.
Nositeka sily n
F
je rovnoben s osou x, preto sila nem otav inok vzhadom na
os x. Bod 0 sa nachdza na osiach x, y a z. Moment sily F vzhadom na bod 0 je pouit
na vpoet momentu sily aj k zvynm dvom sradnicovm osiam. Pre vektor momentu
sily v smere osi y plat
j j j j j k j j M M
y y
M bF bF cF = = + = = ] ) [( ) (
0
.
Vekos momentu sily vzhadom na os mono vypota ako sin vekosti sily a kolmej
vzdialenosti nositeky sily k danej osi. Orientcia momentu je uren pomocou pravidla
pravej ruky.
Nositeka sily n
F
a os z s navzjom mimoben. Vekos momentu sily F vzhadom na
os z poda predchdzajcej defincie je M
z
= cF. Pri pohade z kladnho smeru osi z sila
F ota okolo osi z v smere hodinovch ruiiek, take moment bude zporn. Moment
sily F vzhadom na os z je potom vypotan zo vzahu
k k k k j k k k M M
z z
M cF bF cF = = + = = ] ) [( ) (
0
.
2. Sila Q (obr. B) nem otav inok vzhadom na os x (n
Q
a os x s navzjom rznoben)
a nem otav inok ani vzhadom na os y (n
Q
a os y s navzjom rovnoben).
Nositeka sily n
Q
je mimoben iba s osou z. Kolm vzdialenos mimobeiek je a.
k M aQ
z
+ = .
Moment sily Q vzhadom na os z je potom vypotan pomocou (1.12), t.j.
k k k k k k j j i k k Q r M aQ aQ Q c a
Q z
+ = = + = = ] ) [( ] ) ) [(( ] ) [(

F
z
y
0
x
a
b
c
r
F
i
j
k
n
F
F
z
y
0
Q
x
a
b
c
y
0
Q
x
a
b
c
r
Q
z
i
j
k
n
Q
12
Prklad 1.5
Na pku nstrkovho ka psobia na oboch stranch dve rovnako vek sily F. Vypotajte
vsledn momenty sl vzhadom na os skrutky (bod A) a vzhadom na os kolesa (bod B).
lohu riete ako rovinn.
Dan: a = 0,4 m; b = 0,15 m; F = 200 N

Obr. A

Obr. B

Posuvn inok oboch sl sa navzjom ru
F + (F) = 0.
Vekos momentu od oboch sl vzhadom na os skrutky (bod A) je vypotan ako set
momentov kadej zo sl vzhadom na bod A
Nm ) ( 160 2 = = aF M
A

lohu mono riei aj vyjadrenm vekosti momentu silovej dvojice (sily tvoria silov dvojicu)
Nm ) ( 160 2 = = F a M
A
.
Moment silovej dvojice je kladn, pretoe sily otaj proti smeru hodinovch ruiiek.
Moment od oboch sl vzhadom na os kolesa (bod B)
M
B
= (a + b)F + (a b)F = 2aF = 160 Nm.
Z uvedenho je zrejm, e moment silovej dvojice je rovnak vzhadom na ubovon bod
telesa. Z tohto dvodu sa v technickch aplikcich silov dvojica asto oznauje len smerom
otania a je dan vekosou momentu (obrzok vpravo).

Prklad 1.6
Na oceov platu psob v bode A sila F. Vypotajte moment sily vzhadom na body B a C.
Dan: a = 0,2 m; b = 0,5 m; c = 0,6 m; F = 300 N; = 20

Obr.A

Obr. B
a b
c

F
A
B
C
F
y
F
x
+
x
y
n
F
d
.
r
AB
r
AC
a b
c

F
A
B
C
B
b
a a
A
M
B
b
a a
F
A
F
13
Kee nie je znma kolm vzdialenos d bodu B od nositeky sily n
F
, loha bude rieen
pomocou Varignonovej vety. V slade so zvolenm sradnicovm systmom (obr. B) pre
vekos momentu sily F vzhadom na bod B plat
+ = + = sin cos
B
aF cF aF cF M
y x
,
Nm 6 , 148 20 sin . 300 . 2 , 0 20 cos . 300 . 6 , 0
B
+ = + =
o o
M .
Vektor momentu je kolm na dan rovinu:
B
= + 148,6 M k .
Vpoet momentu sily F vzhadom na bod B pomocou determinantu
k k
k j i
F r M 6 148
0
0 , ) cos sin (
sin cos
AB B
+ = + =

= = cF aF
F F
c a .
Podobne je uren moment sily vzhadom na bod C. Bod C sa nachdza na nositeke zloky
sily F
x
. Zloka sily F
x
nem preto otav inok vzhadom na bod C. Otav inok vznik
len od zloky sily F
y
. Vekos momentu sily F vzhadom na bod C je vyjadren nasledovne:
Nm 8 , 71 20 sin 300 ). 5 , 0 2 , 0 ( sin ) ( ) (
C
= + = + = + =
o
F b a F b a M
y
.
Vektor momentu je taktie kolm na dan rovinu:
C
= 71,8 M k .
Vpoet momentu sily F vzhadom na bod C pomocou determinantu
k k
k j i
F r M 8 71
0
0 0 , ] sin ) ( [
sin cos
) (
AC C
= + =

+ = = F b a
F F
b a .
Prklad 1.7

Na maku eriava psobia sily v lanch. Vypotajte vekos sily
v lane 2 tak, aby vslednica R

=

(R
x
, R
y
) oboch sl mala len vertiklnu
zloku. Zrove vypotajte vekos vslednej sily.
Dan: N
1
= 600 N; = 15; = 45
Vsledok: N
2
= 819,62 N; orientcia pre N
2

R
x
= 0 N
R
y
= 734,85 N; orientcia pre R
y
(+)

Prklad 1.8

Vypotajte moment M
0
od sily F vzhadom na bod 0:
1. pomocou vektorovho sinu pre moment,
2. pomocou vzahu pre vekos vektorovho sinu,
3. pouitm Varignonovej vety.
Dan: F

= 600 N; = 40; l = 2 m; h= 4m
Vsledok: M
0
= 2609,85 k [Nm]

0
h
l

F


N
2
N
1
C
C
14
Prklad 1.9

tyri kolky priemeru 1 mm s upevnen k doske
poda obrzka. Okolo kolkov prechdzaj dve lan
ahan silami o vekosti F
1
= F
2
= F a Q
1
= Q
2
= Q.
Dan: a = 2 mm, b = 4 mm, c = d = 5 mm,
2r = 1 mm, F = 20 N, Q = 35 N
Vypotajte:
1. vekos momentu sl Q
1
, Q
2
vzhadom na bod D,
2. vekos momentu sl Q
1
, Q
2
vzhadom na bod E,
3. vekos momentu sl F
1
, F
2
vzhadom na bod D,
4. vsledn otav inok vetkch sl na dosku M
V
.
Vsledok: 1. M
D
= 140 N.mm; M
D
= 140k 3. M
D
= + 140 N.mm; M
D
= + 140k
2. M
E
= 140 N.mm; M
E
= 140k 4. M
V
= 0

Prklad 1.10
Tuh rmov kontrukcia je zaaen silami F
1
, F
2
. Nositeka
sily n
F1
zviera so sradnicovmi osami x, y, z uhly , , .
Vypotajte:
1. moment od sily F
1
vzhadom na body O, A,
2. moment od sily F
2
vzhadom na body O, A,
3. vsledn moment od oboch sl vzhadom na bod O.
Dan: h = 1 m, l = 0,3 m, d = 0,5 m,
F
1
= 400 N, F
2
= 300 N, = 70, = 30, = 25
Vsledok:
1. M
O
=

535,7i


177,2j

32,9k;

M
A
=

173,2i

177,2j +103,9k
2. M
O
= 150i + 90k; M
A
= 150i + 90k
3. M
V
= 685,7i 177,2j + 57,1k

1.2 STATICK EKVIVALENCIA SILOVCH SSTAV
Mnoina sl psobiacich na ten ist objekt tvor silov sstavu. Kad silov sstavu F
i
,
n i =1 mono veobecne nahradi ekvivalentnou silou R - vslednicou psobiacou v bode 0
a ekvivalentnm momentom M
0
- vslednm momentom vzhadom na bod 0. Veobecne m
silov sstava
posuvn inok
i

= F R , (1.15)
otav inok

= =
i i
i
F r M M
0
0
. (1.16)
Dve silov sstavy s ekvivalentn, ak ich mono nahradi vzhadom na ten ist bod rovnakou
vslednou silou a rovnakm vslednm momentom. Rovnice (1.15) a (1.16) teda definuj zkladn
nahradenie ubovonej silovej sstavy vo vektorovom tvare. Skalrny tvar tchto rovnc
tri zlokov (resp. silov) rovnice:

= =
i i ix x
F F R cos


= =
i i iy y
F F R cos (1.17)


= =
i i iz z
F F R cos

.
.
.
O
A


F
2
F
1
l l
d
h
d
x
n
F1
z
y
E
d
c
y
x c b b
a
a
a
0D
Q
1
F
1

F
2

Q
2

15
tri momentov rovnice silovej sstavy

= = ) (
iy i iz i ix x
F z F y M M


= = ) (
iz i ix i iy y
F x F z M M (1.18)


= = ) (
ix i iy i iz z
F y F x M M

TYPY SILOVCH SSTAV Tabuka 1.1
Typ silovej sstavy Nositeky sl psobiace na objekt Zkladn nahradenie
Veobecn priestorov
silov sstava (VPSS)
* ubovone usporiadan v priestore
* mimobeky

=
=
=
iz z
iy y
ix x
F R
F R
F R

=
=
=
iz z
iy y
ix x
M M
M M
M M

Veobecn rovinn
silov sstava (VRSS)
* ubovone usporiadan v rovine
* rznobeky


= =
= =
0 0 i z
iy y ix x
M M M
F R F R

Rovnoben priestoro-
v silov sstava
(RPSS)
* rovnobene usporiadan v priestore
* rovnobeky s rovnakm sklonom
k sradnicovm osiam x, y, z
(nositeky rovnoben napr. s y)

=
i
F R

=
ix x
M M

=
iz z
M M
Rovnoben rovinn
silov sstava (RRSS)
* rovnobene usporiadan v rovine
* rovnobeky s rovnakm sklonom
k sradnicovm osiam x, y

=
i
F R

=
0 0 i
M M
Centrlna priestorov
silov sstava (CPSS)
* rozloen v priestore prechdzajce
jednm bodom
* vslednica prechdza spolonm
priesenkom nositeliek sl

=
=
=
iz z
iy y
ix x
F R
F R
F R

Centrlna rovinn
silov sstava (CRSS)
* rozloen v rovine prechdzajce
jednm bodom
* vslednica prechdza spolonm
priesenkom nositeliek sl

=
=
iy y
ix x
F R
F R

Priamkov silov
sstava (PSS)
* vetky sily leia na jednej priamke

=
i
F R

Prklad 1.11
V otvoroch B, C, D oceovho driaka s uchyten lan 1, 2, 3 s vekosami osovch sl N
1
,
N
2
, N
3
. Nahrate lan v bodoch B, C, D lanami v bodoch A, E tak, aby sily psobiace v nich
(N
4
, N
5
) mali rovnak inok na driak.
Dan:
N
1
= 3 kN; N
2
= 2 kN, N
3
= 5 kN, = 30; = 45; a = 0,1 m; b = 0,2 m; c = 0,25 m

+
x
y

Obr. A
c
a
a
a
b
B
C
D
E

N
1
N
2
A
N
3
0
a b
B
C
D
E
A

R
R

0
M
0
16

Obr. B

Sily v lanch N
1
, N
2
, N
3
tvoria veobecn rovinn silov sstavu. Najskr pre dan sstavu sl
je vykonan zkladn nahradenie - tvoren vslednicu R psobiacou v bode 0 a vslednm
momentom vzhadom na bod 0 M
0
(obr. A).
V slade so zvolenm sradnicovm systmom s zloky vslednice a vekos momentu
vypotan z rovnc

. ) 2 ( sin cos ) (
, cos
, sin
3 2 1 1 0 0
3 1
2 1

+ + = =
+ = =
+ = =
bN N a c cN N b a M M
N N F R
N N F R
i
iy y
ix x

Odkia po dosaden: Nm 4 , 1504 N; 7598 N; 500
0
= = = M R R
y x
.
Vekos vslednice: N ,4 7614
2 2
= + =
y x
R R R
a jej smer je definovan uhlom: = = = 2 , 86
4 , 7614
500
arccos arccos
R
R
x
R
.
Vsledn moment v zvolenom sradnicovom systme m zporn znamienko. To znamen,
e zmysel otania je v smere hodinovch ruiiek.
Z troch rovnc, ktor mono zostavi pre nahradenie pvodnej silovej sstavy novou sstavou,
, cos cos + = = =
4 5
500 N N F R
ix x


, cos sin ) ( ,
, sin sin
+ = = =
+ = = =

4 5 0 0
4 5
4 1504
7598
cN N b a M M
N N F R
i
iy y

s vypotan tri parametre (vekosti sl N
4
, N
5
a uhol , obr. B), t.j. po prave a dosaden
zskame: = = = 42 59 85 3546 64 5912
5 4
, ; N , ; N , N N .
Pri osovch silch N
4
, N
5
sme dostali kladn znamienko, to znamen, e zvolen orientcia sl
v lanch je sprvna.

c
a
a
a
b
B
C
D
E
A

N
4
N
5

0
c
a
a
a
b
B
C
D
E

N
1
N
2
A
N
3

N
4
N
5

0
17
Prklad 1.12
Pre zaaenie rmu poda obrzka njdite
zkladn nahradenie.

Dan:
F
1
= 500 N; F
2
= 800 N, F
3
= 600 N,
M = 100 Nm,
= 30; = 60;
h
1
= 0,8 m; h
2
= 0,2 m;
l
1
= 0,5 m; l
2
= 0,2 m;
b
1
= 0,4 m; b
2
= 0,3 m.

Dan silov sstava reprezentuje veobecn priestorov silov sstavu - VPSS, ktor mono
nahradi vslednou silou a vslednm momentom k zvolenmu bodu. Za vzan bod je
zvolen zaiatok sradnicovho systmu - bod 0.
Poda tabuky 1.1 mno pre VPSS psa podmienky zkladnho nahradenia, t.j. vekosti
zloiek vslednice s

. N 550 cos sin
, N 433 cos
, N 4 , 280 sin
3 1
1
3 2

= + = =
= = =
= + = =
F F F R
F F R
F F F R
iz z
iy y
ix x

Vektor vslednice R a jej vekos s
k j i R 550 433 4 , 280 + =
N 1 , 754 ) 550 ( ) 433 ( ) 4 , 280 (
2 2 2 2 2 2
= + + = + + =
z y x
R R R R .
Smer nositeky vslednice vzhadom na kladn smer sradnicovch os x, y a z je vyjadren
pomocou smerovch kosnusov, resp. smerovch uhlov
372 , 0
1 , 754
4 , 280
cos =

= =
R
R
x
R

R
= 111,8,
574 , 0
1 , 754
433
cos =

= =
R
R
y
R

R
= 125,
729 , 0
1 , 754
550
cos = = =
R
R
z
R

R
= 43,2.
Vsledn moment psobiaci v bode 0 je uren ako set momentov od jednotlivch sl
vzhadom na sradnicov osi
Nm 9 , 96 cos ) ( sin cos ) (
1 2 1 1 1 3 2 1
= + + + + = =

F l l F h F h h M M M
ix x
,
Nm 3 , 232 ) sin cos (
2
sin
1 3
2
1 1 3
=

+ + = =

F F
b
b l F M M
iy y
,

F
2
M
x
y
z
b
1
l
2
h
1
h
2
h
2
l
1
b
1
b
2
0
F
1
.
z
y
F
3
.
x
z

18
Nm 2 , 42 cos
2
sin ) ( ) (
1
2
1 3 2 1 2 2 1
=

+ + + = =

F
b
b F h h F h h M M
iz z
.
Vektor vslednho momentu a jeho vekos

Nm. 21 , 255 ) 2 , 42 ( ) 3 , 232 ( ) 9 , 96 (
, 2 , 42 3 , 232 9 , 96
2 2 2 2 2 2
0
0
= + + = + + =
+ =
z y x
M M M M
k j i M


Prklad 1.13
Njdite vekos ekvivalentnej sily a polohu jej nositeky, ktorou mono nahradi zaaenie
psobiace na dan nosnk.
Dan: F = 1500 N; q = 100 N.m
-1
; l = 2 m


Obr. A Obr. B
Spojito rozloen zaaenie je v statike nahrdzan vslednicou, ktorej smer a orientcia s
zhodn so smerom spojito rozloenho zaaenia. Psobisko vslednice je v aisku plochy
spojito rozloenho zaaenia a vekos vslednice sa rovn vekosti plochy spojito
rozloenho zaaenia.
V technickej praxi sa nahrdza spojit zaaenie po sekoch znmymi geometrickmi tvarmi.
Spojit zaaenie je v rieenom prpade rozdelen na zaaenie obdnikovho tvaru
a zaaenie trojuholnkovho tvaru (obr. A). Vekos vslednc spojito rozloench zaaen
a vzdialenosti ich nositeliek vzhadom na bod A s (obr. B)

. m 66 , 4
3
2 N 100
2
m, 3
2
N 200
2 2
1 1
=

+ = = =
=

+ = = =
l
l x
ql
Q
l
l x ql Q
Q
Q
(a)
Sily F, Q
1
a Q
2
tvoria rovnoben rovinn silov sstavu, pre ktor mono napsa dve
rovnice zkladnho nahradenia v zvolenom sradnicovom systme

. :
, :
2 Q 2 1 Q 1 R A A
2 1
x Q x Q Rx M M
Q Q F R F R
i
i
= =
= =

(b)
Po prave z rovnc (b) vyplva

. m ,
, . .
. .
N,
Q Q
R
59 0
1800
66 4 100 3 200
1800 100 200 1500
2 2 1 1
2 1
=
+
=
+
=
= + + = + + =
R
x Q x Q
x
Q Q F R

x
y
+
F

x
Q1
Q
1
Q
2
x
Q2
R

x
R
A

q

F

l l/2
Q
1
Q
2
l/2
l/3 2l/3
q

F

l l l
19

Prklad 1.14

Sila F psob na profil v bode A. Nahrate tto silu
ekvivalentnou silou a momentom v bode C.
Dan:
a = 40 mm; b = 96 mm; c = 48 mm; F = 1300 N
Vsledok:
R
x
= 347,4 N (); R
y
= 1252,7 N (+); M
C
= 16,757 Nm ()

Prklad 1.15

Urte vslednicu R danej silovej sstavy a vku h nad bodom
B, v ktorej le nositeka vslednice.
Dan:
l = 600 mm; F
1
= 250 N; F
2
= 300 N; F
3
= 650 N
Vsledok:
R= 100 N; h

= 0,9 m

Prklad 1.16

Urte vsledn moment silovej sstavy psobiacej na
pevn kontrukciu poda obrzka vzhadom na bod 0.
Prierezy a hmotnosti jednotlivch ast kontrukcie
mono zanedba.
Dan:
a = 0,3 m; b = 0,4 m; l = 0,6 m; r = 0,25 m;
F
1
= 600 N; F
2
= 350 N; q = 45 N/m; = 45
Vsledok:
M
0
= 129,82 i 82,2 j + 35,67 k
F
1
F
2
q

r

l

a

b


z

x

y

0

F
1
F
2
F
3
l

l

l

B

a
c
b
b
A
C
B
F
20
1.3 AISKO HMOTNCH OBJEKTOV

aisko hmotnho objektu je mon chpa ako stredisko rovnobench gravitanch sl
viazanch k jeho jednotlivm elementom. Do tohto bodu strediska je sstreden hmotnos
celho telesa, priom posuvn a otav inok sa vzhadom na okolie nezmen.


Obr. 1.7 Obr. 1.8
Pre posuvn inok plat
( )

G
G G d = (1.19)
a pre otav inok plat
( )


G
G r G r d =
T
, (1.20)
kde G = mg je tiaov sila psobiaca v aisku telesa, g je tiaov zrchlenie.
aisko je teda bod, ktorho polohov vektor, resp. jeho sradnice s vyjadren v tvare

=
) (
) (
T
d
d
m
m
m
m r
r , resp.

= = =
) (
) (
T
) (
) (
T
) (
) (
T
d
d
,
d
d
,
d
d
m
m
m
m
m
m
m
m z
z
m
m y
y
m
m x
x . (1.21)
aisko osovo-symetrickch telies le na osi symetrie. aisko stredovo-symetrickch telies
le v strede symetrie.
Pre homognne spojit teles je elementrna hmotnos vyjadren v tvare
dm = dV,
pre homognne plon tvary kontantnej hrbky - h:
dV = hdS,
pre jednorozmern homognne tvary kontantnho prierezu - S:
dV = Sdl,
kde je hustota, V je objem prslunho telesa, S je plocha prslunho tvaru, l je dka
prslunho tvaru.
Poda tvaru objektu, ktorho poloha aiska je hadan, sa vzahy (1.21) upravia.
Polohov vektor aiska diskrtnej sstavy hmotnch tvarov, resp. jeho sradnice maj tvar

=
=
=
n
i
i
n
i
i i
m
m
1
1
T
r
r , resp.

=
=
=
=
=
=
= = =
n
i
i
n
i
i i
n
i
i
n
i
i i
n
i
i
n
i
i i
m
m z
z
m
m y
y
m
m x
x
1
1
T
1
1
T
1
1
T
, , , (1.22)
G
G
1
G
i

G
n
0
x
y
z
r
i
r
n
r
1
r
T
z
dG
r
r
T
x
y
0
m
dm
m
G
21
kde n je poet hmotnch tvarov, ktor tvoria dan sstavu.

Pappus-Guldinove vety
Pappus-Guldinove vety slia na vpoet vekosti povrchu, resp. objemu rotane symetrickho telesa.
1.veta: Plochu povrchu vytvorenho rotciou tvoriacej krivky dky l okolo pevnej osi rotcie
leiacej v rovine krivky (obr. 1.9) mono vypota zo vzahu
l x S

= , (1.23)
kde x
T
je vzdialenos aiska rotujcej krivky dky l od pevnej osi,
je vekos uhla (v radinoch) rotcie krivky okolo danej osi.
Vekos sinu

x je drha, ktor vykonalo aisko krivky pri rotcii okolo osi rotcie.

Obr. 1.9
2.veta: Objem rotane symetrickho telesa vzniknut rotciou plochy S okolo pevnej osi
leiacej v jej rovine (obr. 1.10) a jeho vekos je mon vypota zo vzahu
S x V

= , (1.24)
kde x
T
je vzdialenos aiska rotujcej plochy S od pevnej osi,
je vekos uhla (v radinoch) rotcie plochy okolo danej osi.
Vekos sinu

x je drha, ktor vykonalo pri rotcii aisko plochy.



Obr. 1.10











x
T
y
l
x
0
l
x
0

y
S
y
x
T
x
S
T
0
y
x
T
x
S
0
V

22
Prklad 1.17
Vypotajte sradnice aiska:
1. kruhovho oblka (obr. A)
2. kruhovho vseku (obr. B).
Dan: R;
1
;
2


Obr. A

Obr. B

Obidva geometrick tvary - kruhov oblk aj kruhov vsek je mon vo veobecnosti
povaova za spojit tvary, ktor s definovan v rovine. Z toho vyplva, e poloha ask
oboch tvarov je definovan dvoma sradnicami x
T
a y
T
. Sradnica z
T
je nulov. Na vpoet
ask s vybrat prslun elementy dl (obr. A), resp. dS (obr. B).

Kruhov oblk Kruhov vsek
Vzahy na vpoet sradnc aiska:

=
) (
) (
l
l
T
dl
xdl
x ;

=
) (
) (
l
l
T
dl
ydl
y ,
kde pre oblk plat
, sin , cos = = R y R x
. = Rd dl

Vzahy na vpoet sradnc aiska:

=
) (
) (
S
S
T
dS
xdS
x ;

=
) (
) (
S
S
T
dS
ydS
y ,
kde pre vsek plat
, sin
3
2
, cos
3
2
= = R y R x
.
2
2
= d
R
dS
Sradnice taiska oblka
1 2
1 2
2
sin sin
cos
2
1
2
1


=

R
Rd
d R
x
T
,
1 2
2 1
2
cos cos
sin
2
1
2
1


=

R
Rd
d R
y
T
.
Sradnice taiska vseku
1 2
1 2
2
2 3
2
3
2
2
1
2
2
1
2


=

sin sin
cos
R
d
d R
x
R
R
T
,
1 2
2 1
2
2 3
2
3
2
2
1
2
2
1
2


=

cos cos
sin
R
d
d R
y
R
R
T
.

Uveden vzahy maj veobecn platnos. S pouiten na vpoet sradnc ask
ubovonho kruhovho oblka (kruhovho vseku) s polomerom R, vzhadom na stred
R
x
T
y

2

d
dl
l

x

y
T
T
R
x
y

2

dl

dS
d
T
x
T
y
T
23
oblka (vseku). Uhly
1
a
2
s orientovan uhly od kladnho smeru osi x proti smeru
hodinovch ruiiek. Uhol
1
reprezentuje uhol, pri ktorom oblk (vsek) zana a
2
je uhol,
pri ktorom oblk (vsek) kon.

Prklad 1.18
Pre rovinn plon tvar vypotajte statick momenty k osiam x, y a polohu aiska.
Dan:
a = 30 mm; b = 90 mm; R= 30 mm; r = 20 mm

Rovinn plon tvar je vytvoren stanm plch obdnika, trojuholnka, polkruhu
a odtanm plochy kruhu. V zvolenom sradnicovom systme s vypotan vekosti
jednotlivch plch a sradnice ich aiska, vsledky s uveden v tabuke 1.
Tabuka 1.1
. plochy S
i
[mm
2
] x
Ti
[mm]

y
Ti
[mm]

U
xi
[mm
3
]
*
U
yi
[mm
3
]
**
1 90.60 = 5400 30 45 243 000,00 162 000
2 (60.30)/2 = 900 20 - 10 -9 000,00 18 000
3 30
2
/2 = 1413,7 30 102,7 145 186,99 42 411
4 20
2
= - 1256,64 30 90 -113097,60 -37 699,2
= 6457,06
266 089,39 184711,8
*
U
xi
statick moment i-tej plochy k osi x: U
xi
= y
Ti
S
i
.
** U
yi
statick moment i-tej plochy k osi y: U
yi
= x
Ti
S
i
.
Statick momenty danho rovinnho plonho tvaru k osiam x, y s:

. mm 8 , 184711 U
, mm 39 , 266089 U
3
3
= =
= =

yi y
xi x
U
U

Sradnice aiska celho plonho tvaru s uren zo vzahov (1.22) pre dvojrozmern
teleso, t.j.

[mm]. 2 , 41
06 , 6457
39 , 266089
[mm], 6 , 28
06 , 6457
8 , 184711
T
T
= = =
= = =

i
xi
i
yi
S
U
y
S
U
x

2R
x
y
0
a
r
b
a/3
2R/3
R
x
y
b/2
T
1
0
+
T
2
0
x
y
y
3
0
R
x
y
T
3
x
y
b
0
T
4
R
=
+
C
C C C
24
Pri vpote sradnc aiska polkruhu (plocha 3) je potrebn vyui vzahy (1.21) upraven
pre dvojrozmern teles (obr. A).

Obr. A
=
=
=
sin
cos
d d
y
x
d S

( )
( )
( )
( )
.
3
4
2
d d sin
d
d
, 0
2
R
d d cos
dS
xdS
2
0 0
2
T
2
0
R
0
2
T


= =
=


= =

R
R S
S y
y
x
R
S
S
S
S



Prklad 1.19
Vypotajte polohu aiska objemovho tvaru.
Dan: l =60 mm; h = 25 mm; R = 15 mm; r = 8 mm; t
1
= 10 mm; t
2
= 8 mm
C

C






C C

Objemov priestorov tvar je vytvoren z konenho potu jednoduchch objemovch
tvarov a je vytvoren stanm objemov dvoch kvdrov C + C, objemu polovice valca C,
a odtanm objemu valca C. V zvolenom sradnicovom systme s vypotan vekosti
jednotlivch objemov a sradnice ich ask, ktor s uveden v tabuke 1.



x
y
R
dS
d

d
0
t
1
h

2R

z
x
y
t
1
h

z
x
y
R

t
1
h

r

z
x
y
l

t
1
t
2
x

z

y

2R

l

t
1 t
2
h

R

r

z
x
y
25
Tabuka 1.2
P..
V
i
[mm
3
]
x
Ti

[mm]
y
Ti

[mm]
z
Ti

[mm]
1 =
1
2
t r 2010,62 =
2
1
t
5 = h 25 = R 15
2 =
1
2
2
1
t R 3534,29 =
2
1
t
5
= +
3
4R
h
31,36 = R 15
3 =
1
2Rht 7500,0 =
2
1
t
5 =
2
h
12,5 = R 15
4 =
2
2Rlt 14400,0 = +
2
1
l
t 40 =
2
2
t
4 = R 15

= =
i
V V
23423,67


Vpoet sradnc aiska vzhadom na zvolen sradnicov systm:
smer osi x: 52 , 26
4 3 2 1
4 4 3 3 2 2 1 1
4
1
=
+ + +
+ + +
= =

=
V V V V
V x V x V x V x
V
V x
x
i
i i
T
mm,
smer osi y: 18 9
4 3 2 1
4 4 3 3 2 2 1 1
4
1
, =
+ + +
+ + +
= =

=
V V V V
V y V y V y V y
V
V y
y
i
i i
T
mm,
smer osi z: 0 , 15
4 3 2 1
4 4 3 3 2 2 1 1
4
1
=
+ + +
+ + +
= =

=
V V V V
V z V z V z V z
V
V z
z
i
i i
T
mm.

Prklad 1.20
Vypotajte objem a celkov povrch trojrozmernho telesa pomocou Pappus-Guldinovch viet.
Dan: a =20 mm; b = 30 mm; c = 35 mm; d = 40 mm; h = 40 mm


Pre urenie vekosti objemu danho telesa je potrebn pozna vekos plochy, ktor rotuje
okolo osi y a x-ov sradnicu jej aiska. Vekos plochy lichobenka je vypotan zo vzahu

2
mm 1400 40
2
40 30
2
=
+
=
+
= h
d b
S
a pre x-ov sradnicu aiska plat

=
i
i Ti
T
S
S x
x . (a)
b a
y
0
x
c d
h
S
x
T
T
b
a
y
0
z
x
c d
h
26
Dan plochu vytvorme z troch plch, od obdnika odpotame dva pravouhl trojuholnky.
Pre prehadnos daje potrebn do vzahu (a) udva tabuka 1.3.
Tabuka 1.3
. pl. x
Ti
[mm] S
i
[mm
2
] x
Ti
S
i
[mm
3
]
2
a c d
a
+
+
(d + c a)h
1
47,5 2 200
104 500
3
a c
a

+
2
) ( h a c

2
25 300
7 500
n b a
3
2
+ +
2
nh


3
66,66 500
33 330
x
T
= 45,45 mm
1 400 63 670
Objem telesa vzniknut rotciou danej plochy je

3
400063 1400 48 45 2 mm . , . = = =

S x V .
Na urenie vekosti povrchu danho telesa potrebujeme pozna vekos krivky, ktor rotuje
okolo osi y a x-ov sradnicu jej aiska.
Tabuka 1.4
P.. x
Ti
[mm] l
i
[mm] x
Ti
l
i
[mm
2
]
2
a c
a

+
2 2
) ( a c h +
1
27,5 42,72
1174,8
2
d
c +
d
2
55 40
2 200

2
n
b a + +
2 2
n h +
3
62,5 47,17
2 948,13
2
b
a +
b
b a
y
0
x
c d
h
S
x
T
T
1
3
2
4
n = c+ d a b

4
35 30
1 050

159,89 7372,93
Pre x-ov sradnicu aiska krivky plat (vi. tabuka 1.4): mm ,11 46 = =

i
i Ti
T
l
l x
x .
Povrch telesa, ktor vznikne rotciou danej krivky je

2
89 159 2 mm 46323 , . 46,11 . = = =

l x S .

b a
y
0
x
c d
h
S
x
T
T
1
2
3
n = c+ d a b
n
27
Prklad 1.21
Vypotajte polohu aiska podloky kontantnej
hrbky vzhadom na zvolen sradnicov systm.
Dan: r = 2 mm, R = 8 mm, l = 5 mm,
d
1
= 4 mm, d
2
= 3 mm,
h
1
= 5 mm, h
2
= 12 mm
Vsledok:
x
T
= 18 mm; y
T
= 8,34 mm.

Prklad 1.22
1. Vypotajte vekos povrchu pla S tvaru, ktor
vznikol rotciou danej krivky okolo osi y o uhol .
2. Vypotajte objem V a hmotnos m tvaru, ktor
vznikol rotciou danej plochy okolo osi y o uhol
. Pri rieen aplikujte Pappus-Guldinove vety.
Dan: r = 30 mm; h
1
= 70 mm; = 270;
h
2
= 80 mm; = 7850 kg.m
-3

Vsledok:
1. x
T
= 40,74 mm; l
i
= 362,56 mm; S = 695,76 cm
2

2. x
T
= 33,08 mm; S
i
= 64,07 mm
2
; V = 0,998 dm
3
;
m = 7,84 kg.





x
y
r r
r
h
1
h
2

l l R
2R
d
2
h
1
h
1
R
h
2
d
1
d
2
2r
2r
x
y
28
AISK VYBRANCH LINERNYCH, PLONCH A OBJEMOVCH
TVAROV Tabuka 1.5
tvar Geometrick tvar x
T
y
T
l / S / V
tvrkrunica
r

2
r
2


polkrunica

0
r

2

r
vsek z krunice

sin
r
0
r 2
trojuholnkov
plocha

b
3
2
h
3
1
bh
2
1

tvrkruhov
plocha
r
3
4

2
4
r


polkruhov
plocha

0

r
3
4


2
2
r


tvrelipsov
plocha
a
3
4
ab
4


polelipsov plocha


0
b
3
4

ab
2


kruhov vsek

3
sin 2r

0
2
r
polgua

r
8
3

0
3
3
2
r

kue

h r
2
3
1

ihlan

h
4
1

0
abh
3
1

0
T T y
T
0
x
T

r
r
0
r

T
x
T
b

T
x
T
0
h
h
a
0
x
T
r


0
x
T
r


0
T
y
T
0
x
T

a
T
b
a
T
y
T

x
T

b
h
0


0
x
T
r


0
T
T
y
T
0
x
T

r
r
29
1.4 ROVNOVHA HMOTNCH OBJEKTOV
Hmotn objekt v rovine alebo v priestore me by:
von - nezviazan s inmi objektmi,
viazan - zviazan s inmi objektmi prostrednctvom vzieb.
Stupne vonosti hmotnho objektu - poet nezvislch parametrov jednoznane definujcich
polohu objektu. Zrove vyjadruj poet jeho monch nezvislch pohybov (posunut, otoen).
Vzby - kontrukn prvky viauce objekt na zkladn rm (vonkajie vzby) alebo na in
objekty (vntorn vzby), ktor s nm vytvraj sstavu objektov. Vzby zabrauj niektorm
pohybom objektu - zniuj jeho pohyblivos. V smeroch, v ktorch vzby obmedzuj pohyb
objektu, vznikaj vzbov reakcie. Vzbov reakcie s sily, ktor s vypotan z rovnc
rovnovhy silovch sstav psobiacich na hmotn objekt.
Nahrdzanie vzieb viazanch hmotnch objektov vzbovmi reakciami nazvame metdou
uvonenia. Na uvonen hmotn objekt psob sstava sl, ktor reprezentuj zaaujce sily
a prslun vzbov reakcie.
Vzbov zvislos viazanho objektu n vypotame zo vzahu:
n = n
v
n
o
, (1.25)
6 ,... 2 , 1 ,
1
= =

=
j jr n
n
j
j o
,
kde n je poet stupov vonosti viazanho hmotnho objektu,
n
v
- poet stupov vonosti vonho hmotnho objektu,
n
o
- poet stupov vonosti odobratch vzbami (vntornmi alebo vonkajmi),
j - poet stupov vonosti, ktor odober prslun vzba,
r
j
- poet vzieb odoberajcich j stupov vonosti.
Pri rieen mu nasta prpady:
n = 0 - loha je staticky urit a tvarovo urit
Statick uritos - jednoznan matematick rieenie - je mon zostavi presne toko
rovnc rovnovhy, koko je neznmych vzbovch reakci.
Tvarov uritos - pohyblivos objektu - pre n = 0 objekt je nepohybliv, vzbami s
odobrat vetky stupne vonosti pohybu objektu.
n > 0 - loha je staticky preuren a tvarovo neurit
Statick preurenos - je mon zostavi viacej rovnc rovnovhy, ako je neznmych
vzbovch reakci.
Tvarov neuritos - objekt sa me pohybova.
n < 0 - loha je staticky neurit a tvarovo preuren
Statick neuritos - je mon zostavi menej rovnc rovnovhy, ako je neznmych
vzbovch reakci. Tieto lohy sa nedaj metdami statiky riei. Daj sa riei pridanm
deformanch podmienok k podmienkam rovnovhy.
Tvarov preurenos - objekt sa neme pohybova - vzby odoberaj viacej stupov
vonosti, ako je potrebn.
30
POET STUPOV VONOSTI VONCH HMOTNCH OBJEKTOV
A ZODPOVEDAJCA SILOV SSTAVA PSOBIACA NA OBJEKT Tabuka 1.6

Hmotn
objekt
n
v

Mon
pohyby
n Silov sstava Rovnice rovnovhy
bod
v rovine
2

2 posunutia

=
=
=
2
1
2
j
j o
o
jr n
n n

centrlna
rovinn
silov sstava
(CRSS)

=
=
0
0
iy
ix
F
F

bod
v priestore
3

3 posunutia

=
=
=
3
1
3
j
j o
o
jr n
n n
centrlna
priestorov
silov sstava
(CPSS)

=
=
=
0
0
0
iz
iy
ix
F
F
F

teleso
v rovine
3

2 posunutia
1 otoenie

=
=
=
3
1
3
j
j o
o
jr n
n n
veobecn
rovinn
silov sstava
(VRSS)

=
=
=
0
0
0
0 i
iy
ix
M
F
F

teleso
v priestore
6

3 posunutia
3 otoenia

=
=
=
6
1
6
j
j o
o
jr n
n n
veobecn
priestorov
silov sstava
(VPSS)

=
=
=
0
0
0
iz
iy
ix
F
F
F

=
=
=
0
0
0
iz
iy
ix
M
M
M


x
y
z
x
y
x
y
z
x
y
31
NIEKTOR DRUHY ROVINNCH VZIEB A ICH NAHRADENIE
VZBOVMI REAKCIAMI Tabuka 1.7
Druh vzby Zobrazenie vzby
Uvonenie
telesa
Poet odobratch stupov
vonosti
Neznme vzbov reakcie
lano, prt

j = 1
N
sila psob iba v smere osi
lana, prta
von opretie
dotykov vzba
(hladk povrch)


j = 1
N
sila m smer normly ku
kontaktnmu povrchu
von opretie
dotykov vzba
(drsn povrch)

mykov j = 1
N, T = fN
f - faktor mykovho trenia
valiv j = 2
N , T
posuvn vzba
krtka objmka
(hladk povrch)

j = 1
N
N - sila v smere normly
ku kontaktnmu povrchu
posuvn
rovinn rotan
kb


j = 1
N
sila m smer normly ku
kontaktnmu povrchu
rovinn rotan
kb


j = 2
R
x
, R
y
nhrada dvoma zlokami -
napr. R
x
, R
y
posuvn vzba
kulisa
(hladk povrch)





j = 2
N, M
N - normlov sila,
M - moment zabraujci
otaniu v rovine vzby
votknutie
pevn spojenie

j = 3
R
x
, R
y
, M
R
x
- axilna sila
R
y
- priena silu
M - moment zabraujci
otaniu v rovine vzby
zvar
schematick
znaka
schematick
znaka
schematick
znaka
T
R
N
normla
kulisa
schematick
znaka
normla
krtka objmka
schematick
znaka

N

lano, prt
N
dotynica
normla
dotynica
normla
R
y
R
x
R

N
R
x
R
y
M
normla
N
normla
N
M
32
NIEKTOR DRUHY PRIESTOROVCH VZIEB A ICH NAHRADENIE
VZBOVMI REAKCIAMI Tabuka 1.8
Druh vzby Zobrazenie vzby
Uvonenie
telesa
Poet odobratch stupov
vonosti
Neznme vzbov reakcie
von opretie
dotykov vzba
(hladk povrch)


j = 1
N
sila m smer normly ku
kontaktnmu povrchu
valec alebo koleso
s bonou vzbou
(koleso-koajnica)


j = 2
N, P
N - sila v smere normly ku
kontaktnmu povrchu
P - bon reakcia kolm na
rovinu kolesa
radilne loisko
(tzv. krtke
vedenie)

j = 2
R
y
, R
z
vzba zabrauje pohybu

v rovine kolmej na os
nhrada dvoma zlokami -
napr. R
y
, R
z

2 2
y y
R R R + =
priestorov
sfrick kb


3
R
x
, R
y
, R
z

nhrada tromi zlokami -
napr. R
x
, R
y
, R
z

radilno-axilne
naklpacie loisko


3
R
x
, R
y
, R
z

nhrada tromi zlokami -
napr. R
x
, R
y
, R
z

votknutie
(pevn spojenie)


6
R
x
, R
y
, R
z
, M
x
, M
y
, M
z

R
x
- axilna sila
R
y
, R
z
- priene radilne sily
M
x
, M
y
, M
z
- momenty
zabraujce rotcii okolo
sradnicovch os
x
R
y

R
z

y
z
x
schematick
znaka
y
x
y
z
x
R
x
R
y
R
z
x
y
z
x
y
z N
x
y
z
x
y
z
N
P
x
y
z
x
y
z
R
x
R
y
R
z
M
x

M
y

M
z

R
x
R
y
R
z
y
z
x
33

Prklad 1.23
Objmka hmotnosti m je veden po ikmej dokonale hladkej tyi. V rovnovnej polohe je
objmka dran lanom vedenm okolo dokonale hladkho kolka v bode A. Vypotajte silu
v lane a reakciu od tye vzhadom na objmku.
Dan: m = 2 kg; = 30; =45


Obr. A
Rozmery objmky s zanedban. Na zklade toho me by objmka nahraden hmotnm
bodom A (obr. A). Na objmku psob tiaov sila, ktorej vekos je
G = mg = 2 kg . 9,81 m.s
-2
= 19,62 N.
Rozbor vzieb, v ktorch je objmka uloen - lano odober objmke 1 stupe vonosti
pohybu a medzi objmkou a tyou je vzba odoberajca taktie 1 stupe vonosti pohybu.
Poda vzahu (1.25) je uren statick uritos
0 ) 0 . 3 0 . 2 2 . 1 ( 2
3
1
= + + = = =

= j
j v o v
jr n n n n ,
t.j. loha je staticky aj tvarovo urit.
Vzhadom na silov inky mono kontatova, e na objmku psob v bode A centrlna
rovinn silov sstava.
Metdou uvonenia s vzby odstrnen a nahraden prslunmi vzbovmi reakciami.
Vzbov reakcie s sily, ktorch vekos hadme. Orientcia neznmych vzbovch reakci
je zvolen ubovone. Rovnice rovnovhy pre CRSS (obr. A)

. 0 cos cos : 0
, 0 sin sin : 0
2 1
2
= + =
= =

G R N N F
N R F
iy
ix

Kee medzi kolkom a lanom je dokonale hladk vzba, pre vekosti sl v lane plat: N
1
= N
2
.

N. ,
sin
sin
N, ,
tg
sin
cos
52 57
67 40
1
2
2 1
=


=
=

+
= =
N
R
G
N N


G
A 0


R
N
1
N
2
x
y
m
A


34
Vzhadom na kladn znamienka vo vsledkoch vyplva, e orientcia vzbovch reakci bola
zvolen sprvne.

Prklad 1.24
Doska trojuholnkovho tvaru je zavesen pomocou troch ln na hku poda obrzka.
Tiaov sila dosky psob v bode 0. Vypotajte sily v lanch.
Dan: G = 3,6 kN; a = 0,8 m; b = 1,6 m; c = 2,4 m


Obr. A

Obr. B

Pomocou metdy uvoovania s inky ln nahraden vzbovmi reakciami, ktorch
nositeky s toton s osami ln, priom orientcie sl - vzbovch reakci s ubovone
zvolen. Nositeky vetkch sl (N
1
, N
2
, N
3
, G) sa pretnaj v bode D (obr. A), ie tvoria
centrlnu priestorov silov sstavu.
Kad silu rozlome do prslunch zloiek v smere os x, y, z. Potrebn uhly s vyjadren
z geometrie (obr. B)
c
b
= tg , 69 , 33 arctg = =
c
b

c
2 2
a a
tg
+
= , 24 , 25
a a
arctg
2 2
=
+
=
c

N
1x
N
1z
N
1xz
.


G

x

z

a

a

a

b

D
0
N
1
N
2
N
3
N
3y
N
3x


N
1x
N
1y
N
1z
.



y

x

y

D
0
N
2
z

N
2y
N
2x
N
2z
G

x
A
y

z

a

a

a

b

B
C
D
c

0
N
1
N
2
N
3
G

x

A
y

z

a

a

a

b

B
C
D
c

0
35
a
a
= tg . 45 1 arctg = =

Sily v lanch s vypotame z rovnc rovnovhy.

, 0 : 0
, 0 : 0
, 0 : 0
2 1
3 2 1
3 2 1

= + =
= + + + =
= + + =
z z iz
y y iy
x x x ix
N N F
G y N N N F
N N N F


. 0 sin ) sin ( sin ) sin ( :
, 0 cos cos cos :
, 0 sin cos ) sin ( cos ) sin ( :
2 1
3 2 1
3 2 1
= +
= + +
= +
N N z
G N N N y
N N N x

(c)
(b)
(a)

Rieenm sstavy rovnc (a, b ,c) dostaneme
N
1
= N
2
= 1326,65 N; N
3
= 1442,22 N.
Vzhadom na kladn znamienka vo vsledkoch vyplva, e orientcia sl v lanch bola
zvolen sprvne. Lan s namhan ahom.

Prklad 1.25
Tuh kontrukcia je upevnen k zkladovmu rmu pomocou rovinnho rotanho kbu
(bod A) a posuvnho rovinnho rotanho kbu (bod B). Kontrukcia je zaaen
rovnomernm spojitm zaaenm q, sstredenou silou F a momentovm inkom M.
Poste statick uritos danho objektu a vypotajte vzbov reakcie v bodoch A a B.
Dan: F = 10 kN; = 30; h = 3 m; l
1
= 1 m; l
2
= 2,5 m; M = 20 kNm; q = 5 kNm
-1


Obr. A Obr. B

Rovinn rotan kb v bode A odober rmu 2 stupne vonosti pohybu, posuvn rovinn
rotan kb v bode B odober 1 stupe vonosti pohybu (obr. A).
Statick uritos pre dan teleso v rovine je
0 ) 0 . 3 1 . 2 1 . 1 ( 3 3
3
1
= + + = = =

= j
j o v
jr n n n .
loha je staticky a tvarovo urit. Metdou uvonenia s odstrnen vzby a zrove s
nahraden prslunmi vzbovmi reakciami (obr. B). Orientcia vzbovch reakci je
zvolen ubovone. Na uvonen teleso psob veobecn rovinn silov sstava, pre ktor je
mon zostavi tri rovnice rovnovhy.
M
A
B
q
F
h
l
1 l
2

1
2
M
q
F
h
l
1
l
2

F
x
F
y
Q

B

A
x
A
y
x
Q
x
y

M
A
B
q
F
h
l
1 l
2

36
V slade so zvolenm sradnicovm systmom je zaaujca sstreden sila rozloen na
zloky F
x
a F
y
, pre ktor plat
= cos F F
x
, = sin F F
y
. (a)
Spojit zaaenie nahrdzame sstredenou silou, ktorej psobisko je v aisku plochy
spojitho zaaenia a vekos sily sa rovn vekosti plochy spojitho zaaenia

2
2
Q
l
x = ,
2
.l q Q = . (b)
Rovnice rovnovhy s

. :
, :
, :
A

= + + =
= =
= + =
0 0
0 0
0 0
2
1
2
M Q l F h F h B M
Q F A F
B F A F
l
y x i
y y iy
x x ix
(c)
Po prave rovnc (c) a dosaden zadanch hodnt a vzahov (a), (b) zskavame

kN. 12 , 3
kN, 5 , 17
kN, 54 , 5
1
2
2
= =
= + =
=
+ +
=
x x
y y
x y
l
F B A
Q F A
h
h F l F Q M
B
(d)
Kladn znamienko vo vsledkoch znamen, e voba orientcie reakcie bola sprvna.
Zporn znamienko vo vsledku znamen, e orientcia sily bola zvolen nesprvne a reakcia
v skutonosti psob opane (prpad reakcie A
x
).
Vekos vslednej reakcie A poda (1.3) je:
( ) kN 78 17 5 17 12 3
2 2 2 2
, , , A A A
y x
= + = + = .
Smerov uhol
A
nositeky vslednej reakcie A na zklade skutonej orientcie zloiek A
x
,
A
y
v zvolenom sradnicovom systme le v druhom kvadrante:
= =

= = 37 100 18 0
78 17
12 3
cos
A A
, ,
,
,
A
A
x
.

Prklad 1.26
Votknut zakriven nosnk je zaaen poda obrzka. Poste statick uritos danho
objektu a vypotajte vzbov reakcie vo votknut (bod A).
Dan: F = 500 N; = 45; h = 3 m; r = 1 m; M = 100 Nm; q = 200 Nm
-1



Obr. A Obr. B
M
A
q
F
h
r

r

M
A
q
F
h
r

r
3
x
y

M q
F
2h/3
r


r

h/3
Q
A
x
A
y
M
A
F
x
F
y

37
Votknutie v rovine odober objektu 3 stupne vonosti pohybu (obr. A). Vzbov zvislos pre
teleso v rovine je dan nasledovne:
0 ) 1 . 3 0 . 2 0 . 1 ( 3 3
3
1
= + + = =

= j
j
jr n .
loha je staticky urit. Nosnk uvonme tak, e votknutie odstrnime a nahradme ho
prslunmi vzbovmi reakciami (orientcia vzbovch reakci je zvolen ubovone). Na
nosnk psob veobecn rovinn silov sstava.
V slade so zvolenm sradnicovm systmom je zaaujca sstreden sila rozloen na
zloky F
x
a F
y
, pre ktor plat:
= cos F F
x
, = sin F F
y
. (a)
Spojit zaaenie nahrdzame sstredenou silou
2
qh
Q = . (b)
Rovnice rovnovhy potom s

. 0 ) cos ( ) sin ( : 0
, 0 : 0
, 0 : 0
3
2
A

= + + + =
= =
= + =
h Q r r F r h F M M M
F A F
Q F A F
y x A i
y y iy
x x ix
(c)
Dosadenm vzahov (a), (b) a zadanch hodnt do rovnc (c) s vzbov reakcie
Nm 1 , 7 M N; 6 , 353 N; 6 , 53
A
= + = =
y x
A A .
Vzbov reakcia A
x
v skutonosti psob opane. Vekos vslednej reakcie v bode A
N 6 , 357 ) 6 , 353 ( ) 6 , 53 (
2 2 2 2
= + = + =
y x
A A A .
Smerov uhol
A
nositeky vslednej reakcie A na zklade skutonej orientcie zloiek A
x
,
A
y
v zvolenom sradnicovom systme le v prvom kvadrante

o
62 98 15 0
6 357
6 53
, ,
,
,
cos = =

= =
A
x
A
A
A
.

Prklad 1.27
Bremeno hmotnosti m je pomocou lana vedenho cez dokonale hladk kladku, prichytenho
k tuhmu nosnku v bode C. Nosnk je zaaen silou F a uloen vo vzbch v bodoch
A a B. (vzba A - posuvn rovinn rotan kb, vzba B - thla ty - prt). Poste statick
uritos danho nosnka a vypotajte vzbov reakcie.
Dan: F = 3 500 N; = 50; l
1
= 0,4 m; l
2
= 0,6 m; l
3
= 0,9 m; m = 70 kg


A
B
F
l
1
l
2

l
3
C



m

A
B
F




C
R
A
R
B
N

N
N

G
N
m

x

y


l
1
l
2
l
3
38
Posuvn rovinn rotan kb (bod A) a ty (bod B) odoberaj po jednom stupni vonosti
pohybu. V osi lana (v bode C) psob vzhadom na hladk kladku sila, ktorej vekos je
rovnak ako tiaov sila psobiaca na bremeno (N = G = mg = 686,7 N).
Statick uritos pre teleso v rovine je dan nasledovne
1 ) 0 . 3 0 . 2 2 . 1 ( 3 3
3
1
= + + = =

= j
j
jr n .
Z vsledku vyplva, e loha je staticky preuren. Silov sstava, ktor psob na tuh
nosnk, je rovnoben rovinn silov sstava, pre ktor platia dve rovnice rovnovhy.
Vzhadom na typ silovej sstavy je teleso v rovnovhe. Z toho vyplva, e dve silov rovnice
rovnovhy vzhadom na typ silovej sstavy mono zapsa len pomocou jednej rovnice rovnovhy.

0, cos cos cos cos : 0
0, sin sin sin sin : 0
B A
B A
= + + =
= + + =

R N F R F
R N F R F
yi
xi


0. : 0
B A
= + + + =

R N F R F
i
(a)
Momentov rovnica rovnovhy vzhadom na bod B:
0 cos cos cos ) l (l : 0
3 2 A 2 1 B
= + + + =

N l F l R M . (b)
Rieenm sstavy dvoch rovnc (a, b) s dvoma neznmymi zskame vekosti reakci
N 27 , 5 9 N; 2718,03
B A
+ = + = R R .
Vpotom boli zskan kladn znamienka pri vzbovch reakcich, o znamen, e ich
orientcia bola zvolen sprvne. Ty v bode B je namhan tlakom.

Prklad 1.28
Lomen hriade je pripevnen k rmu radilnym loiskom (B), radilno-axilnym loiskom
(A) a prtom (C) leiacim v osi rovnobenej s osou x. V bode D je na hriade pripevnen
doska s rozmermi hhd. Os hriadea prechdza stredom dosky. Vypotajte vekosti vzbovch
reakci, ak je hriade zaaen danou sstavou sl. Tia hriadea a dosky neuvaujte.
Dan:
F
1
= 15 kN; F
2
= 13 kN; Q
1
= Q
2
= Q

= 10 kN; = 20; = 30; = 70,
a = 0,5 m; c = 0,25m; d = 0,1m; h = 0,4m; l = 1,2 m;
, , s smerov uhly nositeky sily F
2
so sradnicovmi osami x, y, z


Obr.A
D
C

A
x
h
a
c
d
h
a
z
y
x

F
2
F
1
Q
1
Q
2
l
D
A
y
A
z
B
x
B
y
D
m
B
C
A
h
a
c
d
h
a
z
y
x

F
2
F
1
Q
1
Q
2
l
D
39
Radilne loisko (B) odober dva stupne vonosti pohybu, radilno-axilne (A) tri stupne
vonosti a prt (bod C) jeden stupe vonosti pohybu. Statick uritos pre teleso v priestore
0 ) 1 . 3 1 . 2 1 . 1 ( 6 6
6
1
= + + = =

= j
j
jr n .
loha je staticky aj tvarovo urit. Po uvonen s inky vzieb nahraden zodpovedajcimi
vzbovmi reakciami (obr. A). Rovnice rovnovhy pre veobecn priestorov silov sstavu
psobiacu na uvonen objekt v zvolenom sradnicovom systme
0 cos : 0
2
= + + + =

C A B F F
x x ix
, (a)
0 cos : 0
1 2
= + + =
y y iy
A F B F F , (b)
0 ) ( cos : 0
1 2 2
= + + =

Q Q A F F
z iz
, (c)
0 A ) ( ) ( : 0
1
= + + + + =

hQ c l F a c l cB M
y y ix
, (d)
0 . ) ( ) ( cos : 0
2
= + + + + + + =

h Q A c l C a c l cB aF M
x x iy
, (e)
0 - cos : 0
2
1
2
2
= + =

C F aF M
h h
iz
. (f)
Sily Q
1
a Q
2
tvoria silov dvojicu. Posuvn inok je nulov a otav mono vyjadri
nasledovnm momentom silovej dvojice
j i M hQ hQ + =
Q
.
Vekos momentu sa rovn sinu vekosti sily a kolmej vzdialenosti nositeliek
2
2 2
Qh h h Q M
Q
= + = .
Rieenm rovnice (f): C = 13,146 kN.
Rieenm rovnice (c): A
z
= 4,446 kN.
Rieenm rovnc (b, d): A
y
= +14,429 kN; B
y
= 10,687 kN.
Rieenm rovnc (a, e): A
x
= +8,733 kN; B
x
= 7,803 kN.
Zporn znamienka vo vsledkoch znamenaj nesprvne zvolen orientciu vzbovch
reakci, v skutonosti reakcie so zpornmi znamienkami psobia opane, ako boli zvolen.
40
Prklad 1.29
Dva nosnky profilu I s zvaren a uchyten pomocou troch ln. Vypotajte vekos
uhlu a vzdialenos d, v ktorej psob sila F, aby bola kontrukcia v rovnovnej polohe
a vetky tri lan sa nachdzali v rovinch rovnobench s rovinou y-z a mali rovnak
sklon od zvislej osi. Nosnky AB, OC maj hmotnos m
1
, m
2
. aisko nosnka OC je vo
vzdialenosti h od osi y.
Dan: l
1
= 1200 mm; l
2
= 800 mm; l
3
= 1 400 mm; h = 725 mm;
m
1
= 72 kg; m
2
= 50 kg; F = 200 N.

Lan odoberaj po jednom stupni vonosti pohybu. Statick uritos pre teleso v priestore je
3 ) 3 . 1 ( 3 6
6
1
= = =

= j
j
jr n .
loha je staticky preuren a tvarovo neurit. Znamen to, e ak chceme, aby objekt bol
v rovnovhe, musia by z rovnc rovnovhy uren vekosti niektorch prdavnch parametrov.
V tomto prpade je to vzdialenos d nositeky sily F od osi y a uhol sklon ln od zvislej osi.
Lan s odstrnen a nahraden zodpovedajcimi vzbovmi rekciami. Na objekt psob
veobecn priestorov silov sstava, pre ktor je mon zostavi 6 rovnc rovnovhy.
Vekosti tiaovch sl psobiacich v aiskch jednotlivch nosnkov

N. 5 , 490
N, 32 , 706
2 2
1 1
= =
= =
g m G
g m G

V slade so zvolenm sradnicovm systmom s zloky jednotlivch sl v lanch

. sin , sin , sin
, cos , cos , cos
C C B B A A
C C B B A A
N N N N N N
N N N N N N
z z z
y y y
= = =
= = =
(a)
Zostavme sstavu rovnovnych rovnc v zvolenom sradnicovom systme
0 0 : 0 = =
xi
F , (b)
0 : 0
2 1 C B A
= + + =

G G N N N F
y y y yi
, (c)
0 : 0
C B A
= + + =

F N N N F
z z z zi
, (d)
0 : 0
C 3 2
= + =
y xi
N l hG M , (e)
0 : 0
A 1 B 2
= + =

dF N l N l M
z z yi
, (f)
0 : 0
A 1 y B 2
= =
y zi
N l N l M . (g)
x
y
z
O
l
1
F

d
l
3
h

l
2
G
1
G
2

N
A
N
B
N
C
x
y
z
l
1
l
2

l
3
A

B
C
F
d
0
41
Vzhadom na to, e v smere osi x nepsob iadna sila, je zskan sstava piatich rovnc
s piatimi neznmymi: N
A
, N
B
, N
C
, d, .
Do rovnc (c) a (g) dosadme vzahy (a). Rovnicu (c) vynsobme vrazom sin

a rovnicu
(d) vynsobme vrazom cos

a obe rovnice stame:

= + +
= + +
, 0 ]cos F - sin sin sin [
, 0 ) sin ]( cos cos cos [
C B A
2 1 C B A
N N N
G G N N N


. 9,49 tg
, cos sin ) ( 0 cos sin sin
2 1
2 1 2 1
o
= =
= + = +
+G G
F
F G G F G G

Z rovnice (e) N. 52 , 257
cos
N 254 cos
C
C C C
= = = =
y
y
N
N N N
Z rovnice (g)
2
1
A B
l
l
N N = , po dosaden do rovnice (d) vyplva
. N 2 , 382
) (
2 1
2 C
sin
A
=
+

=
l l
l N
N
F

Dosadenm N
A
sptne do predchdzajceho vzahu dostaneme
N. 3 , 573
B
= N
Nakoniec z rovnice (f) vyjadrme vzah na vpoet vzdialenosti d pre nositeku sily F.
0
sin sin
2 B 1 A
=

=
F
l N l N
d .
To znamen, e nositeka sily F mus psobi v smere osi z, aby bol objekt v rovnovhe
a lan zostali v rovine y-z.
Prklad 1.30

Objmka hmotnosti m sa me pohybova po hladkej tyi. V danej
polohe je v rovnovnej polohe udriavan bremenom hmotnosti m
1
,
ktor je zavesen na lane vedenom cez hladk kladku. Vypotajte
vku h, v ktorej je objmka v rovnovhe.
Dan: m = 20 kg, m
1
= 25 kg, a = 1,5 m.
Vsledok: h = 2 m

Prklad 1.31
Bremeno hmotnosti m je zavesen na nehmotnom
lane, ktor je uchyten vo vrchole V. Vo vrchole
s spojen tri tuh prty rovnakho prierezu a
zanedbatenej hmotnosti. Vypotajte vekos sl,
ktormi s jednotliv prty namhan.
Dan:
m = 25 kg; l = 1 m; d = 1,5 m; h = 1,8 m
Vsledok:
prt 3 je namhan na ah N
3
= 319,26 N,
prty 1,2 s namhan na tlak N
1
= N
2
= 122,83 N
x
y
z
0
h
l
l
A
B
C
V
m
d
C
C
C
m
m
1
h
a
42

Prklad 1.32

Nosnk je uloen a zaaen poda obrzka. Poste
jeho statick uritos a vypotajte vzbov reakcie vo
vzbch A, B.
Dan:
F = 300 N; M = 50 Nm; = 30; = 40; l = 0,5 m;
R = 0,2 m; q
1
= 90 N.m
-1
; q
2
= 140 N.m
-1

Vsledok:
n = 0;
x
= 109,291 N ()

y
= 271,878 N (+)

; B

= 234,162 N (N)

Prklad 1.33

Nosnk zaaen tiaovou silou G, spojitm zaaenm
q a dvojicou sl F a pomocou lana (bod A) a posuvnho
rovinnho rotanho kbu (B) je pripojen k zkladovmu
rmu. Poste statick uritos a vypotajte vzbov reakcie.
Dan:
F = 100 N; G = 710 N; l = 1,5 m; q = 120 Nm
-1

Vsledok:
n = 1, na rm psob rovinn rovnoben silov sstava
loha je staticky urit;
R
A
() = + 510 N; R
B
() = + 560 N

Prklad 1.34

Rmov kontrukcia je votknut v bode A a zaaen
poda obrzka. Hmotnos kontrukcie je m = 150 kg.
Vypotajte reakcie vo votknut.
Dan:
F
1
= 650 N; F
2
= 500 N; q =120 Nm
-1
; M = 500 Nm
l
1
= 0,6 m; l
2
= 0,3 m; l
3
= 0,9 m; h
1
= 0,4 m;
h
2
= 0,7 m; h
3
= 1,3 m; h
4
= 0,6 m; = 50; = 45
Vsledok:
x
T
= 0,102 m;
A
x
= 28,26 N (); A
y
= 851,48 N (^);
M
A
= 558,77 Nm ()




F
1
l
1
l
2
l
3
F
2
q





h
1
h
2
h
4
h
3
A 0
x
y
M

A
B
q

l

l

l

l

F
F
G
M
A
B
q
1
h
l

r


F

r

q
2
43
Prklad 1.35
Lomen nosnk (AD) spolu s pstenom tvoria tuh teleso uloen vo vzbch v bodoch A
(priestorov kb), B (krtke radilne loisko), C (nehmotn prt) a zaaen poda obrzka.
Vypotajte vzbov reakcie. Hmotnos telesa zanedbajte. Uhol je uhol, ktor zviera
nositeka sily F
1
s kolmm priemetom do vodorovnej roviny. Uhol je uhol, ktor zviera
dan priemet s osou x.
Dan: F
1
= 4 kN; F
2
= 3 kN; q
1
= 0,5 kNm
-1
; Q

= 2,5 kN; l
1
= a = 0,4 m; l
2
= 0,6 m;
l
3
= 0,3 m; l
4
= 0,5 m; l
5
= 0,8 m; = 35; = 60

Vsledok:

A
x
= 0,545 kN ()
A
y
= 0,346 kN (^)
A
z
= 2,155 kN ( )
B
y
= 5,22 kN (^)
B
z
= 6,45 kN (7 )
C

= 1,56 kN (+)

Prklad 1.36
Hriade je uloen v radilnom (A) a radilno-
axilnom loisku (B). V rovnovhe je udriavan
silou (N) psobiacou v lane 1, ktor je rovnoben
s osou z. Vypotajte vekos sl v lanch, ak pre
vekosti sl psobiacich v lane 2 plat: T
2
= 0,77T
1
.
Urte taktie vekosti vzbovch reakci.
Dan: G
1
= 1 kN; G
2
= 1,5 kN; = 40;
T
2
= 0,8 kN; l
1
= 0,45 m; l
2
= 0,5 m;
l
3
= 0,4 m; r
1
= 0,4 m; r
2
= 0,6 m
Vsledok:

N

= 0,36 kN; A
y
= 1,02 kN (+); A
z
= 0,057 kN ( );
B
x
= 0 kN; B
y
= 5,17 kN (^); B
z
= 0,931 kN (7 ).


1.5 ROVNOVHA ROVINNCH SSTAV TELIES
Sstava telies je mnoina telies (objektov, lenov), ktor s vzjomne spojen vzbami.
Vzby medzi telesami s tzv. vntorn vzby. Niektor z telies sstavy sa vzbami viau na
zkladov rm (vonkajie vzby). Vzby v rovinnej sstave telies mu odobera 1 a 3
stupne vonosti. Statick uritos rovinnej sstavy telies je uren zo vzahu

=
=
3
1
) 1 ( 3
j
j
jr t n , (1.26)
kde t je poet telies sstavy vrtane zkladovho rmu,
j - poet stupov vonosti, ktor odober prslun vzba,
r
j
- poet vzieb odoberajcich j stupov vonosti pohybu.
Q
Q
F
2
A
B
.


F
1
C
.
l
1
.
a
l
2
l
3
a
l
4
l
5
q
1
x

y

z

A B
x

y
z

l
2
l
3 l
1
r
1
r
2
N

C
T
1
T
2
C

G
1
G
2
44
Pri rieen mu nasta prpady:
n = 0 - loha je staticky a tvarovo urit
Statick uritos - jednoznan matematick rieenie.
Tvarov uritos - sstava telies je nepohybliv.
Nepohybliv mechanick sstavy s oznaovan ako rmov kontrukcie.
n > 0 - loha je staticky preuren a tvarovo neurit
Statick preurenos - znamen, e z rovnc rovnovhy je mon okrem vntornch
a vonkajch vzbovch reakci uri aj niektor prdavn parameter z vonkajieho
zaaenia alebo niektor rozmer danej sstavy.
Tvarov neuritos - sstava telies je pohybliv.
Pohybliv sstavy s oznaovan pojmom mechanizmus. Mechanizmy obsahuj
pohybliv asti a s navrhnut na prenos a transformciu vstupnch sl alebo momentov
na vstupn sily alebo momenty.
n < 0 - loha je staticky neurit a tvarovo preuren
Tak ako pri hmotnom objekte tieto lohy v statike nevieme riei. Vieme zostavi menej
rovnc rovnovhy, ako je neznmych vzbovch reakci.
Pri statickom rieen sstav telies je aplikovan metda uvoovania, t.j. vetky vzby
(vonkajie aj vntorn) nahrdzame vzbovmi reakciami. Pri uvoovan vntornch vzieb
je potrebn dodra princp akcie a reakcie.
Zkladnou lohou statickho rieenia sstav telies je urenie vekosti a orientcie vzbovch
reakci. Na ich vpoet je potrebn zostavi prslun rovnice rovnovhy, priom ich poet
zvis od potu telies tvoriacich dan sstavu telies a potu uzlov (bodov), v ktorch s teles
navzjom spojen. Poet rovnc rovnovhy, ktor je mon pre dan sstavu telies zostavi
m t p 2 3 + = , (1.27)
kde p je poet rovnc rovnovhy ,
t - poet telies bez zkladovho rmu 1 = t t ,
m - poet vntornch vzieb,
3 - poet rovnc rovnovhy pre veobecn rovinn silov sstavu, ktor psob na
teleso v rovine,
2 - poet rovnc rovnovhy pre centrlnu rovinn silov sstavu, ktor psob na uzol
(bod), v ktorom s teles vzjomne spojen.

45
Prklad 1.37
Nosn rmov kontrukcia vytvoren z dvoch telies C,C je pripevnen k zkladovmu
rmu C vo vodorovnch rovinch. Zaaen je spojitm bremenom q psobiacim na
teleso C a momentom M psobiacim na teleso C. Poste statick uritos danej
sstavy. Zostavte rovnice rovnovhy na vpoet vntornch a vonkajch vzbovch
reakci a vypotajte ich.
Dan: q = 240 Nm
-1
; M = 30 Nm; l
1
= 1,25 m; l
2
= 0,85 m; h = 1,0 m


Sstava telies sa sklad z troch telies vrtane rmu. Teleso C sa pripja k zkladovmu rmu
C v bode A (vonkajia vzba) - votknutie - odober v rovine 3 stupne vonosti pohybu.
Teleso C sa pripja k zkladovmu rmu C v bode C (vonkajia vzba) - posuvn rovinn
rotan kb - odober 1 stupe vonosti pohybu. Teles C a C s navzjom spojen v bode B
(vntorn vzba) - rovinn rotan kb - odober 2 stupne vonosti pohybu.
Statick uritos sstavy posdime poda vzahu (1.26)
0 ) 1 . 3 1 . 2 1 . 1 ( ) 1 3 .( 3 ) 1 ( 3
3
1
= + + = =

= j
j
jr t n .
loha je staticky urit, to znamen, e ide o nepohybliv sstavu telies. Metdou uvoovania
s teles C a C uvonen. Pri vonkajch vzbch s orientcie vzbovch reakci v jednotlivch
telesch zvolen ubovone. Pri vntornch vzbch v prvom telese, ktor uvoujeme, volme
orientciu reakci ubovone, pri susednom telese je pri orientcii reakci potrebn dodra princp
akcie a reakcie.
Z matematickho hadiska poda sprvnosti by oznaenie vntornch reakci malo by
odlen, pretoe ide sce o vektory rovnakej vekosti, ale opanho smeru. Oznaenie reakci
je mon odli indexmi. Prv index je index telesa, ktor na uvonen teleso psob. Druh
index je index telesa, ktor je uvoovan. Napr. B
x23
predstavuje psobenie telesa C na
teleso C.
V zvolenom sradnicovom systme poda vzahu (1.27) mono zostavi nasledovn poet
rovnc rovnovhy: 8 1 . 2 2 . 3 2 3 = + = + = m t p .
Rovnovne rovnice pre teleso C

, ) ( :
, :
, :
A
0 2 0
0 0
0 0
32 1 32 2 1
2
1
32
32
= + + =
= + =
= + =

h B l B l l Q M M
Q B A F
B A F
x y A
y y yi
x x xi

(c)
(b)
(a)

kde Q je vekos vslednice spojitho zaaenia psobiaceho na teleso a jej hodnota je
) (
2 1
l l q Q + = .
A
B
C
h

l
2
h

l
1
q

M
C
C
C
C
x
y
+
A
x
B
x
Q
h
l
2
h
l
1
C
A
y
B
y
M
A
(B
x32
)
(B
y32
)
B
y32
B
y23
B
x23
B
x32
B
C
h

l
2
C
B
x
B
y
(B
y23
)
(B
x23
)
M
46
Rovnovne rovnice pre teleso C

. 0 : 0
, 0 : 0
, 0 : 0
2 B
32
23
= + =
= + =
= =

M l C M
C B F
B F
y yi
x xi

(f)
(e)
(d)

Rovnovne rovnice pre bod B

. 0 : 0
, 0 : 0
23 32 23 32
23 32 23 32
y y y y y yi
x x x x x xi
B B B B B F
B B B B B F
= = = =
= = = =


(h)
(g)

V prpade, e vntorn reakcie v bode B nie s odlen prslunmi indexmi, rovnice (g), (h)
nie je potrebn zostavi. Vsledn hodnoty reakci v bodoch A, B s

2 2 2 2
y x y x
B B B A A A + = + = , .
Rovnice (a) a (h) predstavuj sstavu smich linernych algebrickch rovnc s smimi
neznmymi, ktor mono zapsa v maticovom tvare.
.
) (
A

+
=

0
0
0
0
0
0 1 0 1 0 0 0 0
0 0 1 0 1 0 0 0
0 0 0 0 0 0 0
1 1 0 0 0 0 0 0
0 0 1 0 0 0 0 0
0 0 0 2 1 0 0
0 0 0 1 0 0 1 0
0 0 0 0 1 0 0 1
2 1
2
1
23
23
32
32
2
1
M
l l Q
Q
C
B
B
B
B
M
A
A
l
l h
y
x
y
x
y
x
(i)
Rieenm sstavy (podmienka rieenia - determinant sstavy D 0) dostaneme vektor
neznmych a jeho transponovan vektor je:
[ ] Nm) (N; , , , , ,
T
29 35 29 35 0 29 35 0 32 573 29 539 0 = x .
Orientcia reakci so zpornmi znamienkami je v skutonosti opan, ako bolo predpokladan.

Prklad 1.38
Pre kontrukciu zaaen poda obrzka zostavte rovnice rovnovhy na vpoet vntornch
a vonkajch vzbovch reakci a vypotajte ich. Poste statick uritos danej sstavy.
Dan: F
1
= 3 kN; F
2
= 3 kN; M = 0,6 kNm; l
1
= 0,5 m; l
2
= 0,2 m; h = 0,25 m

A
B
C
h

l
2
h

l
1
M
C
C
C
C
D
E
l
2
l
1
F
1
F
2 C
F
1
F
2
E
y
E
x
D
y
D
x
M
C
y
D
x
D
y
C
x
A

C
C
B
x
B
y
C
x
E
y
E
x
C
y
x
y
+
47
Vzby v bodoch B, C, D, E predstavuj spojenie telies rovinnmi rotanmi kbmi - kad
vzba odober 2 stupne vonosti pohybu. Vzba v bode A je tzv. posuvn rovinn rotan
kb, ktor odober 1 stupe vonosti pohybu. Vonkajie vzby sa nachdzaj v bodoch A, B.
Statick uritos sstavy je posden poda vzahu (1.26)
0 ) 0 . 3 4 . 2 1 . 1 ( ) 1 4 .( 3 ) 1 ( 3
3
1
= + + = =

= j
j
jr t n .
loha je staticky aj tvarovo urit. To znamen, e ide o nepohybliv rovinn sstavu telies.
Pri uvoovan pri vntornch reakcich mus by dodran princp akcie a reakcie, priom
tieto reakcie nebud odliovan indexmi - zni sa poet rovnovnych rovnc. Pre kad
uvonen teleso s potom zostaven iba 3 rovnice rovnovhy.
Rovnovne rovnice pre teleso C

. ) ( :
, :
, :
B
0 2 0
0 0
0 0
2 1 1
= + + + =
= + =
= + =

h E l l E l C M
E C B F
E C B F
x y y
y y y yi
x x x xi

(c)
(b)
(a)

Rovnovne rovnice pre teleso C

. :
, :
, :
0 2 0
0 0
0 0
= + =
= =
= + + =

h A M h D M
D C F
C D A F
x C
y y yi
x x xi

(f)
(e)
(d)

Rovnovne rovnice pre teleso C

. 0 2 : 0
, 0 : 0
, 0 : 0
1 2 2 1 2
2 1
= + =
= =
= =

l F l F l D M
F F E D F
E D F
y E
y y yi
x x xi

(i)
(h)
(g)

Rovnice (a) a (i) predstavuj sstavu deviatich linernych algebrickch rovnc s deviatimi
neznmymi, ktor mono zapsa v maticovom tvare Ax = b, t.j. plat
.


2 1 1 2
2 1
2
2 1 1
0
0
0
0
0
0
0 0 2 0 0 0 0 0 0
1 0 1 0 0 0 0 0 0
0 1 0 1 0 0 0 0 0
0 0 0 2 0 0 0 0
0 0 1 0 0 1 0 0 0
0 0 0 1 1 0 1 0 0
2 0 0 0 0 0 0
1 0 0 0 0 1 0 1 0
0 1 0 0 0 0 1 0 1
l F l F
F F
M
E
E
D
D
A
C
C
B
B
l
h h
l l h l
y
x
y
x
y
x
y
x
(k)
Rieenm sstavy rovnc je zskan vektor neznmych
[ ] ) N ( , , , , , , , ,
T
25 8 2 25 25 2 2 25 4 11 25 2 8 13 6 4 11 = x .
Orientcia reakci so zpornmi znamienkami je v skutonosti opan, ako bolo predpokladan.
Vsledn hodnoty reakci v bodoch B, C, D, E s
48

N. , ; N ,
N, , ; N ,
52 26 3 25
98 13 88 12
2 2 2 2
2 2 2 2
= + = = + =
= + = = + =
y x y x
y x y x
E E E D D D
C C C B B B


Prklad 1.39
Rovinn kukov mechanizmus schematicky znzornen na obrzku je zaaen silou F.
Vypotajte, ak moment mus psobi na len C, aby bol mechanizmus v danej polohe
v rovnovhe. Vypotajte aj vekos reakci vo vzbch danej sstavy.
Dan: F = 500 N; = 45; h = 0,5 m; r = 1,4 m; l = 2,5 m; l
1
= 1,0 m



Sstava telies sa sklad zo tyroch telies vrtane rmu. Teleso C sa pripja k zkladovmu
rmu C v bode A (vonkajia vzba - rovinn rotan kb - odober 2 stupne vonosti pohybu).
Teleso C sa pripja k zkladovmu rmu C v bode C (vonkajia vzba - kulisa - odober 2
stupne vonosti pohybu). Teles C a C sa spjaj navzjom v bode B (vntorn vzba -
rovinn rotan kb). Teles C a C sa spjaj navzjom v bode C (vntorn vzba - rovinn
rotan kb). Statick uritos sstavy je posden poda vzahu (1.26)
1 ) 4 . 2 ( ) 1 4 .( 3 ) 1 ( 3
3
1
= = =

= j
j
jr t n .
loha je jedenkrt staticky preuren a tvarovo neurit, ie ide o pohybliv sstavu telies,
tzv. mechanizmus. Z toho dvodu je potrebn uri jeden parameter z vonkajieho zaaenia
(moment M psobiaci na len C), ktor uvdza dan sstavu telies do rovnovhy.
Metdou uvoovania s teles C, C a C uvonen. Orientciu vzbovch reakci volme
ubovone a pre vntorn vzby je potrebn dodra princp akcie a reakcie.
Rovnovne rovnice pre teleso C

. sin cos :
, :
, :
A
0 0
0 0
0 0
= + =
= + =
= + =

r B r B M M
B A F
B A F
x y
y y yi
x x xi

(c)
(b)
(a)

Rovnovne rovnice pre teleso C

. 0 ) sin cos ( cos sin : 0
, 0 : 0
, 0 : 0
1
2
1
B
= + =
= =
= + =

l l F l C l C M
C F B F
C B F
y x
y y yi
x x xi

(f)
(e)
(d)

C
M


r

B
y
B
x
A
x
A
y
C
y
C
F


l
.

l
1
C
x


B
y

B
x
S

C
C
yr

C
y

C
x
M
C
x
y
+
C
C
C
C
C
M
F


r

l
l/2
.
l
1
h
A
B
C

49
Rovnovne rovnice pre teleso C

. 0 : 0
, 0 : 0
, 0 : 0
= =
= =
= =

C
yr y yi
x xi
M M
C C F
C F

(i)
(h)
(g)

Vyuitm substitunej metdy pri rieen rovnc s vypotan hodnoty vzbovch reakci
N 41 , 398 N; 59 , 101 ; 0 = = = = = = =
yr y y y C x x x
C C B A M C B A .
Vetky zloky reakci, ktor maj zporn znamienko, maj v skutonosti opan orientciu.
Na udranie mechanizmu v rovnovnej polohe bola z rovnc rovnovhy vypotan aj
vekos momentu psobiaceho na teleso C, t.j. Nm 57 , 100 = M .

Prklad 1.40
Rovinn mechanizmus zdvhacieho zariadenia, znzornen
na obrzku je zaaen silovou dvojicou M.
Vypotajte vekos sily F psobiacej na len C, aby
bol mechanizmus v danej polohe v rovnovhe. Poste
statick uritos sstavy a vypotajte vekosti reakci vo
vzbch danej sstavy telies.
Dan:
M = 50 Nm; r = 0,3 m; = 30;
l
1
= 1,4 m; l
2
= 2,8 m ; R = 0,5 m;
h
1
= 1,8 m; h
2
= 1,6 m; h
3
= 0,4 m; h
4
= 0,25 m;

Sstava telies sa sklad z piatich telies vrtene rmu. Teleso C sa pripja k zkladovmu
rmu C v bode A (vonkajia vzba - rovinn rotan kb - odober 2 stupne vonosti). Teleso
C sa pripja k zkladovmu rmu C v bode H (vonkajia vzba - rovinn rotan kb -
odober 2 stupne vonosti). Teleso C sa pripja k zkladovmu rmu v bodoch C a D (hladk
dotykov vzby - kad odober 1 stupe vonosti). Teles C a C sa spjaj v bode B (vntorn
vzba - hladk dotykov vzba - odober 1 stupe vonosti). Teles C a C sa spjaj navzjom
v bode E (vntorn vzba - rovinn rotan kb - odober 2 stupne vonosti). Teles C a C sa
spjaj navzjom v bode E (vntorn vzba - kulisa - odober 2 stupne vonosti).
Statick uritos sstavy posdime poda vzahu (1.28):
( ) 1 ) 0 . 3 4 . 2 3 . 1 ( ) 1 5 .( 3 1 3
3
1
= + + = =

= j
j
jr t n .
Teleso C
M


r

R
B
A
y
A
x
T
32
Teleso C
h
2

h
3
D
C

T
23
E
x43
E
y43
F

l
2
E
45
M
E45
TelesoC
H
x
H
y


E
x34
TelesoC
E
y34
E
54
M
E54


C
C
M
F


r

R
l
2
h
2
B
C
C
h
3
A
D
E
h
1
C
C
H
C
l
1
h
4
50
loha je jedenkrt staticky preuren a tvarovo neurit, ie ide o pohybliv sstavu telies -
mechanizmus. Je potrebn uri parameter z vonkajieho zaaenia (sila F psobiaca na len
C), ktor uvdza dan sstavu do rovnovhy. Na zklade princpu akcie a reakcie mono
psa rovnice

,
,
34 43
23 32
x x
E E
T T
=
=

(b)
(a)


.
,
,
45 54
45 54
34 43
M M
E E
E E
y y
=
=
=

(e)
(d)
(c)

Pomocou metdy uvonenia s jednotliv teles uvonen a napsan rovnice rovnovhy.
Rovnovne rovnice pre teleso C

, 0 cos : 0
, 0 : 0
, 0 : 0
32 A
32
= =
= =
= =

r T M M
T A F
A F
y yi
x xi

(h)
(g)
(f)


. N 45 , 192 ) g ( rovnice z
N, 45 , 192
cos
) h ( rovnice z
32
32
= =
=

=
T A
r
M
T
y

Rovnovne rovnice pre teleso C

, 0 ) sin (
) sin ( : 0
, 0 : 0
, 0 : 0
4 1 2
4 3 1 2 E
43 23
43
= + + + +
+ + + + =
= =
= + + =

h h r R h D
h h h r R h C M
E T F
D C E F
y yi
x xi

(k)
(j)
(i)

N 45 , 192 (a) (j), rovnc z
32 23 43
= = = T T E
y
.
Rovnovne rovnice pre teleso C

. 0 : 0
, 0 cos : 0
, 0 sin : 0
54 E E
54 34
54 34
= =
= =
= =

M M
E E F
E E F
y yi
x xi

(n)
(m)
(l)

Uhol , ktor vystupuje v rovniciach, urme z geometrie obrzka
, 82 , 17
sin
arctg
sin
tg
1
1 2
1
1 2 o
=

+ +
=
+ +
=
l
h r R h
l
h r R h


. N 26 , 108 N; 12 , 170 : zskame (b) (k), (i), rovnc rieenm
N, 86 , 61 sin (l) rovnice z
N, 15 , 202
cos
(c) (m), rovnc z
54 34
34
54
= =
= =
=

=
C D
E E
E
E
x
y

51
Rovnovne rovnice pre teleso C

, 0 cos ) ( : 0
, 0 cos : 0
, 0 sin : 0
2
2
1 2
2
1 45 45 E H
45
45
= + + + =
= + =
= + =

Fl h h l E M M
F E H F
E H F
y yi
x xi

(r)
(p)
(o)


. N , cos (p) rovnice z
N, ,
cos
) (
(n) (e), (r), rovnc z
N, , sin (d) (o), rovnc z
2 85
25 107
86 61
45
2
2
1 2
2
1 45
45
= =
=
+
=
= =
F E H
l
h h l E
F
E H
y
x

Rieenm sstavy 17 rovnc o 17 neznmych s vypotan vetky neznme. Na dosiahnutie
rovnovhy danho mechanizmu v je potrebn na len 5 psobi silou danej orientcie
a o vekosti 107,25 N. Tie vzbov reakcie, pre ktor vylo pri vpote zporn znamienko,
maj v skutonosti opan orientciu, ako bolo predpokladan.

Prklad 1.41
Trojkbov kontrukcia poda obrzka je zaaen momentom toiacim v smere hodinovch
ruiiek, ktorho vekos je 150 (N m). Vypotajte zloky reakci vo vzbch D, E ak:
1. moment psob v bode A,
2. moment psob v bode B.
Dan: l = 0,6 m; h = 0,4 m

Vsledok:
1. D
x
= 750 N (), D
y


= 250 N (+),
E
x
= 750 N (), E
y


= 250 N (^).
2. D
x
= 375 N (), D
y


= 250 N (+),
E
x
= 375 N (), E
y


= 250 N (^).

Prklad 1.42
Kontrukcia vytvoren z telies C,C,C je pripevnen vertiklne k zkladovmu rmu C.
Zaaen je silou F a bremenom hmotnosti m, ktor je zavesen na nehmotnom lane
vedenom cez hladk kladku. Poste statick uritos danej sstavy. Zostavte rovnice
rovnovhy na vpoet vntornch a vonkajch vzbovch reakci a vypotajte ich.
Dan: F = 1500 N; m = 200 kg; l
1
= 0,8 m;
l
2
= 0,45 m; l
3
= h = 0,9 m; = 60.

Vsledok:
n = 0; A = 1,24 kN ();
B = 0,32 kN;
B
= 322,7;
C = 2,13 kN;
C
= 215;
D = 3,08 kN;
D
= 27,6;
E = 4 kN;
E
= 133.
Poznmka:

i
( i = B, C, D, E) - smerov uhly nositeliek vslednch
reakci.

A
B
C
h

l
2
h

l
1
C
C
C
C
C
h

E
l
3

Z
m

F
D
A
B
C
h

h

l

C
C
C
C
E
D
l l

52
Prklad 1.43
Vypotajte vekosti vzbovch reakci pri danej polohe kontrukcie. Poste aj statick
uritos danej sstavy.
Dan: F = 5 kN; M = 3 kNm; = 70
a
1
= 0,4 m; a
2
= 0,5 m; a
3
= 0,3 m; a
4
= 0,35 m; a
5
= 0,2 m;
b
1
= 0,5 m; b
2
= 0,25 m.

Vsledok:
n = 0;
A = 7,02 kN;
A
= 48;
M
A
(v smere hod. ruiiek) = 5,70 kNm,
B = 3,34 kN;
B
= 347,4;
C = 4,74 kN;
C
= 287,7;
D = 3,51 kN (^);
E
45
= 1,76 kN;
E
= 35;
M
35
= 0; E
35
= 1,76 kN;

i
( i = A ,B, C, E) s smerov uhly
nositeliek vslednch reakci

Prklad 1.44
Mechanizmus je zaaen silou F. Vypotajte vekos silovej dvojice M potrebnej pre rovnovhu
mechanizmu v danej polohe a reakcie vo vzbch C, D.
Dan: F = 300 N; = 45; h
1
= 0,35 m; h
2
= 0,1 m; h
3
= 0,15 m; l = 0,45 m; r = 0,3 m

Vsledok:
M = 63,64 Nm;
C = 289,6 N ();
D = 289,6 N ().


E
D
C
B A
F
b
2
b
1
a
4
a
3
a
2
a
1

C
C
C
C
C
C
a
5
M
B
A

F
l
h
1

h
3

h
2

r
C
C
C
C M
C
C
C
D
h
2

53
1.6 ROVINN PRTOV SSTAVY
Prtov sstava je sstava telies zloen zo thlych lenov (prtov), ktor s v koncovch
bodoch vzjomne spojen stynkmi (uzlami). Prty prtovej sstavy prenaj sily len
v smere svojich os. Jednotliv prty prtovej sstavy s navzjom spjan len vo svojich
krajnch bodoch, priom vzhadom na charakter zaaovania prtov s ich vzjomn spojenia -
stynky povaovan za kby. Prty prtovej sstavy tvoria tzv. prtov teleso. Zaaenie
prtovej sstavy a jej pripojenie k nepohyblivmu rmu je realizovan len v stynkoch.
Prtov sstava - prtov teleso je teda zloen z s stynkov a p prtov. Stynk reprezentuje
bod v rovine s dvoma stupami vonosti pohybu. Stynky s navzjom spojen prtmi, kde
kad prt odober jeden stupe vonosti pohybu. Poet stupov vonosti prtovho telesa
urme zo vzahu: n
v
= 2s p. Spojenie prtovho telesa a nepohyblivho rmu sa realizuje
prostrednctvom vonkajch vzieb, ktor odoberaj telesu 1 a 3 stupne vonosti pohybu
v rovine. Vzbov zvislos rovinnej prtovej sstavy urme zo vzahu:

=
= =
3
1
p) s 2 (
j
j o v
jr n n n . (1.28)
Pri rieen mu nasta prpady podobn ako pri rovinnej sstave telies. V alch vahch
sa budeme zaobera len lohami staticky a tvarovo uritmi, ie n = 0. lohou rieenia
prtovch sstav je uri vekosti osovch sl v prtoch sstavy, spsob namhania
jednotlivch prtov (ah, tlak) a parametre vonkajch vzbovch reakci. Pri rieen psobia
na kad prt v jeho krajnch uzloch dve sily, ktor maj rovnak vekos, spolon
nositeku, ale opan orientciu. Okrem toho medzi stynkom a prtom psobia sily akcie
a reakcie, ie sily rovnako vek, opane orientovan a majce spolon nositeku.
Zkladn pojmy prtovch sstav s na obr. 1.11.

Obr. 1.11
F
A
B
C
D
C
C
C
C
C
a)
C C prty sstavy
A, B, C, D stynky
n = (2.4-5) (1.1+2.1) = 0
b)
uvonen prtov teleso na vpoet
vonkajch vzbovch reakci
F
A
B
C
D
rovnovha stynka C
F + N
2
+N
5
= 0
F
N
2
N
5
F
N
2
N
5
C
N
2
N
2
C
ah
C D
C
N
5
N
5
tlak
C
B
c)
uvonen stynk C na vpoet osovch sl v prtoch 2, 5
uvonen prty 2, 5 a uren druh namhania v nich
54
1.6.1 Analytick rieenie prtovej sstavy - stynkov metda
Postup analytickho rieenie prtovej sstavy pomocou stynkovej metdy:
Posdenie statickej uritosti prtovej sstavy poda vzahu (1.28).
Uvonenie prtovho telesa a zostavenie rovnc pre vpoet vzbovch reakci.
Orientcia vzbovch reakci je zvolen ubovone. Silov inky psobiace na prtov
teleso (vonkajie sily, vzbov reakcie) tvoria veobecn rovinn silov sstavu, pre
ktor je mon napsa tri rovnice rovnovhy. Rieenm sstavy rovnc s vypotan tri
vzbov reakcie. Orientcia vzbovch reakci, pre ktor je vypotan kladn znamienko,
bola zvolen sprvne. Reakcie so zpornm znamienkom psobia opane.
Postupne s uvoovan jednotliv stynky sstavy, pre ktor s zostaven rovnice
rovnovhy na vpoet osovch sl v prtoch sstavy. inky prtov na stynky s
reprezentovan silami psobiacimi v smere osi prtov. Stynk reprezentuje bod v rovine,
na ktor psob centrlna rovinn silov sstava. Z rovnc rovnovhy stynka s vypotan
dve neznme osov sily v prtoch. Pri postupnej stynkovej metde rieenia je vdy
uvoovan stynk, v ktorom s len dve neznme osov sily.
Na zklade zvolenej orientcie a znamienka osovch sl zskanho vpotom je uren
spsob namhania prta na ah alebo tlak.

Prklad 1.45

Prtov sstava je zaaen silami
F
1
, F
2
, F
3
. Vypotajte vekos
osovch sl vo vetkch prtoch
sstavy a urte druh namhania
jednotlivch prtov.
Dan:
F
1
= 2 kN; F
2
= 3 kN; F
3
= 4
kN; a = 1 m; h = 1,5 m


Prtov teleso je k zkladnmu rmu pripojen vo vzbch v bode A (rovinn rotan kb -
odober 2 stupne vonosti) a v bode C (prt - odober 1 stupe vonosti). Statick uritos
prtovej sstavy poda vzahu (1.28)
0 ) 0 . 3 1 . 2 1 . 1 ( ) 13 8 . 2 ( ) 2 (
3
1
= + + = =

= j
j
jr p s n .
F
1
A B C D
E
C C C
C
C C
h
a

G
H
I
F
2
F
3
a
a a
a
11
10
12 13
K
A
x
A
y
C


F
1
A B C D
E
C C C
C

C C
h
a

G H I
F
2
F
3
a
a a
a
11
10
12 13
K
tg = a/a = 1
= 45
55
loha je staticky a tvarovo urit - prtov sstavu je nepohybliv. Uhol potrebn pre
rozklad sily v bode C
( )
o
87 36
2 2
, arctg tg = = =
a
h
a
h
.
Rovnovne rovnice na vpoet vzbovch reakci

0 2 2 : 0
0 sin : 0
0 cos : 0
3 1 2 C
2 1
3
= =
= =
= + =

aF aF aF aA M
F F C A F
F C A F
y
y yi
x xi

. kN 6
kN, 33 , 18
kN, 66 , 18
=
=
=
y
x
A
C
A

Postupne s uvoovan jednotliv stynky tak, aby boli v uvoovanom stynku maximlne
dve neznme veliiny (dve osov sily v prtoch) a pre kad stynk s zostaven rovnice
rovnovhy. Pri nahrdzan inku prtov osovmi silami je potrebn dodra princp akcie
a reakcie. Uvonen stynky (v porad rieitenosti), rovnice rovnovhy a vypotan osov
sily s uveden v tabuke 1.9.
Tabuka 1.9
Stynk Uvonenie Podmienky rovnovhy Osov sila
A

0 sin : 0
0 cos : 0
10
10 1
= + =
= + + =

N A F
N N A F
y yi
x xi

N
1
= 12,66 kN
N
10
= 8,49 kN
B

0 : 0
0 : 0
9
2 1
= =
= + =

N F
N N F
yi
xi

N
2
= 12,66 kN
N
9
= 0 kN
I

0 sin sin : 0
0 cos cos
: 0
10 8
10 8 11 3
= =
= + + +
=

N N F
N N N F
F
yi
xi

N
8
= 8,49 kN
N
11
= 8 kN
E

0 sin : 0
0 cos : 0
4 2
4 13
= =
= =

N F F
N N F
yi
xi

N
4
= 4,24 kN
N
13
= 3 kN
G

0 : 0
0 : 0
5 1
12 13
= =
= =

N F F
N N F
yi
xi

N
5
= 2 kN
N
12
= 3 kN
H

0 sin : 0
0 cos : 0
6 7
6 12 11
= =
= + + =

N N F
N N N F
yi
xi

N
6
= 7,07 kN
N
7
= 5 kN
C

0 cos cos
: 0
3 8 2
= +
=

C N N N
F
xi

N
3
= 8 kN
D

kontroln stynk
0 0 ; 0 sin sin : 0
0 0 ; 0 cos cos : 0
4 6 5
4 6 3
= = + + + =
= = + =

N N N F
N N N F
yi
xi

x
y
N
2
N
7
N
3
N
8

C


x
y
N
5
N
13
N
12
F
1
x
y
N
5
N
3
N
4
N
6

x
y
N
11
N
7
N
12
N
6

x
y
N
4
N
13

F
2
x
y
F
3
N
10
N
11

N
8

x
y
N
9
N
2
N
1
x
y
A
y
A
x
N
10
N
1
56
Vo vetkch prtoch v uvoovanch stynkoch bolo predpokladan ahov namhanie.
V prpade, e vpotom bolo pre osov silu vypotan kladn znamienko, psob v danom
prte ah. V prpade zpornho znamienka psob v prte tlak. Na zklade vpotov s
ahom namhan prty: 1, 2, 6, 10, 11, 12, 13. Tlakom s namhan prty: 3, 4, 5, 7, 8. Sila
v prte 9 m nulov vekos, t.j. prt 9 neprena iadnu osov silu.

1.6.2 Analytick rieenie prtovej sstavy - priesen metda
Priesen metdu je potrebn poui pre tak prtov sstavu, ktor nie je mon riei
stynkovou metdou (nenachdza sa v nej dvojprtov stynk, t.j. v kadom stynku s tri
a viacej prtov). Tto metda je pouiten aj v prpade, ke je potrebn vypota osov sily
v niektorch prtoch prtovej sstavy. Pri tejto metde rieenia je prtov sstava rozdelen
myslenm rezom cez tri prty, priom musia by splnen nasledujce podmienky:
- pri rozrezan troch prtov je prtov sstava rozdelen na dve asti,
- osi vetkch rozrezanch prtov nesm by sasne rovnoben,
- osi vetkch rozrezanch prtov sa nesm pretna v jednom bode.
Pre ubovone zvolen as rozrezanej prtovej sstavy s zostaven rovnice rovnovhy. inky
rozrezanch prtov s nahraden osovmi silami v smere os rozrezanch prtov. Uvoovan
as rozrezanej prtovej sstavy je zaaovan inkami prslunch vzbovch reakci.

Prklad 1.46

Prtov sstava je zaaen silami F
1
, F
2
.
Vypotajte vekos osovch sl vo vetkch
prtoch sstavy.
Dan:
F
1
= 600 N; F
2
= 500 N; l = 2,1 m.


Obr. A

Obr. B
F
1
A
y
C
D
E
C
C
C


G
H
F
2
l /3
10
l /3
l/3
l/3
l/3
l /3
l/2
B
y

B
x

N
11

N
11
N
4
N
1

N
1

F
1
A
y

C
D
E
C
C
C
C

C
C


G
H
F
2
l /3
11
10
l /3
l/3
l/3
l/3
l /3
l/2
B
y
B
x
F
1
A
B
C
D
E
C
C
C
C

C
C


G
H
F
2
l/3
11
10
l/3
l/3
l/3
l/3
l/3
l/2
57
Prtov sstava je upevnen na zkladn rm vo vzbch v bode A (posuvn rovinn rotan
kb - odober 1 stupe vonosti) a v bode B (rovinn rotan kb - odober 2 stupne vonosti).
Statick uritos danej prtovej sstavy poda vzahu (1.28)
0 ) 1 . 2 1 . 1 ( ) 11 7 . 2 ( ) 2 (
3
1
= + = =

= j
j
jr p s n .
loha je staticky a tvarovo urit, ie prtov sstava je nepohybliv. Vzbov reakcie
vypotame z rovnc rovnovhy uvonenho prtovho telesa (obr. A)

0 : 0
0 : 0
0 : 0
2 B
1
2
= =
= + =
= + =

lF lA M
F B A F
F B F
y
y y yi
x xi

N. 500
N, 1100
N, 500
2
1
2
= =
= =
= =
F A
A F B
F B
y
y y
x

Kad stynk v prtovej sstave m tri prty. Na vpoet osovch sl v prtovej sstave je
potrebn poui priesen metdu. Myslen rez je veden cez prty 1, 4, 11 (obr. B). inky
rozrezanch prtov s nahraden osovmi silami, priom orientcia sl je zvolen ubovone.
Pre ubovone zvolen uvonen as s zostaven rovnice rovnovhy (napr. pre prav as),
z ktorch s vypotan osov sily v rozrezanch prtoch, t.j.

0 sin cos : 0
0 cos : 0
0 sin : 0
4
2
4
3
11 B
4 1
11 4 1
= + + =
= =
= =

N N lN M
N F B F
N N N B F
l l
y yi
x xi

N, 67 , 666
N, 03 , 1118
N, 33 , 833
11
4
1
=
=
=
N
N
N

kde = = =

43 , 63 2 tg
3 2
3
l l
l
.
alej s postupne uvoovan jednotliv stynky tak, aby boli v kadom uzle maximlne dve
neznme osov sily. Pre kad takto stynk s zostaven rovnice rovnovhy. Pri nahrdzan
prtov osovmi silami mus by dodran, e v protiahlch stynkoch toho istho prta plat
princp akcie a reakcie. Rieenie jednotlivch stynkov (v porad rieitenosti), rovnice
rovnovhy a vekos osovch sl sa nachdza v tabuke 1.10.
Tabuka 1.10
Stynk Uvonenie Rovnice rovnovhy Osov sily
B

o
3 , 56
2
3
tg
0 sin : 0
0 cos : 0
3
2
3 7
3 1
= = =
= + + =
= =

l
l
y yi
x xi
N N B F
N N B F

N
3
= 600,93 N
N
7
= 1600 N
C

o
69 , 33
3
2
0 sin : 0
2
3
10 11
= = =
= =

l
l
xi
tg
N N F

N
10
=

1201,85 N

D
o
45 1
0 sin : 0
0 cos : 0
3
3
9 8
9 11 2
= = =
= + =
= + + =

l
l
yi
xi
tg
N N F
N N F F

N
8
= 166,67 N
N
9
= 235,70 N
x
y
B
x
B
y
N
7
N
1

N
3
x
y
N
10
N
11
N
7
F
1

x
y
F
2
N
8
N
11
N
9

58
A

= = =
= + + + =
= + + =

56 , 26 tg
0 sin cos : 0
0 sin cos : 0
2
1
2 6 8
6 2 1
3
2
3
l
l
N N N A F
N N N F
y yi
xi
N
2
= 1414,21 N
N
6
= 372,68 N
E

0 cos cos : 0
5 6 9
= =

N N N F
yi

N
5
= 500 N


Vo vetkch prtoch v uvoovanch stynkoch bolo predpokladan ahov namhanie.
V prpade, e vpotom bolo pre osov silu vypotan kladn znamienko, psob v danom
prte ah. V prpade zpornho znamienka psob v prte tlak. Na zklade vpotov s
ahom namhan prty: 2, 3, 4, 5, 10. Tlakom s namhan prty: 1, 6, 7, 8, 11.

Prklad 1.47
Prtov sstava je zaaen silami F
1
, F
2
, F
3
:
- poste statick uritos danej prtovej sstavy,
- vypotajte vonkajie vzbov reakcie,
- stynkovou metdou vypotajte sily v prtoch 19.
Dan:
a = 2 m; b = 3 m;
F
1
= 3 kN; F
2
= 4 kN; F
3
= 2 kN
Vsledok:
A
x
= 6 kN (); A
y
= 2,58 kN (+); C
y
= 5,58 kN (^);
vekosti osovch sl v prtoch namhanch na ah:
N
1
= 4,06 kN; N
2
= 4,19 kN; N
3
= 3,23 kN; N
4
= 0,10 kN; N
8
= 3,33 kN
vekosti osovch sl v prtoch namhanch na tlak:
N
5
= 0,10 kN; N
6
= 6,98 kN; N
7
= 2,13 kN; N
9
= 3,33 kN

Prklad 1.48
Prtov sstava je zaaen silami F
1
, F
2
, F
3
:
- poste statick uritos prtovej sstavy,
- vypotajte vonkajie vzbov reakcie,
- priesenou metdou vypotajte sily v pr-
toch 2, 3,4.
Dan:
a = 0,3 m; b = 0,7 m; = 30;
F
1
= 3,5 kN; F
2
= 5 kN; F
3
= 5,5 kN
Vsledok:
A
x
= 2,25 kN (); A
y
= 0,20 kN (^); E
y
= 8,07 kN (^);
vekos osovej sily v prte namhanho na ah: N
4
= 0,25 kN;
vekosti osovch sl v prtoch namhanch na tlak: N
2
= 2,25 kN; N
3
= 0,25 kN
b b
a

A
F
2

F
1

F
3
B
D
C
E G
C
C
C
C
C

C

a
b b
a
a
A
F
2

F
1

F
3

B
D
C
E
G
C C
C C C
C


x
y
N
5
N
6
N
9


x
y
N
2
N
8

A
y
N
1
N
6

59
1.7 PASVNE ODPORY
V predchdzajcich kapitolch boli uvaovan idelne vzby, ie vzby s dokonale hladkmi
dotykovmi plochami. V takch prpadoch psobili vzbov reakcie iba v smere normly
k dotykovm plochm.
V skutonosti vzby hmotnch objektov nemaj dokonale hladk povrch a v miestach dotyku
vznikaj loklne deformcie. V mieste dotyku vznikaj trecie sily, ktor spsobuj odklon
nositeky vslednej reakcie od normly k dotykovej ploche vzjomne psobiacich telies.
Trecie sily psobia vdy proti smeru relatvnej rchlosti pohybujcich sa telies. Shrnne s
tieto silov inky nazvan pasvne odpory.
1.7.1 mykov trenie pri posuvnom pohybe
Dotykov vzba odober jeden stupe vonosti aj v relnych vzbch. Nahrdzame ju vak
dvoma vzbovmi reakciami: normlovou (F
n
) a trecou (F
t
). Vekos normlovej reakcie je
vypotan z rovnc rovnovhy uvonenho hmotnho objektu. Vekos trecej sily je uren
na zklade tzv. Coulombovch vzahov:
pri relatvnom pohybe
n t
F f F = , kde f je faktor mykovho trenia,
pri relatvnom pokoji
n T
F f F
0 0
= , kde f
0
je faktor adhzie, F
T0
je adhzna trecia sila.
Pre vekos trecej sily v prpade suchho trenia plat, e trecia sila nezvis od celkovej plochy
spolonho dotyku telies. Plat, e faktor adhzie je v ako faktor mykovho trenia. Pri
vpotoch mono orientane uvaova f
0
= (1,1 1,8) f.

Obr. 1.12
a) pohybujci sa hmotn objekt s uvaovanm trenia v dotykovej ploche
b) uvonen hmotn objekt


Prklad 1.49
Vypotajte vekos sily F potrebnej na posunutie homognnej tye s hmotnosou m, dky l.
Ty je opret o stenu vysok 1,8 m. Koniec tye (bod A) je vo vzdialenosti d od zvislej
steny. Faktor mykovho trenia f medzi tyou, stenou a vodorovnou rovinou je rovnak.
Dan: m = 40 kg; l = 3,6 m; h = 1,8 m; d = 2,4 m; f = 0,6


A
B
h
v
F
F
nA
F
tA
x
y
F
nB
F
tB
G
l/2
0
d

l/2
G
F
n
F
t
F
v
tg = f = F
t
/F
n
b)

v = kont.
f
n
a)
A
B
h
l
F d
60
V bodoch A, B s dotykov vzby odoberajce po jednom stupni vonosti pohybu. Statick
uritos pre teleso v rovine je vypotan pomocou vzahu
1 ) 0 . 3 0 . 2 2 . 1 ( 3 3
3
1
= + + = =

= j
j
jr n .
loha je staticky preuren. Z rovnc rovnovhy je mon okrem vzbovch reakci vypota
aj jeden prdavn parameter - vekos sily F.
Po uvonen tye s vzby odstrnen a nahraden reakciami. Kee s predpokladan relne
vzby, okrem normlovch reakci je potrebn uvaova aj trecie sily psobiace v dotykovej
ploche, priom ich orientcia je proti relatvnemu pohybu.
Z geometrickch parametrov s vypotan doplujce daje potrebn k vpotu:
uhol , ktor zviera ty s vodorovnou rovinou: = = 87 , 36 arctg
d
h
,
vekos dky v opretho seku medzi bodmi A, B: m 3
2 2
= + = d h v .
V zvolenom sradnicovom systme s zostaven rovnice rovnovhy
. cos :
, sin cos :
, cos sin :
0 0
0 0
0 0
2
= =
= + =
= + + + =

v F G M
F F G F F
F F F F F
nB
l
A
tB nB nA yi
tB nB tA xi

(c)
(b)
(a)

Rovnice rovnovhy s doplnen vzahmi pre trecie sily

.
,
nB tB
nA tA
F f F
F f F
=
=

) e (
) d (

Rieime sstavu piatich rovnc o piatich neznmych: F, F
nA
, F
tA
, F
nB
, F
tB
.
Z rovnice (c): F
nB
= 188,35 N , potom z rovnice (e): F
tB
= 113,01 N.
Z rovnice (b): F
nA
= 309,53 N a z rovnice (d) F
tA
= 185,72 N.
Nakoniec z rovnice (a) urme vekos sily potrebnej na posuv tye: F = 389,14 N.
1.7.2 mykov trenie rotujcich telies
Proti zmyslu otania apov uloench v loiskch vznik v dotykovch plochch medzi
apom a loiskom trenie - odpor voi otaniu, tzv. apov trenie. Vekos odporovch sl
mono vyjadri trecm momentom, resp. momentom apovho trenia. Poda smeru zaaenia
rotujceho telesa a jeho uloenia sa apy rozdeuj na axilne (zaaenie psob v smere osi
apu) a radilne (zaaenie psob v smere kolmo na os apu).
Moment apovho trenia pre radilny ap:
Q f r M
t
= , (1.29)
kde r

je polomer apu, f

je faktor apovho trenia (f

= sin ), Q je vekos zaaujcej sily.


Moment apovho trenia pre axilny nezabehan ap s dotykovou plochou v tvare
medzikruia:
61

2
1
2
2
3
1
3
2
2
1
2
2
3
1
3
2
3
2
r r
r r
Q f
r r
r r
Q f M
t

= , (1.30)
kde r
1
, r
2
s polomery medzikruia, f je faktor mykovho trenia , Q je vekos zaaujcej sily a
f f

3
2
= je faktor apovho trenia.
Moment apovho trenia pre axilny zabehan ap s dotykovou plochou v tvare medzikruia:
) ( ) (
2
1
2 1 2 1
r r Q f r r Q f M
t
+ = + = , (1.31)
kde r
1
, r
2
s polomery medzikruia, f je faktor mykovho trenia a pre faktor apovho
trenia plat f f

2
1
= .

Obr. 1.13
Rovinn rotan kb - radilny ap
(odober dva stupne vonosti pohybu pre dokonale hladk aj relne vzby).
a) zobrazenie; b) uvonen hmotn objekt pri uvaovan hladkch dotykovch plch;
c) uvonen hmotn objekt pri uvaovan trenia, M
t
= r

f

Q, kde
2 2
y x
R R R Q + = =

Prklad 1.50
Vypotajte minimlnu vekos sily F potrebnej pre rotan pohyb pky okolo apu A
v smere hodinovch ruiiek s uvaovanm apovho trenia. Pku uvaujte nehmotn.
Dan: G = 10
3
N; a = 1 m; b = 2 m; r

= 10 mm; f

= 0,1





Obr. A

Radilny ap (rovinn rotan kb) odober dva stupne vonosti pohybu. Statick uritos je
vyjadren pomocou vzahu
1 ) 0 . 3 1 . 2 0 . 1 ( 3 3
3
1
= + + = =

= j
j
jr n .
loha je staticky preuren. Z rovnc rovnovhy okrem vzbovch reakci je vypotan aj
vekos sily F, potrebn pre rovnomern pohyb pky. Pri uvaovan apovho trenia v
dotykovej ploche medzi apom a pkou je potrebn uvaova aj trec moment M
t
(obr. A).
Rovnovne rovnice pre uvonen pku
G
F
b
a
R
y
R
x
M
t
R
x
R
y
R

R
x
R
y
R

pohyb

M
t
a)
b)
c)
G
F
a
b
r

A
62

. 0 : 0
, 0 : 0
, 0 : 0
= + =
= =
= =

F b G a M M
F G R F
R F
t A
y yi
x xi

(c)
(b)
(a)

Zaaenie apu je v smere kolmom na jeho os, t.j. je uvaovan radilny ap a pre moment
apovho trenia poda vzahu (1.29) mono psa

y t
R f r M = . (d)
Z uvedench rovnc s vypotan:
R
y
= 1500,75 N; M
t
= 1,435 Nm; F = 500,75 N.

1.7.3 Psov trenie
Vznik pri mkan vlkien (oceovch psov, ln, remeov, reaz) po povrchu telies s
drsnou valcovou plochou. Vlkna s uvaovan ako dokonale ohybn a osov sily, vo
vlknach psobia (pripjaj sa alebo odpjaj) v smere dotynice k danej valcovej ploche.
Zvislos medzi vekosami sl N
1
, N
2
na koncoch vlkna je dan Eulerovm vzahom:

2 1
f
e N N = , (1.32)
kde N
1
je vekos ahovej hnacej sily, N
2
je vekos ahovej hnanej sily, f je faktor
mykovho trenia, je uhol opsania kolesa alebo valca vlknom.

Obr. 1.14
Vlkno odober jeden stupe vonosti pohybu pri hladkch aj relnych vzbch, priom plat

N
1
= N
2
pri uvaovan dokonale hladkch povrchov,

N
1
N
2
pri uvaovan drsnch povrchov (N
1
> N
2
pre naznaen smer pohybu vlkna).


N
2

N
1
63
Prklad 1.51
Lano je ovinut okolo pevnej tye poda obrzka. Na jednom konci lana je zavesen zvaie
neznmej hmotnosti m a na druh koniec lana psob sila F. Na zdvhanie bremena smerom
hore je potrebn psobi na lano minimlnou silou o vekosti F = 3 kN. Pri psoben sily
o vekosti F = 0,48 kN sa bremeno zana spa smerom dole. Vypotajte:
1. hmotnos m zavesenho bremena,
2. faktor mykovho trenia medzi lanom a tyou, ak vekos minimlnej sily pre zdvhanie
bremena je F = 5 mg.


uhol opsania = 5/2
1. Vpoet hmotnosti zavesenho bremena:
Sila v lane v mieste upevnenia bremena sa rovn tiaovej sile: G = mg. Vekos sily F je
rzna poda smeru pohybu lana po tyi.
Pre zdvhanie bremena F > G a plat

=
f
e G F , (a)

=
f
e mg 3000 . (b)
Pre spanie bremena F < G a plat

=
f
e F G , (c)

=
f
e mg 480 . (d)
Eliminciou
f
e v rovniach (b), (d) dostaneme:
kg 3 , 122 480 . 3000
81 , 9
1
480
3000
= = = =

m
mg
mg
e
f
.
2. Vpoet faktora mykovho trenia pre zdvhanie bremena silou F = 5 mg:
Pouitm rovnice (a), dosadenm za F plat
2049 , 0
609 , 1 5 ln
5 ln 5 5
2
5
=

= = = =

f f e e mg mg
f f
.
Faktor mykovho trenia medzi lanom a tyou je 2 0, =& f .
1.7.4 Valiv odpor
Pri valivom pohybe telies po podloke vznik odpor voi valeniu, ktor tie psob proti
pohybu. Odpor valenia vznik v dsledku toho, e teleso a podloka nie s dokonale tuh
a pri vzjomnom dotyku sa deformuj. Vekos valivho odporu zvis od kontaktnch
materilov, kvality povrchu telesa a podloky, polomeru telesa, ktor sa val a od vekosti
zdvhanie bremena
F

pohyb
G
spanie bremena

pohyb
F
G
F
64
zaaujcej sily. Proti valivmu pohybu telesa psob tzv. moment valenia, ktor je mon
vyjadri vzahom
e F M
n v
= , (1.33)
kde e [mm] je rameno valivho odporu.
Valiv vzba odober dva stupne vonosti pohybu (obr.1.15a). V dotykovej ploche medzi
telesom a podlokou psob trecia sila. Pri uvoovan telesa je valiv vzba nahrdzan
dvoma vzbovmi reakciami F
t
a F
n
, priom normlov reakcia F
n
je posunut v smere
pohybu o rameno valivho odporu e (obr. 1.15b) alebo reakciami F
t
a F
n
, priom odpor voi
otaniu mus by zohadnen momentom M
v
(obr. 1.15c). Pre valiv pohyb telesa mus
trecia sila spa podmienku

n t
fF F . (1.34)

Obr. 1.15
Prklad 1.52
Valec, na ktor psob tiaov sila G, sa val po naklonenej rovine vplyvom psobiacej sily
F tak, e stred valca m kontantn rchlos v. Vypotajte vekos sily a skontrolujte, i
nedochdza k premykovaniu valca po naklonenej rovine.
Dan: f = 0,2; e = 4 mm; r
1
= 0,2 m; r
2
= 0,3 m; G = 100 N; =15



Valiv vzba odober v rovine dva stupne vonosti pohybu. Pre urenie statickej uritosti
valca valiaceho sa po naklonenej rovine plat vzah
1 ) 0 . 3 1 . 2 0 . 1 ( 3 3
3
1
= + + = =

= j
j
jr n .
loha je staticky preuren. Z rovnc rovnovhy okrem vzbovch reakci je mon vypota
aj vekos sily F, potrebnej pre rovnomern pohyb valca po naklonenej rovine. Pri uvaovan
odporu voi valeniu je nositeka normlovej reakcie posunut v smere pohybu o rameno
valivho odporu e.
Rovnice rovnovhy pre uvonen valec (x-ov os sradnicovho systmu je zvolen v smere
rovnobenom s naklonenou rovinou a y-ov os m smer kolm na naklonen rovinu).


F
S
G
F
t
F
n
e

x
y


F
v
r
1
r
2
G
pohyb
r
a)
r
c)
F
n
F
t
M
v
r
b)
F
n
F
t
e
65

. :
, cos sin :
, sin cos :
S
0 0
0 0
0 0
1 2
= + + =
= + =
= + =

e F r F r F M
G F F F
G F F F
n t
n yi
t xi

(c)
(b)
(a)

Z rovnc (a), (b) s vyjadren vekosti reakci

. cos sin
, cos sin
G F F
F G F
n
t
+ =
=

a nsledne s dosaden do rovnice (c), odkia je vypotan vekos sily F, ktorou musme
psobi na valec na dosiahnutie jeho rovnomernho pohybu po naklonenej rovine
N ,
) esin cos (
) cos e sin (
3 11
1 2
1
=
+
+
=
r r
r G
F .
Pre valiv pohyb bez premykovania mus by splnen podmienka (1.34), t.j. mus plati

n t
fF F ,
N , , , N , 9 19 52 99 2 0 97 14 = = =
n t
fF F .
Kee uveden nerovnica je splnen, nedochdza k premykovaniu valca po naklonenej
rovine, ale len k valivmu pohybu.

1.7.5 Sstavy telies s uvaovanm pasvnych odporov
Uvaovanie pasvnych odporov v sstavch telies je vzahovan k pohyblivm sstavm
telies. Pre rovnovhu danej sstavy telies je uvaovan rovnomern pohyb a je potrebn
vyjadri vekos niektorho prdavnho parametra na dosiahnutie rovnovnej polohy.
Metodika rieenia statickej rovnovhy sstav telies s uvaovanm pasvnych odporov je
zhodn s rieenm sstav telies s idelnymi vzbami. Rozdiel v rieen je v tom, e je potrebn
uvaova relne vzby, ktor s nahraden prslunmi reakciami. K rovniciam rovnovhy,
ktor s zostaven pre uvonen teles, je potrebn zostavi alie doplnkov rovnice, ktor
zohaduj prslun druh pasvneho odporu vyskytujceho sa v sstave.
Vzah na posdenie statickej uritosti je zhodn so vzahom pre sstavy telies s idelnymi
vzbami (1.26).

66
Prklad 1.53
Pkov brzda sli na rovnomern spanie bremena hmotnosti m. Vypotajte akou silou
je potrebn psobi na jednoramenn pku na dosiahnutie rovnomernho pohybu bremena.
apov trenie neuvaujte.
Dan:
f = 0,4; m = 40,77 kg; a

= 0,22 m; b

= 0,06 m;
l

= 0,68 m; r
1
= 0,2 m; r
2
= 0,12 m


Sstava telies sa sklad z pky C, brzdovho kota C a zkladnho rmu C. Bremeno s
hmotnosou m predstavuje zaaujcu silu psobiacu na obvode kota C.
Vonkajie vzby v bodoch A, C s rovinn rotan kby odoberajce po dva stupne vonosti
pohybu. Vntorn vzba medzi telesami v bode B je dotykov vzba, ktor odober 1 stupe
vonosti pohybu, aj ke tu uvaujeme mykov trenie.
Posdenie statickej uritosti rieenej sstavy telies
1 ) 2 2 1 1 ( ) 1 3 ( 3 ) 1 ( 3
3
1
= + = =

= j
j
r j t n .
loha je jedenkrt staticky preuren. To znamen, e pre rovnovhu je potrebn z rovnc
rovnovhy vypota jeden prdavn parameter z vonkajieho zaaenia. V tomto prpade ide
o vekos sily F, ktor je potrebn uri na dosiahnutie poadovanho stavu.
Jednotliv teles s uvonen a pre kad z nich s zostaven prslun rovnice rovnovhy.
Rovnice rovnovhy pre teleso C:

. 0 : 0
, 0 : 0
, 0 : 0
32 32 A
32
32
= + =
= =
= + =

b F a F Fl M
F F A F
F A F
t n
n y yi
t x xi
(a)
Rovnice rovnovhy pre teleso C:

. 0 : 0
, 0 : 0
, 0 : 0
2 23 1 C
23
23
= + =
= + =
= =

r F Gr M
F G C F
F C F
t
n y yi
t x xi
(b)
A
x
F
b

C
A
y
F
t32
F
n32
G = mg

C
F
n23
F
t23
C
x
C
y
G
C
A
F
a

l

b

r
1
m
C
C
pohyb

C
r
2
B
67
Pre princp akcie a reakcie vo vntornej vzbe plat
. ,
32 23 32 23 n n t t
F F F F = = ) c (
Pre prslun pasvny odpor - mykov trenie je pouit vzah

23 23 n t
fF F = . (d)
Rieenm sstavy 9-tich rovnc (a)-(e) s vypotan hodnoty 9-tich neznmych.

[ ]
[ ] (N) 5 , 1266 6 , 666 6 , 666 5 , 1666 6 , 666 5 , 1666 3 , 480 1 , 1186 6 , 666
(N)
T
23 23 32 32
T
=
=
x
x
y x t n t n y x
C C F F F F F A A

Sila F vzhadom na zporn znamienko zskan vpotom mus psobi opane.


Prklad 1.54

Na naklonenej rovine je poloen blok s hmotnosou m,
ktor je ahan lanom prevesenm cez nehmotn hladk
kladku, na ktorom vis bremeno s hmotnosou m
0
. Faktor
mykovho trenia medzi blokom a naklonenou rovinou je f.
Pasvne odpory v uloen kladky neuvaujte. Vypotajte
vekos hmotnosti m
0
tak, aby bol blok v rovnovhe na
naklonenej rovine.
Dan: m = 100 kg; f = 0,3; = 20
Vsledok: m
0
= 62,39 kg



Prklad 1.55

Pri rovnomernom zdvhan bremena hmotnosti m
1
je potrebn
psobi na ap bubna momentom M. Vypotajte faktor apovho
trenia v klznom loisku - radilny ap. Hmotnos bubna je m.
Dan:
m
1
= 500 kg; m = 100 kg;
M = 1510 Nm; R = 300 mm; r

= 25 mm
Vsledok:
f

= 0,262














m
1
M

R

r

m


f
m
m
0
68
Prklad 1.56
Vypotajte vekos sily F, psobiacej na
teleso C sstavy telies tak, aby sa pohybovala
rovnomerne smerom doprava. Tiaov sila
psobiaca na valec je G
2
, tiaov sila
psobiaca na nosnk je G
3
. Pri vpote
uvaujte faktor apovho trenia f

, rameno
valivho odporu je e, faktor mykovho trenia
medzi nosnkom a podlokou f. Poloha
sstavy je dan vzdialenosou a.
Dan:
G
2
= 200 N; G
3
= 300 N; f

= 0,2; f = 0,3;
e = 4 mm; a = 300 mm; r = 200 mm;
r

= 20 mm; l = 500 mm; h = 50 mm


Vsledok: F = 67,97 N































T
G
3
G
2
C
C
a
l
l
r
A
B
F
pohyb
r

C
h
69
2 KINEMATIKA


2.1 KINEMATIKA BODU
Spojit sled polh, ktor tvor bod pohybujci sa v priestore, sa nazva trajektria. Drha
bodu je vekos seku trajektrie, ktor bod preiel za urit asov interval.
Pohyb bodu je definovan funkciou argumentu asu, tzv. polohovm vektorom r
) (t r r = . (2.1)
Rchlos bodu v danom asovom okamihu v je dan vzahom
r
r
v
&
= =
dt
d
. (2.2)
Rchlos charakterizuje asov zmenu polohovho vektora. Je to vektorov veliina, ktor
le na dotynici k trajektrii v danom asovom okamihu a je orientovan v smere pohybu.
Jednotkou rchlosti je ms
-1
. Rchlos bodu sa pri jeho pohybe men o do vekosti aj smeru.
Zmena rchlosti je definovan zrchlenm.
Zrchlenie bodu v danom asovom okamihu a je dan vzahom
r v
v
a
& & &
= = =
dt
d
. (2.3)
Jednotkou zrchlenia je ms
-2
.


2.1.1 Pohyb bodu v kartezinskej sradnicovej sstave
Poloha bodu v kartezinskej sradnicovej sstave je uren polohovm vektorom
k j i k j i r ) ( ) ( ) ( ) ( t z t y t x z y x t + + = + + = , (2.4)
kde x = x(t), y = y(t), z = z(t) s parametrick rovnice definujce polohu bodu a i, j,
k s jednotkov vektory.
Vektor rchlosti pohybu bodu
k j i k j i k j i
r
v
z y x
v v v z y x
dt
dz
dt
dy
dt
dx
dt
d
+ + = + + = + + = = & & & , (2.5)
kde v
x
,v
y
, v
z
s zloky rchlosti v smere osi x, y, z.
Vekos rchlosti

2 2 2
z y x
v v v v + + = = v . (2.6)
Vektor zrchlenia bodu
k j i k j i k j i
v
a
z y x z y x
a a a z y x v v v
dt
d
+ + = + + = + + = = & & & & & & & & & , (2.7)
kde a
x
,a
y
, a
z
s zloky zrchlenia v smere osi x, y, z.
Vekos zrchlenia

2 2 2
z y x
a a a a + + = = a . (2.8)
70
Pohyb bodu v rovine
Ak sa bod pohybuje iba v rovine x, y, z toho vyplva, e vetky z-tov zloky kinematickch
velin v predchdzajcich vzahoch s nulov, t.j. plat
0 , 0 , 0 = = =
z z
a v z .
2.1.2 Pohyb bodu vo valcovej sradnicovej sstave
Vo valcovch sradniciach je pohyb bodu dan parametrickmi sradnicami
). ( ), ( ), ( t z z t t = = = (2.9)
Transforman vzahy medzi valcovmi a kartezinskymi sradnicami
. , sin , cos z z y x = = = (2.10)


Obr. 2.1 Obr. 2.2
Polohov vektor bodu vo valcovom sradnicovom systme je dan

z
zk i r + =

. (2.11)
Vektor rchlosti bodu

z z z z
v v v z
dt
dz
dt
d
dt
d
k j i k j i k
i
i v + + = + + = + +

& & & , (2.12)


kde plat

= + = j j i
i
& & & ) (cos ) sin (
dt
d
.
Pre vekos rchlosti plat

2 2 2 2 2 2
) ( ) ( ) ( z v v v v
z
& & & + + = + + = =

v (2.13)
Vektor zrchlenia bodu

, ) 2 ( ) (
2
z z z
z
a a a z
dt
z d
dt
d
dt
d
dt
d
dt
d
k j i k j i
k
j
j j
i
i a
+ + = + + + =
+ +

+ + +

& & & & & & & &


&
&
&
& & &
&
(2.14)
kde

= + = i j i
j
& & & ) sin ( ) cos (
dt
d
,
- radilna zloka zrchlenia

= i a ) (
2
& & & , (2.15)
- transverzlna zloka zrchlenia

+ = j a ) ( & & & & 2 . (2.16)
x
y
z
z(t)
(t)
(t)
r
L
i


k
z
= k
i
j
k k
j i j
j i i
=
+ =
+ =

z
cos sin
sin cos

i
j
i

j



71
Pre vekos zrchlenia plat

2 2 2
z
a a a a + + = =

a . (2.17)
Ak sa 0 = z , 0 =
z
v , 0 =
z
a hovorme o polrnom sradnicovom systme s jednotkovmi
vektormi i

, j

(obr. 2.2). Polrne sradnice s pecilnym prpadom valcovch sradnc.



2.1.3 Pohyb bodu v prirodzenej sradnicovej sstave
Zaiatok tejto sradnicovej sstavy je pevne spojen s bodom pohybujcim sa po krivke,
jeho poloha sa s asom men. V kadej polohe bodu je mon zostroji dotynicu (t), normlu
(n) a binormlu (b). Tieto priamky tvoria osi prirodzenej sradnicovej sstavy a zrove
tvoria tzv sprievodn trojhran priestorovej krivky s jednotkovmi vektormi
b n t
k j i , , (obr. 2.3).

Obr. 2.3
Pohyb bodu je uren rovnicou
), ( resp. ) ( s t r r r r = = ) ( kde t s s = .
Rchlos bodu

t t
s v i i v & = = , (2.21)
kde
ds
d
t
r
i = je jednotkov tangencilny vektor. (2.22)
Zrchlenie bodu

n n t t n t
t
t
a a
R
v
v
dt
d
v
dt
dv
dt
d
j i j i
i
i
v
a + = + = + = =
0
2
& , (2.23)
kde R
0
je polomer krivosti drhy bodu,

K
n
K
j = - jednotkov normlov vektor, (2.24)

ds
d
t
i
K = - vektor flexnej krivosti,

2
3
2
3
] ) ( 1 [ ) (
1
2 2 2 0
y
y
y x
x y y x
R
K
+

=
+

= =
& &
& & & & & &
- flexn krivos rovinnej krivky, (2.25)

dx
dy
y = ,
2
2
dx
y d
y = - prv a druh derivcia poda sradnice x,
i
t
j
n
i

j

k

x

y

z

k
b
t

n

b

s

r

v

a
t
a
n
a

72
v a
t
& = - tangencilna zloka zrchlenia,

0
2
R
v
a
n
= - normlov zloka zrchlenia.
Vekos vslednho zrchlenia

2 2
n t
a a a + = = a . (2.26)
Tangencilne zrchlenie charakterizuje zmenu vekosti rchlosti a normlov zrchlenie
charakterizuje zmenu smeru rchlosti. Pohyb bodu sa odohrva v rovine tvorenej dotynicou
a normlou, v tzv. oskulanej rovine. Zloka zrchlenia v smere binormly je vdy nulov a
b
= 0.
Jednotkov vektory
b n t
k j i , , (vektory sprievodnho trojhranu priestorovej krivky) tvoria pravouhl
pravotoiv systm - sradnicov systm, preto plat

n t b
j i k = ,
b n t
k j i = ,
t b n
i k j = . (2.27)


Pohyb bodu po priamke
Ak je traktriou bodu priamka, hovorme o priamoiarom pohybe.
Pohyb bodu je uren rovnicou ) (t s = (obr. 2.4). Pre pohyb bodu po
priamke mus plati: . kont =
t
i Bod m len zrchlenie tangencilne,
normlov zrchlenie je vdy nulov, smer rchlosti sa nemen
Obr. 2.4
a) Pri rovnomernom pohybe kont. = v a 0 = =
dt
dv
a .
Vekos drhy prejden bodom po priamke urme integrciou rchlosti v (pri uvaovan
zaiatonch podmienok: t = 0, s = s
0
:
0
s vt s + = .
b) Pri rovnomerne zrchlenom (spomalenom) pohybe kont. = =
t
a a :
- rovnomerne zrchlen pohyb: sgn v = sgn a.
- rovnomerne spomalen pohyb: sgn v sgn a.
Vekos rchlosti bodu urme integrciou tangencilneho zrchlenia a
t
) ( v a
t
& = , a
n
= 0
(pri uvaovan zaiatonch podmienok: t = 0, a
t
= a = kont., v = v
0
, s = s
0
:
0
v at v + = .
Vekos drhy prejden bodom urme integrciou rchlosti v ) ( s v & = :
0 0
2
2
1
s t v at s + + = .
c) Veobecn pohyb bodu po priamke nastva, ak: ) (t
t
a a =
- nerovnomerne zrchlen pohyb: sgn v = sgn a.
- nerovnomerne spomalen pohyb: sgn v sgn a.

Pohyb bodu po krivke
Ak je traktriou bodu krivka, hovorme o krivoiarom pohybe. Pohyb bodu je uren
rovnicou ) (t s s = . Pre pohyb bodu po krivke mus plati: . kont
t
i
a) Pri rovnomernom pohybe kont. v = a 0 , 0
0
2
= = =
R
v
a v a
n t
&
& .
b) Pri rovnomerne zrchlenom (spomalenom) pohybe 0 kont., =
n t
a a :
- rovnomerne zrchlen pohyb: sgn v = sgn a.
- rovnomerne spomalen pohyb: sgn v sgn a.
0
L
s
i
t
73
c) Veobecn pohyb bodu po krivke nastva, ak: 0 ), ( =
n t t
a t a a :
- nerovnomerne zrchlen pohyb: sgn v = sgn a.
- nerovnomerne spomalen pohyb: sgn v sgn a.

Pohyb bodu po krunici
Pohyb bodu po krunici (obr. 2.5a) je uren pohybom jeho sprievodia ) (t = a mono ho
skma rznymi spsobmi.
a) V kartezinskych sradniciach je opsan rovnicou
j i r y x + = , (2.28)
kde = = sin , cos r y r x . (2.29)
a)

b)

Obr. 2.5

Vektor rchlosti bodu
j i r v cos sin & & &
r r + = = . (2.30)
Vekos rchlosti
= + = & & & r r r v
2 2
) cos ( ) sin ( . (2.31)
Vektor zrchlenia bodu
j i j i a ) sin cos ( ) cos sin (
2 2
+ + = + = & & & & & & & & & & r r y x . (2.32)
Vekos zrchlenia

4 2 2 2 2 2
) sin cos ( ) cos sin ( + = + = & & & & & & & & & r r r r r a . (2.33)
Trajektria bodu je uren z rovnc pohybu x, y (2.29)

2 2 2
2
2
2
2
2 2
1
sin
cos
1 ) (sin ) (cos
r y x
r
y
r
x
r
y
r
x
= + = +

=
=
= +
, (2.34)
t.j. rovnica krunice s polomerom r a stredom v zaiatku sradnicovho systmu.
b) V prirodzench sradniciach je pohyb bodu dan drhou prejdenou po krunici s
x
t
r
L
v
s
n
a
t
a
n
a

0
x
y
r

L(x, y)
0
74
r s = . (2.35)
Rchlos bodu

t
vi v = a pre vekos rchlosti & & r s v = = . (2.36)
Zrchlenie bodu

j i a
n t t
a a + = ,
4 2
+ = & & & r a (2.37)
kde & & & r v a
t
= = je tangencilna zloka zrchlenia,
2
2
0
2
) (
=

= = &
&
r
r
r
R
v
a
n
je normlov
zloka zrchlenia, r R =
0
je polomer krivosti drhy, ktor sa rovn polomeru krunice.

Prklad 2.1
Os pohybujcej sa tye prechdza stle
bodom O a bod B leiaci na tyi sa
pohybuje kontantnou rchlosou c po
priamke p. Urte rovnice pohybu
a vyjadrite vektor rchlosti bodu M .
Dan: c = kont; l, l
0
v ase t = 0 s:
B B
0
, os tye x, sstava je v kude

Bod M kon krivoiary pohyb v rovine. Njs rovnice pohybu znamen uri v kartezinskom
sradnicovom systme jeho sradnice x
M
,

y
M
ako funkcie asu.
Bod B sa pohybuje po priamke p kontantnou rchlosou c, priom pre drhu prejden bodom
B mono psa
B
0
B = s = ct.
V ubovonom ase je z geometrie trojuholnka OB
0
B vyjadren uhol

l
ct
l
ct
l
arctg
B B
tg
0
= = = . (a)
Pre sradnice bodu M v kartezinskej sradnicovej sstave mono psa

, arctg sin sin
, arctg cos cos
0 0 M
0 0 M

+ = + =

+ = + =
l
ct
l ct l ct y
l
ct
l l l l x
(b)
kde uhol je funkciou asu (a) a potom polohov vektor bodu M
. arctg sin arctg cos
0 0 M M M
j i j i r

+ +

+ = + =
l
ct
l ct
l
ct
l l y x (c)
Vektor rchlosti bodu M dostaneme derivciou polohovho vektora (c) poda asu
O
B
B
0
M
M
0
x
y c
l
l
0
p
r
M
y
M

x
M

75

. arctg cos
) (
arctg sin
) (

2 2
0
2 2
0
M M M
j i
j i v
4 4 4 4 4 3 4 4 4 4 4 2 1 4 4 4 4 4 3 4 4 4 4 4 2 1
& &
y x
v v
l
ct
ct l
cl
l c
l
ct
ct l
cl
l
y x

+
+ +

+
=
+ =

Vekos rchlosti bodu M je mon vyjadri pomocou vzahu

2 2
M y x
v v v + = .
Goniometrick funkcie vo vzahu (b) mono vyjadri aj pomocou prepony pravouhlho
trojuholnka OB
0
B

2 2
) ( OB ct l + = , (d)

2 2 2 2
) (
cos ;
) (
sin
ct l
l
ct l
ct
+
=
+
= .
Dosadenm do vzahov (b) polohov vektor mono zapsa potom aj v tvare
j i r

+
+ +

+
+ =
2 2
0
2 2
0 M
) ( ) ( ct l
ct
l ct
ct l
l
l l .
Vektor rchlosti m potom tvar
j i v

+
+ +
+
=
3
2
3
2
] ) ( [ ] ) ( [
2 2
2
0
2 2
2
0
M
ct l
cl l
c
ct l
t lc l
.
Polohu bodu M a jeho pohyb je mon uri aj v polrnom sradnicovom systme. Sradnice
bodu M v polrnom sradnicovom systme s

2 2
0
) ( ) ( ct l l t + + = = ,

l
ct
t arctg ) ( = = .
Polohov vektor bodu M v polrnom sradnicovom systme


+ + = = i i r
2 2
0 M
) (ct l l .
Vektor rchlosti mono vyjadri

+ +
+
+
= + = + = j i j i j i v
4 4 4 3 4 4 4 2 1
43 42 1
& &
v
v
ct l
ct l l lc
ct l
t c
v v
2 2
2 2
0
2 2
2
) (
) (
) (

a jej vekos

2 2 2 2
) ( ) ( + = + = =

& & v v v v .
76
Prklad 2.2

Po ramene o otajcom sa poda vzahu
t = , kde = kont., sa pohybuje bod M
kontantnou rchlosou vekosti c.
Vypotajte vekos rchlosti a zrchlenia
bodu M. Na zaiatku pohybu bolo rameno vo
vodorovnej polohe a bod M sa nachdzal vo
vzdialenosti l
0
od bodu 0 (M
0
).
Dan: c = kont; = kont., l
0


1. spsob rieenia - v kartezinskom sradnicovom systme (0, x ,y)
Drha, ktor bod M prejde kontantnou rchlosou je vyjadren vzahom
ct s = = M M
0
.
Parametrick rovnice pohybu bodu M maj tvar

. sin ) ( sin ) (
, cos ) ( cos ) (
0 0 M
0 0 M
t ct l ct l y
t ct l ct l x
+ = + =
+ = + =

Vekosti zloiek vektora rchlosti bodu M

[ ]
[ ]
. cos ) ( sin
sin ) (
, sin ) ( cos
cos ) (
0
0
0
0
t ct l t c
dt
ct l d
y v
t ct l t c
dt
t ct l d
x v
y
x
+ + =
+
= =
+ =
+
= =
&
&

Pre celkov vekos rchlosti bodu M plat

2
0
2 2 2 2
) ( ct l c v v v
y x
+ + = + = .
Vekosti zloiek vektora zrchlenia bodu M

, ] sin ) ( cos 2 [
] cos ) ( sin [
, ] cos ) ( sin 2 [
] sin ) ( cos [
2
0
0
2
0
0
t ct l t c
dt
t ct l t c d
v a
t ct l t c
dt
t ct l t c d
v a
y y
x x
+ =
+ +
= =
+ + =
+
= =
&
&

a pre celkov vekos zrchlenia bodu M dostaneme

2
0
2 2 2 2
) ( 4 ct l c a a a
y x
+ + = + = .
2. spsob rieenia - v polrnej sradnicovej sstave
Parametrick rovnice pohybu bodu M maj tvar:

.
,
0
t
ct l
=
+ =

0
M
M
0

x
y
c
l
0
y
M

x
M

o
77
Vekosti zloiek vektora rchlosti bodu M

. ) (
,
0
+ = =
= =

ct l v
c v
&
&

Vekos vektora rchlosti bodu M je potom vyjadren v tvare

2
0
2 2 2 2 2 2
) ( ) ( ct l c v v v + + = + = + =

& & .
Vekosti zloiek vektora zrchlenia bodu M

. 2 2
, ) (
2
0
2
= + =
+ = =

c a
ct l a
& & & &
& & &

Vekos vektora zrchlenia bodu M

2 2
0
2 2 2 2 2 2
4 ) ( ) 2 ( ) ( c ct l a a a + + = + + = + =

& & & & & & & .

Prklad 2.3

Bod kon zrchlen pohyb po krivke tak, e jeho
oblkov sradnica vzhadom na zaiaton polohu
definovan bodom 0 je s = be
kt
[m], a to tak, e vektor
vslednho zrchlenia pohybu bodu zviera s vektorom
rchlosti stle kontantn uhol .
Urte vekosti nasledujcich kinematickch velin:
v(t), v(s), a
t
(t), a
n
(t), a(t), R
0

Dan: kontanty b [m] > 0, k [1/s] > 0

Vzhadom na to, e je definovan oblkov sradnica polohy bodu, bude teda pohyb bodu
skman v prirodzenej sradnicovej sstave.
Vekos rchlosti je vyjadren v tvare

kt
e bk s t v = = & ) ( ,
resp. ako funkcia sradnice s
s k s v = ) ( .
Vekos tangencilnej zloky zrchlenia je

kt
t
e bk v t a
2
) ( = = & .
Vektor vslednho zrchlenia a je mon rozloi pomocou kolmch priemetov do smeru
dotynice a normly drhy bodu v prslunom asovom okamihu.
Vzhadom na to, e nositeka rchlosti a tangencilneho zrchlenia je toton priamka, pre
vekos tangencilnej zloky zrchlenia potom plat

= =
cos cos
cos
2 kt
t
t
e bk a
a a a .
Vekos normlovho zrchlenia je vyjadren podobnm spsobom v tvare
=

= = tg e bk
e bk
a a
kt
kt
n
2
2
sin
cos
sin .
0
s
v
t

n


a
a
t
a
n
78
Polomer krivosti drhy je uren v tvare

= =
tg
tg
) (
2
2 2
0
kt
kt
kt
n
be
e bk
e bk
a
v
R .

Prklad 2.4
Pre zadan parametrick rovnice pohybu bodu M v kartezinskej sradnicovej sstave je
potrebn uri:
1. kinematick veliiny ako funkcie asu: v, v, a, a, a
t
, a
n
,
2. polomer krivosti trajektrie R
0
.
Dan: x =ktsin(bt) [m], y = ktcos(bt) [m], k = 1,0 ms
1
, b = 1,0 s
1


Polohov vektor bodu M vzhadom na zadan parametrick rovnice m tvar
j i r t t t t cos sin + = .
1. Vpoet poadovanch kinematickch velin:
Polohov vektor
j i r ) cos ( ) sin ( t t t t + = .
Vektor rchlosti
j i
r
v ) sin cos ( ) cos sin ( t t t t t t
dt
d
+ + = = .
Vektor zrchlenia
j i
v
a ) cos 2sin ( ) sin 2cos ( t t t t t t
dt
d
+ = = .
Vekosti vektorov rchlosti a zrchlenia s zskan pomocou vzahov

. 4 ) cos 2sin ( ) sin 2cos (
, 1 ) sin cos ( ) cos sin (
2 2 2 2 2
2 2 2 2 2
t t t t t t t a a a
t t t t t t t v v v
y x
y x
+ = + = + =
+ = + + = + =

Pre vekos tangencilnej zloky zrchlenia

2
2
1
1
t
t
t
dt
d
dt
dv
v a
t
+
=

+ = = = & .
Na zklade vekosti vslednho zrchlenia a a jeho tangencilnej zloky zrchlenia a
t
je
mon vyjadri vekos normlovej zloky zrchlenia a
n
, t.j. plat

2
2
2 2 2 2
1
2
t
t
a a a a a a
t n n t
+
+
= = + = .
2. Vpoet polomeru krivosti drhy R
0
je uren zo vzahu pre vpoet normlovej zloky
zrchlenia, t.j. plat

2
2
2
2
2 2
0
0
2
2
) 1 (
1
2
1
2
3
t
t
t
t
t
a
v
R
R
v
a
n
n
+
+
=
+
+
+
= = = .

79
Prklad 2.5
Pre zadan parametrick rovnice pohybu bodu M v kartezinskej sradnicovej sstave je
potrebn uri:
1. jednotkov vektory sprievodnho trojhranu priestorovej krivky ako funkcie asu i
t
, j
n
, k
b
,
2. polomer krivosti trajektrie R
0
.
Dan: ) ( sin t b k x
1
2
1
= [m], ) sin( t b k y
2 2
= [m],
1
k = 1,0 m,
2
1
2
= k m,
1
b = 1,0 s
1
,
2
b = 2,0 s
1


1. Vpoet jednotkovch vektorov sprievodnho trojhranu priestorovej krivky:
Polohov vektor bodu M m tvar
j i j i r ) 2 sin ( ) (sin
2
1
2
M M M
t t y x + = + = .
Totlny diferencil polohovho vektora je vyjadren vzahom
dt t t dt t t t dt
dt
d
d ] ) 2 (cos ) 2 [(sin ] ) 2 (cos ) cos sin 2 [(
M
M
j i j i
r
r + = + = = ,
priom pre jeho vekos plat
dt dt t t ds = + =
2 2
M
) 2 (cos ) 2 (sin .
Pre jednotkov tangencilny vektor i
t
plat
j i
j i
v
r
r
i t t
t t t
v
dt
ds
dt
d
ds
d
t
2 cos 2 sin
1
) 2 cos 2 ( ) cos sin 2 (
2
1
M
M
M
M
M
M
+ =
+
= = = = .
Vektor flexnej krivosti je vyjadren v tvare
j i
i i
i
K t t
v
dt
d
dt
ds
dt
d
ds
d
t t
t
2 sin 2 2 cos 2
M
M
M
= = = =
a jeho vekos
2 ) 2 sin 2 ( ) 2 cos 2 (
2 2 2 2
= + = + = t t K K K
y x
m
1
.
Pre jednotkov normlov vektor potom plat
j i
K
j t t
K
n
2 sin 2 cos = = .
Jednotkov binormlov vektor je vyjadren z vektorovho sinu
k
k j i
j i k ) 2 cos 2 sin (
0 2 sin 2 cos
0 2 cos 2 sin
2 2
t t
t t
t t
n t b
=

= = .
2. Polomer krivosti drhy R
0
je mon vypota aj z vekosti vektora flexnej krivosti, t.j.

2
1 1
0
= =
K
R m.

80
Prklad 2.6
Bod sa pohybuje po priamke tak, e jeho poloha v ase t je dan vzahom: t x x = sin
0
[m],
kde x
0
[m] a [rad/s]s kontanty. Vyjadrite:
1. vekos rchlosti bodu ako funkciu drhy v(x),
2. vekos zrchlenia bodu ako funkciu drhy a(x),
3. vekos zrchlenia bodu ako funkciu vekosti rchlosti a(v).

1. Vpoet vekosti rchlosti ako funkcie drhy.

0
0
sin sin
x
x
t t x x = = ,

0
0
cos cos
x
v
t t x
dt
dx
v

= = = .
Pouitm vzahu 1
2 2
= + sin cos je zskan rovnica

2 2
0
2
0
2
0
) ( 1 x x x v v
x
x
x
v
= = =

,
o je rovnica elipsy znzornen na obr. A.
2. Vpoet vekosti zrchlenia ako funkcie drhy.
) ( sin
2
0
2
x a x t x
dt
dv
a = = = = .
Uveden rovnica pre a = a(x) predstavuje rovnicu priamky (obr.B).
3. Vpoet vekosti zrchlenia ako funkcie rchlosti.

. cos cos
, sin sin
0
0
0
2
0
2
x
v
t t x v
x
a
t t x a

= =

= =

Po dosaden do vzahu 1 sin cos
2 2
= + je zskan rovnica

2 2
0
2
2
0
2
0
2
1 v x a
x
v
x
a
= =

,
o predstavuje rovnicu elipsy (obr. C).




Obr. A Obr. B Obr. C

v
a
0 x
0

2
x
0
x
a
0
a =
2
x

x
v
0 x
0
x
0
81
Prklad 2.7
Body K a L s spojen lanom dky l. Bod K sa pohybuje zo zaiatonej polohy K
0
, v ktorej
mal nulov rchlos, kontantnm zrchlenm a
K
po vodorovnej rovine. Urte drhu,
rchlos a zrchlenie bodu L ako funkciu asu t. Jeho zaiaton poloha je dan bodom L
0
.
Dan: a
K
= kont; l, h

Vsledok:
h t a h x + =
4 2
K
2
2
1
4
4 2
K
2
3 2
4
K
t a h
t a
v
+
=

2
3
K
] 4 [
) 12 (
4 2
K
2
2 2 4 2
K
2
t a h
t a t a h
a
+
+
=


Prklad 2.8
Pre zadan parametrick rovnice pohybu bodu M v kartezinskej sradnicovej sstave je
potrebn uri:
1. kinematick veliiny ako funkcie asu: v, v, a, a, a
t
, a
n
,
2. polomer krivosti trajektrie R
0
.
Dan:
2
1
t k x = [m], ) ln(bt k y
2
= [m],
1
k = 1,0 ms
2
,
2
k = 1,0 m, b = 1,0 s
1

Vsledok:
j i v
t
t
1
2 + =
2
1
2
4
t
t v + = j i a
2
1
2
t
=
4
1
4
t
a + =
1 4
1 4
4 2
4
+

=
t t
t
a
t

1 4
4
4
+
=
t
a
n

2
4
0
4
) 1 4 (
2
3
t
t
R
+
=

Prklad 2.9
Automobil sa pohybuje po priamoiarej ceste rchlosou v
0
. Jeho spomalenie je dan
vzahom kv a = [ms
2
] (k je kontanta [s
1
]). Poas spomalenia m zablokovan vetky
koles.
Vypotajte:
1. vekos rchlosti automobilu poas brzdenia ako funkciu asu v(t),
2. vekos brzdnej drhy ako funkciu asu x(t),
3. vekos rchlosti ako funkciu vekosti drhy v(x).
Vsledok:
kx v x v v e
k
v
t x x e v t v v
kt kt
= = = = = =

0
0
0
) ( ) 1 ( ) ( ) (





h
x
s
K
0
K

L
0
L

H

a
K
82
Prklad 2.10
Vypotajte jednotkov vektory prirodzenho sradnicovho systmu, ak je pohyb bodu
definovan polohovm vektorom:
k j i r t v t b t b
0
sin cos + + = ,
kde ] [m.s , ] [rad.s , [m]
-1 -1
0
v b s kontanty > 0.
Vsledok:
k j i k
j i j
k j i i

+
=
=

+
+

=
2
0
2 2 2
0
2 2
0
2
0
2 2
0
2
0
2 2
0
2
0
2 2 2
0
2 2
cos sin
sin cos
cos sin
v b
b
t
v b
v
t
v b
v
t t
v b
v
t
v b
b
t
v b
b
b
n
t





2.2 KINEMATIKA TUHHO TELESA
2.2.1 Posuvn pohyb telesa v rovine


Obr. 2.6
Teleso kon posuvn pohyb (obr.2.6) ak vektor
pevne spojen s telesom r
BA
pri pohybe nemen
svoju vekos ani smer. Vetky body telesa sa
pohybuj po zhodnch trajektrich.
Poda tvaru trajektrie je posuvn pohyb telesa:
priamoiary - drhy bodov s priamky,
krivoiary - drhy bodov s krivky.
Pre jednotliv vektory plat:
kont.
kont. ) (
kont. ) (
BA
A A
B B
=
=
=
r
r r
r r
t
t

kont.
kont. ) (
kont. ) (
BA BA
A A
B B
= =
=
=
r
t r
t r
r
r
r

Polohov vektor ubovonho bodu B telesa

BA A B
r r r + = . (2.37)
Rchlos bodu B telesa

A
BA
A B
v
r
v v
0
= + =
3 2 1
dt
d
. (2.38)
Zrchlenie bodu B telesa
i
j
x

y

krivoiary
posuvn pohyb
0

r
A
r
B
r
BA
A

B

r
BA
A

B

B

r
BA
A

priamoiary
posuvn pohyb
83

A B
a a = . (2.39)
Vetky body telesa konajceho posuvn pohyb maj rovnak rchlos a zrchlenie. Pri
posuvnom pohybe telesa sta skma pohyb jeho jedinho bodu.

2.2.2 Rotan pohyb telesa
Teleso kon rotan pohyb, ak jedna jeho priamka zostva trvalo v kude (obr. 2.7a). Tto
priamka je stla os rotcie s jednotkovm vektorom e. Vetky body na stlej osi rotcie maj
nulov rchlos. Rotan pohyb telesa je definovan, ak je znma asov zvislos uhla
pootoenia = (t), ktor zviera sprievodi ubovonho bodu s pevne zvolenou priamkou p.
asov zmena uhla pootoenia je charakterizovan uhlovou rchlosou
=

= = &
dt
d
t) ( [rad/s]. (2.40)
Uhlov rchlos je vektorov veliina. Je to vektor viazan s osou rotcie, t.j. plat
e = . (2.41)
asov zmena uhlovej rchlosti je charakterizovan uhlovm zrchlenm
=

= =

= = & & &


2
2
) (
dt
d
dt
d
t [rad/s
2
]. (2.42)
Uhlov zrchlenie je tie vektorov veliina, t.j. plat
e = . (2.43)
a)
b) c)
Obr. 2.7

Vetky body telesa sa pohybuj po zhodnch trajektriach (obr. 2.7b), ktor s sstrednmi
krunicami. Trajektria bodu B je teda krunica leiaca v rovine, ktor je kolm na os rotcie
telesa. Vzdialenos r ubovonho bodu od osi rotcie (bod 0) sa nemen a poloha bodu B je
uren polohovm vektorom r
BA
(
BA
r = r ).
Pre jednotliv vektory plat (obr.2.7c):

kont. ) (
kont.
kont. ) (
BA BA
A
B B
=
=
=
t
t
r r
r
r r

kont.
kont.,
kont., ) (
BA BA
A A
B B
= =
= =
=
r
r
t r
r
r
r

Polohov vektor ubovonho bodu B telesa

BA A B
r r r + = . (2.44)
Rchlos pohybu bodu B pri rotanom pohybe telesa vyjadren v tvare
A0
p

t
n
v
B
a
Bt
a
Bn
a
B
r
BA
B
r
A

r
B

r
BA

A
B
0
x
y
x`
y`

e
0



.
p
B
84

{
BA BA
BA
0
A B
B
r v
r r r
v = = + = =
dt
d
dt
d
dt
d
r v = =
B B
v ; ) (
BA
v . (2.45)
Zrchlenie bodu je definovan vzahom

. ) (
, ) (
B B BA BA B
BA
BA BA
BA B
B
n t
dt
d
dt
d
dt
d
dt
d
dt
d
a a r r a
r
r

r
v v
a
+ = + =
+ = = = =
(2.46)
Zrchlenie je pri rotanom pohybe telesa rozdelen na dve zloky:
tangencilne zrchlenie - le na dotynici k trajektrii bodu B

BA B
r a =
t
r a
t t
= =
B B
a ; ) (
BA
r , (2.47)
normlov zrchlenie - le na normle k trajektrii bodu B a smeruje do osi rotcie

BA BA B
) ( v r a = =
n
r a
n n
2
B B
= = a ; ) (
BA
v . (2.48)
Priebeh rotanho pohybu
a) Rovnomern rotan pohyb telesa: kont. = , 0 = =
&
.
Pre zaiaton podmienky: t = 0 s, =
0
je uhol pootoenia sprievodia bodu telesa:

0
+ = t .
b) Rovnomerne zrchlen (spomalen) rotan pohyb telesa: kont. = :
- rovnomerne zrchlen rotan pohyb: sgn = sgn ,
- rovnomerne spomalen rotan pohyb: sgn sgn .
Pre zaiaton podmienky - t = 0 s, (0) =
0
, (0) =
0
:
- uhlov rchlos:
0
) ( + = = t t ,
- uhol pootoenia:
0 0
2
2
1
+ + = t t .
c) Veobecn rotan pohyb telesa: . kont ) ( = t :
- nerovnomerne zrchlen rotan pohyb: sgn = sgn ,
- nerovnomerne spomalen rotan pohyb: sgn sgn .








85
2.2.3 Veobecn pohyb telesa v rovine

Veobecn pohyb telesa v rovine je mon rozdeli na translan pohyb, charakterizovan
pobybom referennho bodu - bod A (obr. 2.8) a rotan pohyb telesa okolo osi rotcie, ktor
prechdza referennm bodom.

a)

b)
Obr. 2.8

Pre jednotliv vektory plat:
kont. ) (
kont. ) (
kont. ) (
BA BA
A A
B B
=
=
=
t
t
t
r r
r r
r r

kont.
kont., ) (
kont., ) (
BA BA
A A
B B
= =
=
=
r
t r
t r
r
r
r

Polohov vektor bodu B je vyjadren v tvare
.
BA A B
r r r + = (2.49)
Rchlos a zrchlenie bodu B telesa (obr. 2.9) s uren pomocou prvej, resp. druhej
derivcie polohovho vektora r
B
poda asu

BA A BA A
B
B
dt
r v v v
r
v + = + = =
d
, (2.50)

4 43 4 42 1 3 2 1
BA
BA
BA
BA A BA A
2
B
2
B
B
) (
dt
dt
n t
d d
a
r
a
r a a a
r v
a + + = + = = = . (2.51)
Rchlos v
A
a zrchlenie a
A
reprezentuj translan pohyb telesa vzhadom na pevn
sradnicov systm (0, x, y).
Rchlos v
BA
a zrchlenie a
BA
reprezentuj rotan pohyb telesa okolo osi rotcie
prechdzajcej referennm bodom vzhadom na pohybliv sradnicov systm (A, x, y).

2.2.4 Okamit stred otania
Okamit stred otania - OSO - oznaen bodom K je bod, ktorho rchlos je v danom
asovom okamihu nulov. Na zklade (2.50) pre rchlos bodu K plat
0 r v v = + = ) (
KA A K
. (2.52)
Odtia pre polohu OSO vzhadom na pevn sradnicov systm plat

2
A
A KA A K

v
r r r r

+ = + = . (2.53)
Rchlos ubovonho bodu telesa vzhadom na OSO je uren vzahom

LK L
r v = ,
kde r
LK
je vzdialenos bodu L od OSO (bod K),
- vektor uhlovej rchlosti rotcie telesa okolo osi prechdzajcej OSO (bod K).
r
A
r
B
r
BA
A
B
0
x
y
x
y

0
r
A

r
B

r
BA

A
B
A= A
B
B
86

Obr. 2.9
Veobecne je okamit stred otania definovan ako priesenk
norml k trajektrim bodov danho telesa v danom okamihu.
Rchlos kadho bodu telesa v danom asovom okamihu vidme
z OSO pod rovnakm uhlom .
Grafick zostrojenie vektora rchlosti ubovonho bodu telesa
ak je znma poloha OSO a vektor rchlosti jednho bodu, je na
obrzku (obr. 2.10).
Vektor rchlosti danho bodu vdy le na dotynici, ktor je
kolm na normlu danho bodu. Uhol je orientovan uhol a je
orientovan od normly bodu vdy rovnakm smerom, t.j.
v smere otania uhlovej rchlosti . Koncov bod vektora
rchlosti je uren priesenkom ramena uhla s dotynicou
prslunho bodu.

Prklad 2.11

Kukov mechanizmus schematicky nakreslen
na obrzku sa sklad z kukovho hriadea C,
ojnice C a z piesta C. Kukov hriade sa ota
okolo bodu 0, priom uhol jeho rotcie je
kt =
21
, priom piest kon posuvn pohyb vo
vodorovnom smere.
Vypotajte vekos rchlosti a zrchlenia piesta.
Dan: r [m], l [m], k = 4 rad.s
1
v ase t = 0 s vetky leny sa nachdzaj na osi x.

Vzhadom na vzbu, v ktorej je piest uloen, me sa pohybova len vo vodorovnom smere.
Sradnice bodov piesta sa v smere osi y nemenia. V smere osi x je poloha piesta dan
vzdialenosou bodov od bodu 0. Vzhadom na to, e piest kon posuvn pohyb, vetky body
piesta maj rovnak rchlos a zrchlenie.
Na vpoet vekosti rchlosti piesta teda postauje vypota rchlos niektorho bodu piesta,
napr. bodu B, o je x-ov sradnica piesta C
+ = + = cos cos 0
21 B
l r SB S x .
Pomocn uhol je vyjadren z pravouhlho trojuholnka ASB, t.j. plat

l
r
21
sin
sin

= .
Pouitm vzahu
= = +
2 2 2
sin 1 cos 1 cos sin ,
je mon vzdialenos x
B
zapsa v tvare

2 2 2
21 B
) 4 sin ( 4 cos sin 1 cos t r l t r l r x + = + = .
Vekos rchlosti bodu B piesta
C
C

21
r

C
0
x
C
l

y
S B


x
B
A
v
L

r
LK

K
L
n
L

D

v
D

n
D
r
DK

t
D
= = =
DK
D
LK
L
tg
r
v
r
v
87

2 2
2
B B
) 4 sin (
8 sin 2
4 sin 4
t r l
t r
t r x v

= = & .
Vekos zrchlenia je zskan alou derivciou, t.j. plat

2
3
] ) 4 sin ( [
) 8 cos 1 ( 8 cos 4
4 4 cos 16
2 2
2 2 2
2
B B
t r l
t r t l
r t r v a

+
= = & .

Prklad 2.12

Zotrvank polomeru R sa ota tak, e poas doby rozbehu T pri
kontantnom uhlovom zrchlen vzrastie vekos uhlovej rchlosti
z hodnoty
1
[rad.s
1
]

na hodnotu
2
[rad.s
1
]

.
Vypotajte:

1. vekos uhlovho zrchlenia,
2. vekos rchlosti a zrchlenia bodu B na obvode zotrvanka na
konci rozbehu,
3. poet otoen zotrvanka poas rozbehu.
Dan: R = 0,9 m; T = 10 s;
1
= rad.s
-1
;
2
= 2,5 rad.s
-1


1. Vpoet vekosti uhlovho zrchlenia:
Veobecne je uhlov zrchlenie definovan vzahom
dt d
dt
d
=

= . (a)
Vzhadom na to, e uhlov zrchlenie je kontantn, predchdzajca rovnica spa
podmienky separcie premennch a jej integrovanm v prslunch hraniciach dostaneme

2 1 2
1 2
0
rad/s 47 , 0
2
1
=

= = =

=
=

T
T dt d
T t
t
.
2. Vpoet vekosti rchlosti a zrchlenia bodu B na konci rozbehu:
V okamihu dokonenia rozbehu m zotrvank uhlov rchlos
2
. Vekos rchlosti
bodu B je dan vzahom

1
2
ms 7 9 , 0 5 , 2

= = = R v ,
kde R je vzdialenos bodu od osi rotcie.
Vekos tangencilneho zrchlenia bodu B je

2
ms 423 , 0 9 , 0 47 , 0

= = = R a
t
.
Vekos normlovho zrchlenia bodu B je

2 2 2
2
ms 52 , 55 9 , 0 ) 5 , 2 (

= = = R a
n
.

Vekos vslednho zrchlenia bodu B je vypotan pomocou

2 2 2
ms 69 , 61

= + =
n t
a a a .
3. Vpoet potu pootoen zotrvanka p poas rozbehu.
Poet pootoen je mon vypota zo vzahu
B
R
88

=
2
R
p , (b)
kde
R
je uhol otoenia zotrvanka poas rozbehu.
Najskr je potrebn vyjadri asov zvislos uhlovej rchlosti. Pouitm vzahu (a)
dostaneme

1 1
0
1
+ = = =

t t d dt
t
.
asov zvislos uhla pootoenia je mon vyjadri z defincie uhlovej rchlosti, t.j. plat
t t dt t dt d dt d
dt
d
t t
1
2
0
1
0 0
2
1
) ( + = + = = =


Do predchdzajceho vzahu je dosaden za as t doba rozbehu rotora T = 10 s, m je
vyjadren vekos uhla pootoenia zotrvanka poas rozbehu
rad 92 , 54
2
1
1
2
R
= + = T T .
Poet pootoen zotrvanka poas rozbehu je potom zo vzahu (b)
74 , 8
2
=

=
R
p otok.

Prklad 2.13
Na bubon je navinut lano, na konci ktorho je zavesen bremeno pohybujce sa zrchlenm
a = kont. K bubnu s pevne pripojen ramen dky l, ktor rotuj spolu s bubnom.
V zvislosti od drhy s bremena vypotajte rchlos a zrchlenie bodov nachdzajcich sa
na koncoch ramien. Na zaiatku pohybu v ase t = 0 s je rchlos bremena nulov.

Obr. A

Obr. B

Ramen pripojen na navjac bubon konaj spolu s bubnom rotan pohyb. Na zaklade toho
body na konci ramien konaj pohyb po krunici.
Rchlos pohybu bremena v zvislosti od prejdenej drhy je uren nasledovnm postupom:

ds
dv
v
ds
dv
dt
ds
ds
ds
dt
dv
dt
dv
kont a = = = = = . ,
odkia po separcii premennch a nslednej integrcii plat
l
r
l
s
a = kont.
l
r
l
s
a = a
t
= kont.
A


a
A
v
A
a
nA
v

89
ads vdv =

=
s v
ads vdv
0 0
as
v
=
2
2
as s v 2 ) ( = .
Rchlos pohybu bremena je rovnak ako obvodov rchlos bodov na obvode otajceho
sa navjacieho bubna, t.j. pre uhlov rchlos navjacieho bubna plat
r v = r v =
r
as
r
s v
s
2 ) (
) ( = = .
Zrchlenie pohybu bremena je rovnak ako tangencilna zloka zrchlenia bodov na obvode
bubna a pre uhlov zrchlenia dostaneme vzah
r a a = = = . kont
t
r a =
r
a
= .
Vekos rchlosti bodu A mono vyjadri v tvare

A A
r v = ) (
2
) )( ( ) (
A
l r
r
as
l r s s v + = + = .
Pre zrchlenie bodu A plat
) (
A A A A A
r r a a a + = + =
n t
.
Vekosti zloiek zrchlenia bodu A
. ) (
A
kont
r
l r
a l r a
t
=
+
= + =
s
r
l r
a s v s s a a
n n
2
A A A
2 ) ( ) ( ) (
+
= = = .
Vekos vslednho zrchlenia

2 2
2
2
A
2
A A A
4 ) ( s r
r
l r
a a a s a a
n t
+
+
= + = = .

Prklad 2.14
Dve ozuben koles s vonkajm ozubenm sa otaj rovnomerne zrchlene. Je dan
vekos uhlovho zrchlenia kolesa 1. Na zaiatku pohybu boli obe koles v kude.
Vypotajte:
1. vekos uhlovej rchlosti druhho kolesa ako funkciu asu,
2. vekos rchlosti a vekos zrchlenia v zvislosti od asu pre bod C, ktor sa nachdza
na kolese 2 vo vzdialenosti r
C
od osi rotcie.
Dan: d
1
; d
2
; r
C
;
1
= kont.


1. Ozuben koles konaj rotan pohyb okolo stlej osi rotcie, ktor je kolm na rovinu
otania kolies a prechdza bodmi 0
1
(koleso 1) a 0
2
(koleso 2) V zbere zubov (bod D)
oboch ozubench kolies mus by rovnak rchlos a rovnak tangencilne zrchlenie.
Zrove mus by rovnak obvodov rchlos a rovnak tangencilne zrchlenie pre

C
C
D

v
D
a
D
v
C
a
Ct a
Cn
a
C

1

d
1 d
2

2
C
C
r
C
D

0
1
0
2
90
vetky body nachdzajce sa na obvodoch oboch rozstupovch krunc. Na zklade
vzahu pre tangencilne zrchlenie je uren vekos uhlovho zrchlenia kolesa 2
kont.
2 2
2
1
1 2
2
2
1
1
= = = =
d
d d d
a
t

Veobecne na vyjadrenie uhlovho zrchlenia plat
dt
d
2
2

= .
Separciou premennch a integrovanm v prslunch hraniciach je uren vekos uhlovej
rchlosti
2
ako asov funkcia, t.j.
t
d
d
t dt d dt d
t
2
1
1 2 2
0
2
0
2 2 2
2
= = = =

.
2. Vekos rchlosti bodu C je vypotan zo vzahu

C
2
1
1 C 2 C
r t
d
d
r v = = .
Tangencilna zloka zrchlenia a normlov zloka zrchlenia bodu C s

C
2
1
1 C 2 C
r
d
d
r a
t
= = ,
C
2
2
1
1 C
2
2 C
r t
d
d
r a
n

= = .
Vekos vslednho zrchlenia bodu C urme pomocou vzahu
1
2
2
1
1
4
2
1
1 C
2
C
2
C C
+

= + =
d
d
t
d
d
r a a a
t n
.

Prklad 2.15
Vypotajte rchlos bodu A a uhlov rchlos a uhlov zrchlenie tye AB dky l
0
. Na
zaiatku bola ty v kude a jej body A a B sa nachdzali v polohe A
0
, B
0
. Ty sa uvdza do
pohybu bodom B, ktor sa zane pohybova v ase t = 0 s vodorovne kontantnou
rchlosou vekosti v.


Teleso kon veobecn pohyb. Drha bodu B je v zvislosti od asu vyjadren nasledovne:
v

B
A
B
0
A
0
y
A
x

y

0

x
B
x
0 vt


i

j


k

v
l
0

B
A
B
0
A
0
h

x
0
91

0 B
0
B
0
kont. x vt x vdt dx vdt dx
dt
dx
v
t x
x
+ = = = = =

.
Parametrick rovnice pohybu bodu A s vzhadom na zvolen sradnicov systm vyjadren
v tvare

2
0
2
0
2
B
2
0 A A
) ( y 0, x vt l x l x + = = = .
Kinematick veliiny bodu A s:
polohov vektor
j r
2
0
2
0 A
) ( x vt l + = ,
vektor rchlosti
j j
r
v
2
0
2
0
0
2
0
2
0
0 A
A
) (
) (
) (
) ( 2
2
1
x vt l
x vt v
x vt l
x vt v
dt
d
+
+
=
+
+
= = ,
vektor zrchlenia
j
v
a
2
3
] ) ( [
2
0
2
0
2
0
2
A
A
x vt l
l v
dt
d
+
= = .
Na zaiatku pohybu zvierala ty s osou y uhol
0
, ktorho vekos je uren zo vzahu

0
0
0
0
0
0
arctg tg
l
x
l
x
= = .
Uhol sklonu tye je poas jej pohybu dan funkciou asu a mono ho vyjadri v tvare

0
= .
Uhol mono vyjadri v tvare

0
0
0
0
arctg tg
l
x vt
l
x vt +
=
+
=
a nsledne pre uhol potom plat

0
0
0
0
arctg arctg
l
x
l
x vt

+
= .
Uhlov rchlos tye je
k =
+ +
= = ;
) (
2
0
2
0
0
x vt l
v l
& ,
kde k je jednotkov normlov vektor kolm na rovinu a vchdza do roviny.
Uhlov zrchlenie tye je
k =
+ +
+
= = ;
] ) ( [
) ( 2
2 2
0
2
0
0
2
0
x vt l
x vt v l
& .
92
Prklad 2.16
Na valci s polomerom r je navinut lano, ktorho jeden koniec je pevne uchyten na rm
a druh je upevnen na valec. Valec pad bez zaiatonej rchlosti, priom sa lano
rozmotva. Vekos rchlosti osi valca je . 3
3
2
gy v = Vypotajte:
1. rovnice pohybu valca,
2. polohov vektor okamitho stredu otania vzhadom na pevn a pohybliv sradnicov
systm,
3. rchlos bodov D, C nachdzajcich sa na obvode valca poda obrzka,
4. vekos rchlosti bodov D, C pomocou okamitho stredu otania.

Obr. A

Obr. B

Obr.C



Obr. D



Obr. E

Valec kon veobecn rovinn pohyb. Prslun sradnicov systmy s na obr. B. Pevn
sradnicov systm je (T
0
, x, y, z) s jednotkovmi vektormi (i, j, k). Pohybliv sradnicov
systm je (T, x, y, z) s jednotkovmi vektormi (i, j, k). Vzhadom na rovnobenos
sradnicovch os oboch systmov je mon pre jednotkov vektory psa
i = i, j = j, k = k.
1. Njs rovnice pohybu znamen vyjadri sradnicu y
T
=

y
T
(t) pre posuvn pohyb referennho
bodu telesa a uhol pootoenia = (t) pre rotan pohyb telesa okolo osi rotcie
prechdzajcej referennm bodom. Referenn bod (stred valca - bod T) sa pohybuje po
priamke. Vekos jeho sradnice v smere osi x je nulov. Pre vekos rchlosti v smere
osi y mono preto psa
gy
dt
dy
v v
y
3
3
2
= = = .
Separciou premennch a integrovanm pre nulov zaiaton podmienky dostvame

2
T
0 0
3
3
3
2
T
t
g
y dt g
y
dy
t y
= =

.
asov zvislos vekosti rchlosti a zrchlenia stredu valca je
g
dt
dv
a t
g
dt
dy
v
3
2
,
3
2
T
T
T
T
= = = = .
Uhol pootoenia valca okolo aiska je vyjadren zo vzahov pre uhlov rchlos
t
r
g
r
v
dt
d
3
2
T
= =

= .
r
T
v
T
C
D T
v
T
T
0 x
x
y y
y
T
z
z

T
r
T
T
0 x
x
y y
k

z
z

K

r
K
r
KT
T
0
T
r
T
x
y y
K

r
DT
D
C
r
CT
n
T,
n
D T
K

D
C
v
T

t
D
v
D
n
C

t
C v
C
r
r

93
Separciou premennch a integrovanm pre nulov zaiaton podmienky dostvame

2
0 0
3 3
2
t
r
g
tdt
r
g
d dt d
t
= = =

.
Vekos uhlovej rchlosti a uhlovho zrchlenia valca je

r
g
dt
d
t
r
g
dt
d
3
2
3
2
=

= =

= .
Vyjadren veliiny s kinematick veliiny charakterizujce veobecn pohyb telesa
v rovine a pomocou nich mono uri rchlos a zrchlenie ktorhokovek bodu telesa.
Kee s to vektorov veliiny, mono ich zapsa v tvare vektorov, t.j.
j j a j j v j j r

= =

= =

= =
3
2
,
3
2
,
3
T T T T
2
T T
g
a t
g
v t
g
y ,
k k k k

= =

= =
r
g
t
r
g
3
2
,
3
2
.
2. Polohov vektor okamitho stredu otania - OSO (bod K) vzhadom na pohybliv
sradnicov systm (Obr. C) je dan vzahom
i j k
v
r r
gt
r
gt
t
r
g
=

=
3
2
3
2
9
4
1

2
2
2 2
T
KT
.
Vzhadom na pevn sradnicov systm polohov vektor okamitho stredu otania je
i j i j r r r r t
g
r y

= = + =
2
T KT T K
3
.
Body na priamke k sa postupne stvaj okamitmi stredmi otania (obr. C).
3. Na zklade znmych kinematickch velin s vyjadren vektory a vekosti rchlost
ubovonch bodov telesa.
Polohov vektor bodu D vzhadom na referenn bod T je
i r r =
DT
.
Rchlos bodu D je
j j j i k j r v v
T T T T DT T D
2 v v v r v = + = + = + =
a jeho vekos rchlosti je

T D
2v v = .
Poloha bodu C vzhadom na referenn bod T je
j r r =
CT
.
Rchlos bodu C je
i j j k j r v v
T T T CT T C
v v r v + = + = + =
a vekos rchlosti bodu C je
2
T
2
T
2
T C
v v v v = + = .
94
4. Vpoet a zobrazenie vektorov rchlost bodov C a D (obr. E).
Okamitm stredom otania prechdzaj v danom asovom okamihu normly vetkch
bodov telesa a vidie z neho koncov body vektorov rchlosti vetkch bodov pod
rovnakm uhlom . Uhol mono zostroji, ak je znmy vektor rchlosti aspo jednho
bodu telesa - napr. bodu T.
Pre vekosti rchlost bodov C a D (obr. E) mono psa

T D
D T
2
2
tg v v
r
v
r
v
= = = ,
2 tg
T C
2 2
C T
v v
r r
v
r
v
=
+
= = .

Prklad 2.17
Ozuben skolie je zloen z dvoch navzjom pevne spojench ozubench kolies
s polomermi rozstupovch krunc r a R. Kad z ozubench kolies je v zbere s ozubenm
hrebeom, priom vie ozuben koleso je v zbere s dolnm hrebeom, ktor sa pohybuje
rchlosou v
1
a zrchlenm a
1
a s menm ozubenm kolesom je v zbere horn hrebe
pohybujci sa rchlosou v
2
a zrchlenm a
2
.
Vypotajte rchlos a zrchlenie bodu T a uhlov rchlos a uhlov zrchlenie ozubenho
skolia

Obr. A

Obr. B

Vzhadom na charakter pohybu kon ozuben skolie veobecn pohyb. Vyuitm vzahov
(2.49) a (2.50) s polohov vektory bodov T a B vyjadren nasledovne:

BA A B
r r r + = ,

TA A T
r r r + = .
Pre rchlosti oboch bodov potom plat

BA A BA A B
r v v v v + = + = , (a)

TA A TA A T
r v v v v + = + = , (b)
kde je vektor uhlovej rchlosti otania ozubenho skolia.
T
B
A
r
R
v
1
= v
A

v
2
= v
B
a
2

a
1

OSO
v
T

v
TA

v
BA

r
TA

r
BA

e
;
|e| = 1

T
B
A
r
R
v
1

v
2

a
2

a
1

95
Vzhadom na to, e vetkch p vektorov rchlost v
A
, v
B
, v
T
, v
BA
, v
TA
m smer jednej
nositeky, je mon rovnice (a) a (b) pretransformova do skalrnych rovnc tak, e obidve
rovnice s skalrne vynsoben jednotkovm vektorom e, t.j.

BA A B
v v v + = /.e
BA A B
v v v + = , (c)

TA A T
v v v + = /.e
TA A T
v v v + = . (d)
Pre jednotliv vekosti rchlosti plat

1 A
v v =
1 A
v v = ,

2 B
v v =
2 B
v v = ,

BA BA
r v = ) (
BA
R r v + = ,

TA TA
r v = R v =
TA
.
Po dosaden predchdzajcich rovnc do (c) a (d)
) (
1 2
R r v v + + = ,
R v v + =
1 T
,
odkia s vyjadren vekosti rchlost:
rchlos bodu T
R r
R v r v
v
+
+
=
2 1
T
,
uhlov rchlos skolia
R r
v v
+

=
1 2
.
Vekosti zrchlen s zskan derivciou rchlosti v
T
a uhlovej rchlosti poda asu, t.j.
zrchlenie bodu T
R r
R a r a
a
+
+
=
2 1
T
,
uhlov zrchlenie skolia
R r
a a
+

=
1 2
.

Prklad 2.18
Vypotajte rchlos a zrchlenie zdvhacieho stola, ktor je zdvhan mechanizmom.
Dan: l, v = kont

Vsledok:
j v



= v
vt l l
vt l
2
0
2
0
) (

[ ]
j a
2
2
0
2
2
2
3
v
vt l l
l
) (
=

v
l/2
l/2
l/2
l/2
l
0
96
Prklad 2.19
Zotrvank kon rotan pohyb, priom vzhadom na sradnicov sstavu je jeho poloha
definovan uhlom potoenia bt kt ln
2
= [rad], k = 1,0 rad.s
2
, b = 1,0 s
1
. Vypotajte
asov zvislosti vekosti uhlovej rchlosti = (t) a vekosti uhlovho zrchlenia = (t).
Vsledok: t t t + = ln 2 [rad.s
-1
], 3 2 + = t ln [rad.s
-2
]

Prklad 2.20

Vypotajte rchlos bodu T a uhlov
rchlos telesa 3 zdvhacieho kladkovho
systmu s kladkami s polomermi r
1
, r
2
a r
3
.
Kladky 1 a 2 s pohan a otaj sa
kontantnmi uhlovmi rchlosami
1
,
2
.
Vsledok:
2
2 2 1 1
T
r r
v
+
= [m.s
-1
]
3
1 1 2 2
3
2r
r r
= [rad.s
-1
]



2.3 RCHLOS A ZRCHLENIE PRI SASNCH POHYBOCH
Uvaujme tri teles T
1
, T
2
, T
3
. S telesom T
1
je spojen nepohybliv sradnicov systm
(0, x, y, z). S telesom T
2
je spojen pohybliv sradnicov systm (A, x,y,z). Bod A je
bodom telesa T
2
a bod B je bodom telesa T
3
(obr. 2.11). Vzjomn pohyby telies s:
pohyb telesa T
3
vzhadom na teleso T
1
- vsledn pohyb,
pohyb telesa T
2
vzhadom na teleso T
1
- unav pohyb,
pohyb telesa T
3
vzhadom na teleso T
2
- relatvny pohyb.

Obr. 2.11
Cieom je vyjadri rchlos a zrchlenie bodu B telesa
T
3
pri pohybe vzhadom na T
1
. t.j. vslednho pohybu.
Polohov vektor bodu B

32 B 21 A 31 B
r r r + = . (2.54)
Vsledn rchlos bodu B je zskan derivciou
vektora r
B31
poda asu v priestore telesa T
1



+ =
1 32 B 1 21 A 1 31 B
] [ ] [ ] [ r r r . (2.55)
Vzhadom na to, e vektor r
BA32
je vektorov funkcia
definovan v priestore telesa T
2
a asov derivcia m
by v priestore telesa T
1
, je potrebn tto derivciu
realizova v slade s pravidlami na derivovanie funkci
v rznych priestoroch.

2
r
1
r
2
r
3
T

r
A21

r
B31
r
B32

A
B
0 x
y
x
y
z
T
1
z
T
2
T
3
97
Vo veobecnosti, ak je asovo zvisl vektorov funkcia r(t) dan v priestore telesa T
m
, jej
derivcia poda asu v priestore telesa T
n
je vyjadren vzahom

m mn m m n m
] [ ] [ r r r + =

. (2.56)
Vzah (2.55) potom mono zapsa

32 B 21 2 32 B 1 21 A 1 31 B
] [ ] [ ] [ r r r r + + =

,
resp.
32 B 21 32 B 21 A 31 B
r v v v + + =
r u v B B B
v v v + = , (2.57)
kde
31 B B
v v =
v
- vsledn rchlos bodu B ) T T pohyb , T B (
1 3 3
,

32 B 21 21 A B
r v v + =
u
- unav rchlos bodu B ) T T pohyb , T B (
1 2 2
, (2.58)

2 B3 B
v v =
r
- relatvna rchlos bodu B ) T T pohyb , T B (
2 3 3
,

21 A
v - unav rchlos bodu A ) T T pohyb , T A (
1 2 2
,

u
=
21
- unav uhlov rchlos telesa T
2
(pohyb T
2
T
1
).
Vsledn zrchlenie pri sasnch pohyboch dvoch telies je zskan derivciou prvej z rovnc
(2.57) poda asu v priestore T
1



+ + + =
1 32 B 21 32 B 1 21 1 32 B 1 21 A 1 31 B
] [ ] [ ] [ ] [ ] [ r r v v v
, (2.59)
resp. po pouit vzahu (2.56) a nslednej prave m vsledn zrchlenie bodu B tvar

cor r u v B B B B 31 B
a a a a a + + = = , (2.60)
kde
32 B 21 32 B 21 21 21 A B
) ( r r a a + + =
u
- unav zrchlenie bodu B , T B (
2

) T T pohyb
1 2
,

2 B3 B
a a =
r
- relatvne zrchlenie bodu B ) T T pohyb , T B (
2 3 3
,

32 B 21 B B
2 2 v v a = =
r u cor
- Coriolisovo zrchlenie bodu B,

21 A
a - unav zrchlenie bodu A ) T T pohyb , T A (
1 2 2
,

u
=
21
- unav uhlov zrchlenie telesa T
2
) T T pohyb (
1 2
.
Uhlov rchlos a uhlov zrchlenie pri sasnch pohyboch
Ak s unav pohyb (T
2
T
1
) a relatvny pohyb (T
3
T
2
) telesa rotan pohyby, potom
uhlov rchlos
31
vslednho pohybu (T
3
T
1
) telies je mon vyjadri v tvare

21 32 31
+ =
u r v
+ = . (2.61)
Uhlov zrchlenie vslednho pohybu (T
3
T
1
) je zskan derivciou (2.61) poda asu
v priestore T
1


+ + = + =
1 21 32 21 2 32 1 21 1 32 1 31
] [ ] [ ] [ ] [ ] [ , (2.62)
resp.
32 21 21 32 31
+ + =
res u r v
+ + = ,
kde
31
=
v
- vsledn uhlov zrchlenie telesa T
3
) T T pohyb (
1 3
,

21
=
u
- unav uhlov zrchlenie telesa T
2
) T T pohyb (
1 2
,

32
=
r
- relatvne uhlov zrchlenie telesa T
3
) T T pohyb (
2 3
,
98

32 21
=
res
- Ressalovo uhlov zrchlenie.
Prklad 2.21

Obr. A
Rrka dky l rotujca okolo bodu 0 sa ota
kontantnou uhlovou rchlosou . V rrke
sa kontantnm zrchlenm a pohybuje bod
B. V ase t = 0 s sa rrka nachdzala vo
vodorovnej polohe a bola v kude. Bod B
bol v ase t = 0 s v zaiatonej polohe vo
vzdialenosti s
0
od bodu 0.


Vypotajte rchlos a zrchlenie bodu B
v zvislosti od asu vzhadom na pevn
sradnicov systm.
Dan:
= kont., a = kont., l, s
0


lohu mono chpa ako sasn pohyby dvoch telies. Rrka C kon rotan pohyb okolo
stlej osi rotcie - unav pohyb telesa T
2
T
1.
Bod 0 A je referennm bodom telesa C.
Bod B (teleso C) sa pohybuje vo vntri rrky relatvny pohyb telesa T
3
T
2
. S telesom C
je zviazan v bode 0 pevn sradnicov systm (0, x, y, z). S telesom C je zviazan v bode
A pohybliv sradnicov systm (A, x, y, z). Osi x a x, resp. y a y sradnicovch
systmov s navzjom pootoen o uhol , preto plat (obr. A)
k k j i j j i i = + = + = , cos sin ; sin cos . (a)
Vekos rchlosti bodu B v rrke
at v adt dv
dt
dv
a
t v
= = = =

0 0
. kont
a pre vekos drhy plat

2
0 0 0
2
1
. kont at s atdt vdt ds vdt ds
dt
ds
v
t t s
= = = = =

.
Vekos uhla potoenia telesa C
t dt d
dt
d
t
= = =

0 0
. kont .
I. spsob rieenia pomocou vzahov pre sasn pohyby.
Polohov vektor referennho bodu A telesa C vzhadom na teleso C
0 r =
21 A
. (b)
Polohov vektor bodu B telesa C vzhadom na referenn bod A telesa C
C
C


0 = A

x
y
a
C


s
0
L
l

x
y
B
s(t)
i
j
i
j


99
i i r + = + = ) ( )) ( (
B
2
2
1
0 0 32
at s t s s (c)
a vektor jeho rchlosti
) sin (cos
B
j i i i v + = = = at at v
32
.
Polohov vektor bodu B vzhadom na teleso C

32 21 31 B A B
r r r + = . (d)
Pre vektor rchlosti bodu B telesa C plat (2.57)

{
32 21 32 21 31 B B A B
r v v v
0
+ + =
=
, (e)
kde k =
21
je uhlov rchlos unavho pohybu.
Po dosaden za jednotliv leny do rovnice (e) je vektor rchlosti vslednho pohybu bodu B

. ] cos ) ( sin [ ] sin ) ( cos [
)], sin )(cos [( ) sin (cos
, ) (
2
2
1
0
2
2
1
0 31 B
2
2
1
0 31 B
2
2
1
0 31 B
j i v
j i k j i v
i k i v
+ + + + =
+ + + + =
+ + =
at s at at s at
at s at
at s at
.
Pre vektor zrchlenia bodu B na telese C plat (2.60), t.j. plat
] [
B B B B A B 32 21 21 32 21 32 21 32 21 31
2 r r v a a a + + + + = ,
kde 0 a =
21 A
,
j i i a + = = sin cos
B3
a a a
2
,
j i j i k v + = = = cos sin
B3
at at at at 2 2 2 2 2
2 21
,
0 r =
32 21 B
( = kont. = 0),
) sin )(cos ( ] [
B
j i r + + =
2
2
1
0
2
32 21 21
at s .
Vsledn zrchlenie bodu B vzhadom na pevn sradnicov systm

. ] sin ) ( cos 2 sin [
] cos ) ( sin 2 cos [
2
2
1
0
2
2
2
1
0
2
1 B3
j
i a
+ + +
+ =
at s at a
at s at a

II. spsob rieenia - vyjadrenm polohy guky vzhadom na (0, x ,y, z) ako funkcie asu.
j i r + + + = sin ) ( cos ) (
2
2
1
0
2
2
1
0 31 B
at s at s ,
kde uhol pootoenia t = je funkciou asu.
Pre derivcie uhla pootoenia poda asu
0 , = = & & & .
Derivciou polohovho vektora
31 B
r poda asu dostaneme:
vektor rchlosti bodu B vzhadom na C
j i v ] cos ) ( sin [ ] sin ) ( cos [
2
2
1
0
2
2
1
0 31 B
+ + + + = at s at at s at & & ,
vektor zrchlenia guky B vzhadom na C
100

. ] sin ) ( cos sin [
] cos ) ( sin cos [
2
2
1
0
2
2
2
1
0
2
31 B
j
i a
+ 2 + +
+ 2 =
at s t a a
at s t a a
& &
& &

Pouitm obidvoch spsobov rieenia boli zskan rovnak vsledky.
Prklad 2.22
Rameno AL (teleso C) je otone uloen na vertiklnej tyi (teleso C), okolo ktorej sa ota
kontantnou uhlovou rchlosou vekosti
1
. V bode L ramena je otone uloen disk
(teleso C) s polomerom r
2
. Disk sa ota okolo osi y prechdzajcej bodom L a kolmej na
rovinu disku kontantnou uhlovou rchlosou vekosti
2
.
Vypotajte rchlos a zrchlenie vslednho pohybu bodu nachdzajceho sa na obvode
disku vzhadom na pevn sradnicov systm x, y, z.
Dan:
1
= kont.,
2
= kont., r
1
, r
2




V rieenej lohe ide o sasn pohyby telies C a C, obe teles konaj rotan pohyb.
Rameno AL kon rotan pohyb okolo vertiklnej osi (os z) - unav pohyb T
2
T
1.
Bod L
je referennm bodom telesa C. Disk C kon rotan pohyb okolo osi rotcie prechdzajcej
bodom L (os y) - relatvny pohyb T
3
T
2
. S telesom C je pevne zviazan sradnicov
systm (0, x, y, z). S telesom C je zase zviazan sradnicov systm (L, x, y, z). Pre
jednotkov vektory sradnicovch systmov plat

.
, cos sin
, sin cos
21 21
21 21
k k
j i j
j i i
=
+ =
+ =
(a)
Veobecne na vyjadrenie (pre pohyb T
3
T
1
) polohovho vektora
31 B
r , vektora rchlosti
31 B
v a vektora zrchlenie
31 B
a platia vzahy

). ( 2
,
,
32 BL 21 21 32 BL 21 32 BL 21 32 BL 21 L 31 B
2 BL3 21 2 BL3 1 L2 1 B3
2 BL3 1 L2 1 B3
r r v a a a
r v v v
r r r
+ + + + =
+ + =
+ =
(b)
Vyjadrenie potrebnch kinematickch velin vo vektorovom tvare

. ] [ , cos sin
, ] [ ,
0 j i j
0 k
= = = =
= = =

2 32 32 21 2 21 2 2 32
1 21 21 1 21
(c)
Bod L sa pohybuje (pre pohyb T
2
T
1
) po krunici s polomerom r
1
a so stredom v bode A.
Polohov vektor, vektor rchlosti a vektor zrchlenia bodu L
C
C
A = 0

y
z
C

1
r
2
L
r
1
y
z
B

2
x
x
z
r
2

r
1

x
y
x
y
z
k
i
j

21

32
i
j
k

21
=
1

32
=
2

101

. sin cos
) ( ] [
, sin cos ] [
, sin cos
L
L L L L
L L L
L
j i
r r v a
j i r r v
j i r
v 0
21 1
2
1 21 1
2
1
21 21 21 21 21 1 21 21
21 1 1 21 1 1 21 21 1 21 21
21 1 21 1 21
21
=
= + = =
+ = = =
+ =
= =

r r
r r
r r
43 42 1 43 42 1
(e)
Bod B sa pohybuje (pre pohyb T
3
T
2
) po krunici s polomerom r
2
a so stredom v bode L.
Polohov vektor, vektor rchlosti a vektor zrchlenia bodu B

, ] [
, sin cos sin cos cos
sin cos
2 BL3 32 2 2 BL3 2 BL3
32 2 32 21 2 32 21 2
32 2 32 2 2 BL3
r r v
k j i
k i r
= =
+ + =
= + =

r r r
r r


, cos sin sin sin cos
sin cos sin cos cos
0 cos sin
] [
32 32 21 32 21 2 2
32 32 21 32 21
21 21 2 2 2 BL3
k j i
k j i
v
+ =
=

=
r
r
(f)
, ) ( ] [
2 BL3
2 BL3 32 32 2 BL3 32 2 2 BL3 2 BL3
4 43 4 42 1 43 42 1
v 0
r r v a
= =
+ = =



. sin cos sin cos cos
cos sin sin sin cos
0 cos sin
] [
32 32 21 32 21 2
2
2
32 32 21 32 21
21 21 2
2
2 2 BL3
k j i
k j i
a
+ + =
=

=
r
r

alie leny v rovniciach (b) s vyjadren nasledovne

, cos cos cos sin
sin cos sin cos cos
1 0 0
] [
32 21 32 21 2 1
32 32 21 32 21
2 1 2 BL3 21
j i
k j i
r
+ =
=

=
r
r


, cos sin cos cos
cos cos cos sin
) (
] [
BL3
j i
k j i
r
32 21 32 21 2
2
1
32 21 32 21
2
2
1 2 21 21
0
1 0 0
=

=
r
r
(g)
102

. sin cos sin sin
cos sin sin sin cos
] [
BL3
j i
k j i
v
32 21 32 21 2 2 1
32 32 21 32 21
2 2 1 2 21
2
1 0 0 2 2
=

=
r
r

Po dosaden (c)-(g) do rovnc (b) po prave dostaneme
k j i r
32 2 21 32 2 1 21 32 2 1 1 B3
sin sin ) cos ( cos ) cos ( + + + + = r r r r r ,

, ] cos [ ] cos cos sin ) sin [(
] cos sin cos ) sin ( [
B3
k j
i v
32 2 2 32 21 2 1 21 32 2 2 1 1
32 21 2 1 21 32 2 2 1 1 1
+ + +
+ + =
r r r r
r c r r


. ] sin [
)] sin cos cos sin ( sin ) cos ( [
)] sin sin cos cos ( cos ) cos ( [
B3
k
j
i a
32 2
2
2
32 21 1 32 21 2 2 2 21 32 2 1
2
1
32 21 1 32 21 2 2 2 21 32 2 1
2
1 1
2
2
+
+ + + +
+ + =
r
r r r
r r r

Pre vsledn uhlov zrchlenie (pre pohyb T
3
T
1
) plat

{ {
]. sin cos [
cos sin
j i
k j i

0

0
21 21 2 1
21 2 21 2
1 32 21 32 32 21 21 31
0
0 0
+ =

= = = + + =
=
=
=
res
res
res
43 42 1


2.4 ANALYTICK RIEENIE MECHANIZMOV
Cieom analytickho rieenia mechanizmov je uri polohu, rchlos a zrchlenie
ubovonho bodu ktorhokovek lena mechanizmu alebo polohu rchlos a zrchlenie
niektorho lena mechanizmu v zvislosti od polh, rchlost a zrchlen hnacch lenov
mechanizmu.
2.4.1 Trigonometrick metda
Mechanizmus je v uritej vzjomnej konfigurcii lenov rozdelen na vhodn trigonometrick
obrazce, trojuholnky, v ktorch figuruj hadan dky a uhly ako parametre mechanizmu.
Kinematick veliiny (rchlosti a zrchlenia, resp. uhlov rchlosti a uhlov zrchlenia)
bodov, resp. lenov mechanizmu s potom zskan poda vzahov uvedench
v predchdzajcich kapitolch. Je potrebn njs toko vzahov medzi polohovmi veliinami
hnacch a hnanch lenov, koko je neznmych.
2.4.2 Vektorov metda
truktra mechanizmu je nahraden uzatvorenm mnohouholnkom, ktorho strany tvoria
vektory
i
r , navzjom sa nasledujce a tvoriace uzavret vektorov sluku. Pre set vektorov
vo vektorovej sluke potom mus plati podmienka uzavretosti, t.j.
103
0 j i r = + =

= =
n
i
i i
n
i
i
y x
1 1
) ( . (2.63)
Vektorov rovnicu (2.63) mono rozpsa do dvoch skalrnych rovnc
, 0 sin , 0 cos
1 1 1 1
= = = =

= = = =
i
n
i
i
n
i
i i
n
i
i
n
i
i
r y r x (2.64)
kde
i
s smerov uhly vektorov a
i
r s dky vektorov nahrdzajcich jednotliv leny.
Z rovnc (2.64) mono vyjadri dva parametre na definovanie polohy, resp. uhla pootoenia
lena, ktorho kinematick parametre s potan.
2.4.3 Metda komplexnch sel
Postup pri rieen metdou komplexnch sel je podobn ako pri vektorovej metde.
Rovina mechanizmu je povaovan za komplexn rovinu a kinematickej schme mechanizmu
je priraden uzatvoren mnohouholnk. Strany mnohouholnka s komplexn sla. Pre set
komplexnch sel
i
z mus plati podmienka uzavretosti, t.j. plat
0
1
=

=
n
i
i
z . (2.65)

Obr. 2.12
Tvar vyjadrenia komplexnho sla:
kartezinsky
i i i
jb a z + = ,
exponencilny
i
j
i i
e z

= ,
polrne sradnice ) sin (cos
i i i i
j z + = ,
kde 1 = j je imaginrna jednotka,
- meriame od kladnho smeru relnej osi,
Re - relna os,
Im - imaginrna os.
Rovnicu (2.65) mono rozpsa do dvoch rovnc
0
1
=

=
n
i
i
a 0
1
=

=
n
i
i
b ,
resp.
0 cos
1
=

=
i
n
i
i
. 0 sin
1
=

=
i
n
i
i

Z uvedench rovnc, z ktorch s vyjadren dva parametre na definovanie polohy, resp. uhla
pootoenia lena, ktorho kinematick parametre s potan.




Re
Im
0
z
i

i
a
i
b
i
104
Prklad 2.23
Njdite sradnice bodu B lena C pri jeho pohybe vzhadom na len C mechanizmu poda
obrzka.
Dan: l, h, = ( t)


Obr. A

Obr. B

Obr. C

1. spsob rieenia - trigonometrick metda (obr. A)
Zakreslme polohov vektor bodu B vzhadom na zaiatok sradnicovho systmu. Z
pravouhlho trojuholnka je uren vzdialenos 0C

= =
tg
0C
0C
tg
h h
.
Pomocou vzdialenosti 0C urme sradnice bodu B
l
h
l C x +

= + =
tg
0
B
,
h y =
B
.

2. spsob rieenia - vektorov metda (obr. B)
Vyjadrme vektory l
1
a l
3
vzhadom na zvolen sradnicov systm.
j i l h
h
+

=
tg
1
,
C
C
C
h
C
B
l

Re
0
Im

1
A
C
z
1
z
2
z
3
z
4

3

4

2
= 0
C
C
C
h
C
B
l

x
0
y

A
C
y
B

x
B

l
1
l
2
l
3
C
C
C
h
C
B
l

x
0
y

A
C
r
B

y
B

x
B

C
C
C
h
C
B
l

x
0
y

105
i l l =
2
,
j i l
B B
y x =
3
.
Mus plati podmienka uzavretosti
[ ] [ ] 0 j i i j i l l l l = + +

= + + =

=
B B
i
i
y x l h
h
tg
3 2 1
3
1
,
resp. [ ] 0 j i = +

B B
y h x l
h
tg
.
Vektorovu rovnicu rozpeme do dvoch skalrnych rovnc:

. 0 : 0
,
tg
0
tg
: 0
B B
B B
h y y h l
l
h
x x l
h
l
yi
xi
= = =
+

= = +



3. spsob rieenia - metda komplexnch siel (obr. C)
Vyjadrme komplexn sla z
1
a z
4
vzhadom na zvolen komplexn rovinu:
komplexn slo z
1
(
1
= )

1
2
2
1
2
2
1
sin
tg
cos
tg
+

+ +

= h
h
j h
h
z ,
tg
1
h j
h
z +

=
komplexn slo z
2
(
2
= 0)

2 2 2
sin cos + = l j l z ,
2
l z =
komplexn slo z
3
(
3
= 270)

3 B 3 B 3
sin cos + = y j y z
B 3
y j z = ,
komplexn slo z
4
(
4
= 180)

4 B 4 B 4
sin cos + = x i x z
B 4
x z = ,
kde 1 = j je imaginrna jednotka.
Op mus plati podmienka uzavretosti,
[ ] [ ] 0 ] [
tg
4 3 2 1
4
1
= + + +

= + + + =

=
B B
i
i
x jy l jh
h
z z z z z .
Nsledne pre relne asti a pre komlexn asti komplexnch siel dostaneme

. 0 : 0 Im
,
tg
0
tg
: 0 Re
B B
B B
h y y h
l
h
x x l
h
= = =
+

= = +


Poloha bodu B je definovan polohovm vektorom, ktorho dve sradnice boli uren tromi
rznymi metdami. Polohov vektor bodu B potom m tvar
106
j i r h l
h
+

=
tg
B
.
Rchlos bodu B
i v

=
2
B
sin
&
h .
Zrchlenie bodu B
i a


=
4
2 2
B
sin
2 sin sin & & &
h .
Prklad 2.24

Mechanizmus na obrzku sa dostva do
pohybu otanm kuky C okolo bodu 0
1
kontantnou uhlovou rchlosou
21
.
Vypotajte:
1. vekos uhlovej rchlosti lena C,
2. vektor a vekos rchlosti bodu L, vektor
a vekos zrchlenia bodu L.
Dan: r, l, d,
v ase t = 0 s sa oba leny nachdzaj vo
vodorovnej polohe a s v kude.
1. Vpoet uhlovej rchlosti lena C.
asov zvislos uhla
21
je vyjadren zo zkladnej defincie uhlovej rchlosti
. kont
21
21
=

=
dt
d
,
odkia po separcii premennch a integrcii
t dt d dt d
t
21 21
0
21
0
21 21 21
21
= = =

.
Z pravouhlho trojuholnka je vyjadren uhol
31
, t.j. plat

) cos(
) sin(
arctg
cos
sin
S 0
AS
tg
21
21
31
21
21
2
31
t r d
t r
r d
r
+

=
+

= = .
Vektor a vekos uhlovej rchlosti
31
lena C vzhadom na rm C dostaneme derivciou

31
poda asu
k
31 31
= ,
21
21
2 2
21
2
31 31
) cos( 2
) cos(

+ +
+
= =
t rd d r
t rd r
& .
Vekos uhlovho zrchlenia lena C vzhadom na rm C
C
C

21
r

C
0
1
x
y
S
l
C

21
0
2

31
d
A
L
i
j
r
L
107
k
31 31
= , ) sin(
)] cos( [
) (
t
t rd d r
rd d r
21
2
21
2
21
2 2
3 3
31 31 31
2
2

+ +

= = = & & & .
2. Rchlos a zrchlenie bodu L je pre tento prpad mon vyjadri dvoma spsobmi:
I. spsob II. spsob
Polohov vektor bodu L vzhadom na 0
2
:
j i r
31 31 31 L
sin cos + = l l .
Bod L je bod, ktor je bodom telesa C,
konjceho rotan pohyb okolo stlej osi
rotcie prechdzajcej bodom 0
2
.
Vektor rchlosti:
. cos sin
31 31 31 31
31 L 31 L
j i
r v
+ =
= =
& &
&
l l

Vekos rchlosti:
.
, ] cos [ ] sin [
31 31 L
2
31 31
2
31 31 31 L
=
+ =
l v
l l v & &
Vektor rchlosti:
. ) cos ( ) sin (
], ) sin ) cos [(
,
31 31 31 31 31 L
31 31 31 31 L
31 L 31 31 L
j i v
j i k v
r v
+ =
+ =
=
l l
l l
Vekos rchlosti:
l v
31 31 L 31 31 L
90 sin = = r .
Vektor zrchlenia:
. ] cos sin ) ( [
] sin cos ) [(
31 31 31
2
31 31 L
31 31 31
2
31 31 L
31 L 31 L 31 L 31 L
j a
i a
a a v a
+ =
+ =
+ = =
& & &
& & &
&
l
l
y
x
y x

Vekos zrchlenia:
4
31
2
31
2
31
2
31
+ = + = l a a a
y x L L L
,
kde ,
31 31
= &
31 31
= & & .
Vektor zrchlenia:
), sin (cos
) (
), sin (cos
,
31 31 31 31
31 L 31 31 31 L
31 31 31
31 L 31 31 L
31 L 31 L 31 L
i j k
r a
j i k
r a
a a a
=
= =
+ =
= =
+ =
l
l
n
t
n t

. ] sin cos [
] cos sin [
31
2
31 31 31
31
2
31 31 31 31 L
j
i a
+
+ =
l
l

Vekos zrchlenia:
4
31
2
31
2
31 L
2
31 L 31 L
+ = + = l a a a
n t


108
Prklad 2.25
Mechanizmus zobrazen na obrzku je uvdzan do pohybu rotujcim lenom C, ktorho
uhol pootoenia je definovan funkciou asu
2
21
kt = [rad]. V ase t = 0 s bol mechanizmus
v kude a len C sa nachdzal na osi x. Vyjadrite:
1. rovnice pohybu bodu B lena C vzhadom na len C,
2. kinematick veliiny:
B31 B31 B31 B31 B31 B31
, , , , ,
n t
a a a v a v ,
3. polomer krivosti drhy bodu B,
4. rovnicu trajektrie bodu B.
Dan: R [m]; h [m]; l [m]; e [m]; k = 4,0 rad.s
2


Obr. A

Obr. B

len C mechanizmu kon rotan pohyb okolo osi prechdzajcej bodom 0 a kolmej na
rovinu. Bod A sa pohybuje po krunici s polomerom R a je bodom lena C aj C. len C
vzhadom na vzbu s lenom C kon iba posuvn pohyb. Kee pri posuvnom pohybe sa
vetky body telesa pohybuj po zhodnch trajektrich, vetky body lena C sa bud
pohybova po kruniciach s polomerom R. len C kon posuvn pohyb vzhadom na vzbu
v bode D. Vetky jeho body lena C sa pohybuj po priamke v horizontlnom smere.
1. V zvolenom sradnicovom systme je zakreslen polohov vektor bodu B. Pomocou strn
v pravouhlom trojuholnku OCA s vyjadren sradnice bodu B lena C vzhadom na len C

. sin
, cos
B31
B31
21
21
=
=
R l y
R x
(a)
2. Vyjadrenie kinematickch velin:
B31 B31 B31 B31 B31 B31
, , , , ,
n t
a a a v a v .
Pre bod B s vyjadren nasledovn kinematick veliiny:
polohov vektor
j i j i r ) 4 sin ( ) 4 cos ( ) sin ( ) cos (
2 2
21 21 B31
l t R t R R l R + = = , (b)
vektor rchlosti
j i
r
v
2 2
4 8 4 8 t Rt t Rt
dt
d
cos sin
B31
B31
+ = = , (c)
vekos vektora rchlosti
Rt t Rt t Rt v v v
y x
8 4 8 4 8
2 2 2 2 2 2
= + = + = ) cos ( ) sin (
B31 B31 B31
[m.s
1
], (d)
vektor zrchlenia

, ] cos sin ) ( [
] sin cos ) ( [
B31
B31
j
i
v
a
2 2 2
2 2 2
4 8 4 8
4 8 4 8
t R t R t
t R t R t
dt
d
+ +
+ = =
(e)
C
C
R
h

B
C
0
l
C
A
e

x
y

21

D
C
C
R
x
B
B
C
0
l
C
A
e

x
y

21
C
r
B
y
B
D
109
vekos vektora zrchlenia

,
, ] cos sin ) ( [ ] sin cos ) ( [
B31
B31
4
2 2 2 2 2 2 2 2
64 1 8
4 8 4 8 4 8 4 8
t R a
t R t R t t R t R t a
+ =
+ + =
(f)
vekos tangencilnej zloky zrchlenia
R v
dt
dv
a
t
8 = = =
BB31
B31
B31
& [m.s
2
], (g)
vekos normlovej zloky zrchlenia

2 2 4 2 2
B31
2
B31 B31
64 64 ) 64 1 ( 64 Rt R t R a a a
t n
= + = = [m.s
2
]. (h)

3. Polomer krivosti drhy bodu B mono vyjadri na zklade vzahu
R
Rt
t R
a
v
R
R
v
a
n
n
= = = =
2
2 2
B31
2
B31
0 B
0
2
B31
B31
64
64
[m.s
2
]. (i)
4. Rovnica trajektrie je vyjadren z parametrickch rovnc pohybu (a)

21 B
cos = R x
R
x
B
21
cos = ,
l R y =
21 B
sin
R
l y +
=
B
21
sin .
Po dosaden do vzahu 1 sin cos
21
2
21
2
= + , dostaneme
1
2
B
2
B
=

+
+

R
l y
R
x

2 2
B
2
) ( R l y x
B
= + + ,
o je rovnica krunice s polomerom R a so stredom v bode [0, l].
Prklad 2.26
Mechanizmus schematicky zobrazen na obrzku je uvdzan do pohybu lenom C,
ktorho uhol pootoenia je nasledovnou funkciou asu
2
21
4t = [rad]. V ase t = 0 s bol
mechanizmus v kude a len C a nachdzal sa na osi x. Vyjadrite:
1. rovnice pohybu bodu M, ktor sa nachdza na lene C,
2. kinematick veliiny:
M41 M41 M41 M41 M41 M41
, , , , ,
n t
a a a v a v .
Dan: R [m]; h [m]; l [m]; e [m]; k = 4,0 rad.s
2

Vsledok:
1 -
21
= cos
M41
R h x [m], e y =
M41
[m]
2 - i v
2
M41
4 sin 8 t Rt =
2
M41
4 sin 8 t Rt v = [m.s
1
]
i a ] 4 sin 8 4 cos ) 8 [(
2 2 2
M41
t R t R t + =
2 2 2
M41
4 sin 8 4 cos ) 8 ( t R t R t a + = [m.s
2
]
2 2 2
M41
4 sin 8 4 cos ) 8 ( t R t R t a
t
+ = [m.s
2
]
0
2
M41
2
M41 M41
= =
t n
a a a [m.s
2
]

C
C
R
h

M
C
0
l
C
A
e

x
y

21

D
110
Prklad 2.27

Vlonk C je uvdzan do pohybu hydraulickm
valcom C poda danej schmy. Vekos rchlosti piesta
v (bod A) je kontantn.
Vypotajte:
1. vektor uhlovej rchlosti a vektor uhlovho zrchlenia
vlonka,
2. vekos rchlosti a vekos zrchlenia bodu L.
Dan: l, d, v
Vsledok: k
2 2 2
t v d
v d
+
= k
2 2 2 2
3
) (
2
t v d
t v d
+

=

2 2 2
t v d
v d
l v
L
+
=
4
2 2 2
2
2 2 2 2
3
) (
2

+
+

=
t v d
v d
t v d
t v d
l a
L


Prklad 2.28

Rovinn mechanizmus zdvhacieho zariadenia
schematicky znzornen na obrzku sa uvdza do
pohybu otanm kruhovej vaky C s excentricitou
r. Pohyb je dan asovou zvislosou definovanou
pre uhol
21
. Vypotaje vekos uhlovej rchlosti

51
a vekos uhlovho zrchlenia
51
lena C. Na
zaiatku pohybu bol mechanizmus v kude. seka
AB a taktie len C sa v ase t = 0 s nachdzali vo
vodorovnch polohch.
Dan:
21
=
21
(t); l
1
; l
2
; r; R; h
2
; h
1
= h
2
+ R
Vsledok:

51 51 21
21
2 2 2
1
21 1
51 51
=
+

= = & & & & ;
sin
cos
r l
r l


Prklad 2.29

Urte vekos rchlosti a zrchlenia zdvhadla C, ktor je
zdvhan excentricky uloenou kruhovou vakou C. Kuka vaky
(0S) sa ota poda zkona ) (t
21 21
= [rad]. Na zaiatku pohybu
(t = 0 s) je kuka vaky vo vodorovnej polohe a je v kude.
Dan: e, r, d
Vsledok:
21 21
= cos e v &
21 21 21
2
21
+ = cos sin e e a & & &
C
e
S
r
C
C

x
d

0
y

A

h

C
C

21
r

R
l
2
h
2

B
C
C
A
D
h
1

C
C
C
l
1
x
y
D
0
B
0

5
=?
l
x
d
A
0
A

0


L

v
y
C

C

C


111
Prklad 2.30
Kladka C spojen s telesom C apom je zasvan kontantnou rchlosou v
0
pod teleso C,
ktor kon rotan pohyb. Vypotajte:
1. vekos uhlovej rchlosti
41
a vekos uhlovho zrchlenia
41
,
2. vekos rchlosti v
A41
a uhlovho zrchlenia a
A41
pohybu bodu A telesa C vzhadom na
teleso C.
Zaiaton podmienky: t = 0 s ; B B
0
.
Dan: l, h, r, v
0
= kont

Vsledok:
2 2
0 0 0 0
0
41
) ( ) ( r t v h t v h
rv

=
2
0
2 2
0 0
2
0 0
2 2
0 0
41
2
3
] ) [( ) (
) ( 2
rv
r t v h t v h
r t v h


=
l v
41 41 A
= ; l a
t 41 41 A
= ; l a
n
2
41 41 A
=
4
41
2
41
2
41
2
41 41
+ = + = l a a a
n t A A A

2 2
0 0
2 2
0 0
2 2
0 0 0 0
2
0
41
3 4
r t v h
r t v h
r t v h t v h
lrv
a



=
) (
) (
] ) )[( (
A



























r
l
v
0
h
0
A
s

B
0
B

C
C
C
C
C
112
3 DYNAMIKA


3.1 DYNAMIKA HMOTNHO BODU
Pohybov rovnica hmotnho bodu
Newtonov spsob zostavovania pohybovch rovnc (metda zrchujcich sl)

=
= =
n
i
i
m
1
F F a , (3.1)
kde m je hmotnos a a je zrchlenie hmotnho bodu, F
i
- primrne-akn sily a sily od
inku vzieb - reakcie, zskan metdou uvoovania (i = 1n).
Vyjadrenie pohybovch rovnc v kartezinskom sradnicom systme
. , ,
1 = 1 = 1 =

= = = = = =
n
i
zi z
n
i
yi y
n
i
xi x
F z m ma F y m ma F x m ma & & & & & & (3.2)
DAlembertov spsob zostavovania pohybovch rovnc (fiktvna dynamick rovnovha)
0 F F = +

=
n
i
i
1
D
, (3.3)
kde a F m =
D
je dAlembertova zotrvan sila.
Metodika rieenia loh dynamiky pomocou pohybovch rovnc
1. Pohybujci sa hmotn objekt uvoni vo veobecnom asovom okamihu.
2. Definova inky vonkajch zaaujcich sl a inky vzbovch sl.
3. Zostavi zkladn pohybov rovnicu vo vektorovom tvare (prpadne v skalrnom tvare).
4. Vektorov pohybov rovnicu rozpsa do skalrnych zlokovch rovnc vo vhodnom
sradnicovom systme.
5. Vyjadri vzahy medzi kinematickmi veliinami, akciami a reakciami a vzahy pre
pasvne odpory.
6. Rieenie sstavy skalrnych pohybovch rovnc a vpoet hadanch silovch, resp.
kinematickch velin.
Zkladn vety dynamiky hmotnho bodu
Veta o zmene hybnosti
F
H
=
t d
d
, resp.
F
t
t I F H H = =

0
0
d , resp.
F
m m I v v =
0
. (3.4)
Veta o zmene momentu hybnosti
M
L
=
t d
d
, resp.

=
t
t
0
d d
0
M L
L
L
, resp.
M
I L L =
0
. (3.5)
Veta o zmene kinetickej energie

=
r
r
r F
0
d
2 2
2
0
2
mv mv
, resp. A E E E
k k k
= =
0
. (3.6)
Veta o zachovan mechanickej energie
.
0 0
kont E E E E
p k p k
= + = + (3.7)
113
Veliiny uveden vo vzahoch: H je vektor hybnosti, L je vektor momentu hybnosti, I
F
je
vektor impulzu sily, I
M
je vektor impulzu momentu, E
p
je potencilna energia, E
k
je
kinetick energia, A je prca aknch sl.

Prklad 3.1
Po povrchu dokonale hladkej gule s polomerom R sa pohybuje hmotn bod s hmotnosou m.
Zaiaton rchlos bodu je v
0
. Vypotajte uhol, pri ktorom sa bod odpta od povrchu gule.

Obr. A


Obr. B


Obr. C

Obr. D
1. spsob rieenia
Rieenie pomocou pohybovch rovnc zostavench metdou zrchujcich sl pre hmotn
bod (HB) vo veobecnej polohe v smere dotynice t a v smere normly n (obr. C), t.j.
= sin G ma
t
, (a)
+ = cos G F ma
n n
, (b)
kde F
n
je dotykov sila medzi HB a povrchom guovej plochy v smere normly.
Okamih odptania sa HB od guovho povrchu nastva v prpade 0
n
F . Pohybov
rovnica (b) v smere normly m potom tvar
= cos G ma
n
, resp. = cos g a
n
. (c)
Pre normlov zloku zrchlenia pri pohybe po krunici plat R v a
n
) (
2
= . Na zklade
rovnice (c) je potom minimlna vekos obvodovej rchlosti na odptanie sa HB od
povrchu guove plochy vyjadren nasledovne
m

R


r
0
F
n
G


dr

r
v
0
v()


r

r

v
0
R

v()

m

R


n

t

a
t
()

a
n
()

F
n
G


v
0
m

R


0

114

m m
gR v = cos ) ( . (d)
Drhu s HB pohybujceho sa po krunici je mon vyjadri pomocou vzahu = R s .
Z rovnice (a) vyplva
=

= = = = = sin g
Rd
dv
v
ds
dv
v
ds
dv
dt
ds
ds
ds
dt
dv
dt
dv
a
t
, (e)
resp. = d gR vdv ) sin ( .
Integrciou poslednej rovnice v prslunch hraniciach je vypotan rchlos HB pri jeho
pohybe po guovom povrchu v zvislosti od uhla



=
0
) (
) sin (
0
d gR vdv
v
v
,
resp. ) cos 1 ( 2 ) (
2
0
2
+ = gR v v . (f)
Uhol
m
definujci polohu HB pri odptan od guovho povrchu je vypotan zo
vzahov (d) a (f), t.j.
) cos 1 ( 2 cos
2
0 m m
gR v gR + = ,
odkia
gR
gR v
m
3
2
arccos
2
0
+
= .
2. spsob rieenia
Rieenie pomocou vety o zmene kinetickej energie (obr. D). Pre tento prpad rieenia plat


= + =
k
k
E
E
k n
dE d d
0 0 0
) (
r
r
r
r
r G F r F , (g)
kde r je polohov vektor HB, E
k
je kinetick energia v mieste definovanom polohovm
vektorom r, E
k0
je kinetick energia na zaiatku pohybu v polohe r
0
. Sila F
n
je poas
pohybu HB stle kolm na prrastok polohovho vektora dr, z oho vyplva, e skalrny
sin F
n
.
dr = 0. Integrciou vzahu (g)

0 0 k k
E E = = r G r r G ) ( ,
resp.
2
0
2
2
1
) (
2
1
) cos ( cos mv mv R R G r G = = ,
odkia pre rchlos pohybu bodu po krunici plat
) cos 1 ( 2 ) (
2
0
2
+ = gR v v . (h)
Na zklade vzahov (h) a (d) je uhol odptania sa hmotnho bodu od povrchu gule

gR
gR v
m
3
2
arccos
2
0
+
= .
3. spsob rieenia
Rieenie pomocou zkona zachovania mechanickej energie, t.j. plat
115
. k
0 0
ont E E E E
p k p k
= + = + (i)
V prpade nulovej potencilnej hladiny umiestnenej na rovni stredu guovej plochy (obr.
B) mono rovnicu (i) vyjadri v tvare
, ) cos 1 ( ) (
2
1
2
1
2 2
0
kont mgR mv mgR mv = + = +
odkia pre rchlos pohybu bodu po krunici plat
) cos 1 ( 2 ) (
2
0
2
+ = gR v v . (j)
Pouitm vzahov (j) a (d) je uhol vyjadren v tvare

gR
gR v
m
3
2
arccos
2
0
+
= .
Prklad 3.2
Hmotn bod s hmotnosou m = 0,5 kg je pripojen k nehmotnej vertiklnej tyi s dkou R = 1,0 m.
Pohyb je hmotnmu bodu udelen zaiatonou rchlosou v
0
= 0,25 ms
-1
. Vypotajte:
a - vekos sily v nehmotnej tyi v zvislosti od uhla natoenia ,
b - rchlos hmotnho bodu v prpade horizontlnej polohy tye.

Poas pohybu sa hmotn bod m pohybuje po krunici s polomerom R. Jeho pohybov rovnice
s zostaven pomocou metdy zrchujcich sl v sradnicovom systme normla-dotynica:
, sin = G ma
t
, sin = g a
t
(a)
. cos
n n
F G ma + = . cos
m
F
g a
n
n
+ = (b)
Pre tangencilnu a normlov zloku zrchlenia pri pohybe bodu po krunici plat

= = =
Rd
dv
v
ds
dv
dt
ds
ds
ds
dt
dv
a
t
,
R
v
a
n
2
= . (c)
Po dosaden pre a
t
do rovnice (a) a po separcii premennch s integrovan obidve strany
zskanej rovnice v prslunch hraniciach



=
0
) (
sin
0
d gR vdv
v
v
, (d)
R

F
n
G

m
t
n
v
0

a
n
a
t
v()
s
R

m
v
0
116
odkia pre rchlos hmotnho bodu vo veobecnej polohe plat
) cos 1 ( 2 ) (
2
0
+ = gR v v . (e)
Pre prpad horizontlnej polohy tye (t.j. 2 = ) je vekos rchlosti

1 2 2
0
ms 436 , 4 0 , 1 . 81 , 9 . 2 ) 25 , 0 ( ) 2 ) (

= + = + = gR v v .
Vekos sily v tyi je vypotan z pohybovej
rovnice (b) po dosaden za a
n
zo vzahu (c)

= cos
) (
2
mg
R
v
m F
n
,
resp. po dosaden za v() a po prave je vekos
sily v tyi v zvislosti od uhla vyjadren
pomocou vzahu

+ = ) cos 3 2 (
2
0
g
R
v
m F
n
.
Sila v tyi sa poas rotcie men z tlakovej na
ahov, z oho vyplva zmena znamienka sily
v uvedenom grafe.
0 30 60 90
-6
0
6
12
s
i
l
a

v

t
y

i



[
N
]
uhol natoenia []


Prklad 3.3
Z vky h sa po dokonale hladkej naklonenej rovine pohybuje hmotn bod m. Po
zomyknut z naklonenej roviny sa hmotn bod dostane na drsn vodorovn plochu. Medzi
vodorovnou plochou a hmotnm bodom je trenie definovan faktorom mykovho trenia f.
Vypotajte, ak drhu vykon hmotn bod po vodorovnej ploche, a sa zastav.


Pre pohyb HB po naklonej rovine plat zkon zachovania mechanickej energie v zaiatonej
polohe a v polohe, ke sa HB dostane na vodorovn plochu
kont.,
0 0
= + = +
p k p k
E E E E resp. 0
2
1
0
2
1
+ = + mgh mv . (a)
Rchlos HB na zaiatku pohybu po horizontlnej rovine
gh v 2
1
= . (b)
Pomocou dAlembertovej metdy je pre pohyb HB po naklonenej rovine zostaven pohybov
rovnica
m
h
s = ?
f
m
G F
N
F
T
a

F
D
i

j

117
0 F F = +

=
n
i
i D
1
, resp. 0 G F F F = + + +
T N D
. (c)
kde a F m
D
= je dAlembertova zotrvan sila, ktorej vekos je ma F
D
= .
Skalrnym prensobenm rovnice (c) jednotkovmi vektormi i a j dostaneme
0 =
T D
F F , resp. 0 = G F
N
. (d)
Po dosaden za dAlembertovu zotrvan silu ma F
D
= a za treciu silu
N T
fF F = dostaneme
fmg fF F ma
N T
= = = , resp. fg a = , (e)
odkia pre zrchlenie plat
fg
ds
dv
v
ds
dv
dt
ds
ds
ds
dt
dv
dt
dv
a = = = = = . (f)
Po separcii premennch a integrcii dostaneme


=
s
v
fgds vdv
0
0
1
, resp. fgs
v
=
2
2
1
. (g)
Hmotn bod na drsnej vodorovnej ploche zastav po prejden drhy f h s = .

Prklad 3.4
Z vky h pad hmotn bod s hmotnosou m na pruinu, ktor m kontantn tuhos k.
Vypotajte, ak vek je stlaenie pruiny.



Pouitm zkona o zmene kinetickej energie medzi polohami B
0
a B
k
dostaneme

0 k k
E E A + = , (a)
- prca sily v pruine (F
k
= kx)
2
0 0
2
1
k
x
k F
kx kxdx d A
k
k
= = =

r
r F ,
- prca gravitanej sily ) ( x h G A
G G
+ = = r G ,
- kinetick energia na zaiatku 0
2
1
2
0 0
= = mv E
k
,
- kinetick energia na konci 0
2
1
2
= =
k k
mv E .
B
k
B
0
h
x
k
k
l
p0
m
v
k
= 0
v
0
= 0
x

B
0
h
m
F
k
G

r
G

r
B
k
x
k
k
l
p0
F
k
x

dx


x

F
k
x

kx
x
k
B
0
m
h
k
l
p0
118
Veta o zmene kinetickej energie m po dosaden tvar
0 0
2
1
) (
2
= +
k k
kx x h mg .
Rovnica popisujca stlaenie pruiny m tvar
0 2 2
2
= h
k
mg
x
k
mg
x
k k
, (b)
odkia deformcia pruiny

+ + =
mg
hk
k
mg
x
k
2
1 1 . (c)
Pouitm
k
mg
x
st
= - deformcia pruiny pri statickom zaaen

+ + =
st
st k
x
h
x x
2
1 1 . (d)
Pre prpad 0 h , je
st k
x x 2 = . Z tohto vzahu vyplva, e pri nhlom pripojen hmoty m
k pruine je vekos dynamickej deformcie dvojnsobok statickej deformcie vznikajcej
pomalm psobenm gravitanej sily.

Prklad 3.5
Ku dnu oceovej rrky dky l
0
je pripojen pruina tuhosti k stlaen na dku l
p0
, na konci
ktorej je umiestnen hmotn bod m. Nezaaen pruina m dku l
p
. Rrka je od horizontu
sklonen pod uhlom . Pruina je v ase t = 0 uvonen. Hmotn bod sa pohybuje najskr vo
vntri rrky a potom je v bode C vymrten a let vzduchom uvaovanom ako bezodporov
prostredie. Medzi hmotnm bodom a rrkou je uvaovan faktor mykovho trenia f .
Vypotajte, do akej vzdialenosti od bodu O dolet hmotn bod.


Obr. A
m


l
p
x
1
l
p0
F
k

G
F
T
F
N
O

A

B

a
1
119

Obr. B

Obr. C


Pohyb hmotnho bodu je potrebn riei v troch etapch:
Prv etapa (obr. A) pohybu hmotnho bodu je uvaovan v seku od zaiatonho
stlaenia pruiny l
p
a po polohu definovan maximlnou dkou nezaaenej pruiny l
p0
(medzi bodmi A a B). Etapu charakterizuje viazan pohyb hmotnho bodu v rrke, priom
na hmotn bod poas jeho pohybu psob gravitan sila G, sila od stlaenej pruiny F
k
,
normlov sila F
N
a trecia sila F
T
. Pohybov rovnica v tomto seku m tvar
= sin
1
G F F ma
T k
. (a)
Druh etapa (obr. B) pre pohyb bodu nastva v okamihu, ke pruina nadobudne svoju
dku l
p0
v nezaaenom stave (nastva strata kontaktu s hmotnm bodom) a po okamih,
ke opa rrku (sek medzi bodmi B a C). Op je to viazan pohyb hmotnho bodu v
rrke ako v prvej etape, priom na hmotn bod poas jeho pohybu psob gravitan sila
G, normlov sila F
N
, trecia sila F
T
, ale prestala psobi sila od stlaenej pruiny F
k
.
Pohybov rovnica v tomto seku m tvar
= sin
2
G F ma
T
. (b)
Pre obidve etapy mono z rovnice rovnovhy v smere kolmo na pohyb hmotnho bodu
vyjadri vekos normlovej sily
= = cos cos mg G F
N
,
resp. pre treciu silu plat
= = cos fmg fF F
N T
.
Tretia etapa (obr. C) nastva v okamihu, ke hmotn bod opust stie rrky (bod C) a let
vone vzduchom a po dopad v bode D. Pohybov rovnice s potom v tvare
0
3
=
x
ma , 0
3
=
x
a (c)
mg G ma
y
= =
3
g a
y
=
3
. (d)
Zrchlenie hmotnho bodu v prvej etape v intervale
0 1
,
p p
l l x po dosaden do rovnice (a)
za treciu, normlov a gravitan silu je vyjadren pomocou rovnice
) cos (sin ) (
1 1
+ = f g l x
m
k
a
p
. (e)
Z uvedenej rovnice je mon vyjadri vzah pre zvislos rchlosti od drhy bodu

1
1
1
1
1 1
1
1 1 1
1
dx
dv
v
dx
dv
dt
dx
dx
dx
dt
dv
dt
dv
a = = = = .
m
l
0
G

v
C
O
C
D
a
x3
a
y3
y

x

l
0
cos x
D
= ?
l
0
sin
m


l
0
x
2
l
p
G
F
T
F
N
O

B

C

a
2
120
Po separcii premennch v rovnici (e) a integrovan v prslunch hraniciach

1 1
0
1 1
0
] [ ) cos (sin ) ( dx f g l x
m
k
dv v
p
p
B
l
l
p
v

+ = ,
odkia pre rchlos hmotnho bodu v polohe definovanej v bode B plat
) )( cos (sin 2 ) (
0
2
0 p p p p B
l l f g l l
m
k
v + = . (f)
Pre zrchlenie hmotnho bodu v druhej etape, t.j. v intervale
0 0 2
, l l x
p
plat
) cos (sin
2
+ = f g a
a po prave podobnej ako v prvej etape

2 2 2
0
0
) cos (sin dx f g dv v
l
l
v
v
p
C
B

+ = ,
je rchlos hmotnho bodu v polohe C vyjadren v tvare
) )( cos (sin 2 ) (
0
2
0 p p p C
l l f g l l
m
k
v + = . (g)
Pohybov rovnice v tretej etape s rieen nasledujcim spsobom:
, 0
3
3
= =
dt
dv
a
x
x
, 0
3
=
x
dv , 0
3
cos
3
=


x
C
v
v
x
dv
g
dt
dv
a
y
y
= =
3
3
gdt dv
y
=
3
,
0 sin
3
3

=

t
v
v
y
gdt dv
y
C

odkia po integrcii pre rchlosti hmotnho bodu v tretej etape pohybu v smere x a y

dt
dx
v v
C x
3
3
cos = = ,

dt
dy
gt v v
C y
3
3
sin = = .
Po separcii a integrcii s vyjadren sradnice polohy bodu poas letu
dt v dx
t
C
x

=
0 0
3
cos
3
= cos
3
t v x
C
, (h)
dt gt v dy
t
C
y

=
0 0
3
) sin (
3

2
sin
2
3
t
g t v y
C
= . (i)
Po elimincii asu t z oboch predchdzajcich rovnc je zskan vzah popisujci drhu bodu

=
2 2
2
3
3 3
2 cos
tg
C
v
gx
x y . (j)
Poloha bodu D, t.j. miesta doletu je vzhadom na sradnicov systm x, y
] sin ; [ D
0
l x
D
.
121
Po dosaden do (j)

=
2 2
2
0
2 cos
tg sin
C
D
D
v
gx
x l ,
odkia
) 2 sin(
2
sin
2
1 1
2
2
0

+ + =
g
v
v
gl
x
C
C
D
,
resp. vzdialenos bodov O a D je

D OD
x l l + = cos
0
.
Prklad 3.6
Na kbovo upevnenej nehmotnej tyi dky R = 2 m je pripojen hmotn bod s hmotnosou
m = 3 kg, ktor je pomocou lana vychlen od vertiklnej osi o uhol = 30. Na zaiatku je sstava
v pokoji. V ase t = 0 s je lano preseknut. Vypotajte silu N v tyi pre nasledovn prpady:
a - poloha pred preseknutm lana (N
0
),
b - poloha ihne po preseknut lana (N
1
),
c - v najniom bode drhy hmotnho bodu (pre = 0) (N
2
).


Obr. A

Obr. B

Obr. C
a - sila v tyi v polohe pred preseknutm
Tento prpad (obr. B) je rovnovny, priom na hmotn bod psobia tri sily - sila v lane S,
sila v tyi N
0
a gravitan sila G. Rovnovne rovnice s:

=
i
xi
F 0 0 sin
0
= + N S , (a)

=
i
yi
F 0 0 cos
0
= + N G . (b)
Z tchto rovnc s vypotan neznme sily:
- sila v lane N mg N S 99 16
0
, tg sin = = = ,
- sila v tyi N
mg
N 98 , 33
cos
0
=

= .
b - sila v tyi v polohe okamite po preseknut


G
a
t

a
n

N
1

R
R

N
0
G
S
122
V tomto prpade nastva limitn stav medzi rovnovhou a pohybom. Sila v lane prestala
psobi a pohyb ete nenastal, t.j. rchlos pohybu je nulov. Na hmotn bod psobia dve sily:
sila v tyi N
1
a gravitan sila G. Pohybov rovnice s:
= = sin sin mg G ma
t
= sin g a
t
, (c)

1 1
cos cos N mg N G ma
n
+ = + = . (d)
Vekos normlovej zloky zrchlenia pre pohyb bodu po krunici

R
v
a
n
2
= . (e)
Vzhadom na to, e v okamihu po preseknut lana je rchlos nulov, je aj normlov zloka
zrchlenia nulov, t.j. 0 =
n
a . Rovnica (d) po dosaden za normlov zrchlenia

1
cos 0 N mg + = = cos
1
mg N .
Sila v tyi v okamihu preseknutia lana:
N mg N 49 , 25 cos
1
= = .
c - sila v tyi v polohe v najniom bode drhy hmotnho bodu (obr.C, pre = 0)
Na hmotn bod v tomto prpade psobia dve sily - sila v tyi N
2
a gravitan sila G.
Pohybov rovnice s:
) sin( = mg ma
t
) sin( = g a
t
, (f)

2
) cos( N mg ma
n
+ =
2
2
) cos( N mg
R
v
m + = . (g)
Pre tangencilne zrchlenie plat

= = = = =
Rd
dv
v
ds
dv
v
ds
dv
dt
ds
ds
ds
dt
dv
dt
dv
a
t
, (e)
kde je pouit vyjadrenie = R s - dka oblka.
Po dosaden do (f) dostaneme
) sin( =

g
Rd
dv
v


=
0
0
) sin( d gR vdv
v
,
odkia
) cos 1 (
2
2
= gR
v
) cos 1 ( 2 = gR v . (f)
Po dosaden z (f) do rovnice (g) dostaneme pre silu v tyi
N mg N 32 , 37 ) cos 2 3 (
2
= = .




123
Prklad 3.7

Akou minimlnou nbehovou rychlosou v
min
sa mus
pohybova hmotn bod pred vstupom (bod A) na vertiklne
umiestnen kruhov drhu s polomerom r = 2,8 m, aby sa
dostal do hornej polohy drhy (bod B)? Hmotn bod m
hmotnos m = 70 kg. Pasvne odpory zanedbajte.
Vsledok:
v
min
= 11,72 m.s
-1


Prklad 3.8

Na blok s hmotnosou 90 kg, ktor le na vodorovnej
podloke, psob sla F = 1,5 kN poda obrzku. Faktor
mykovho trenia medzi blokom a podlokou je f = 0,25.
Vypotajte vekos zrchlenia bloku.
Vsledok:
a = 9,035 ms
-2


Prklad 3.9

Na kruhovom disku rotujcom okolo zvislej osi le
vo vzdialenosti r
0
= 8 cm hmotn bod s hmotnosou
m = 1,5 kg. Faktor mykovho trenia medzi hmotnm
bodom a diskom je f = 0,5. V ase t = 0 s sa disk
rozta z kudovej polohy kontantnm uhlovm
zrychlenm = 0,5 rad/s
2
. Vypotajte as t, pri
ktorom sa zane hmotn bod mka po povrchu
disku.
Vsledok:
t = 15,66 s.

Prklad 3.10

Hmotn bod o hmotnosti m sa pred nbehom na kruhov
drhu pohybuje zaiatonou rchlosou v
0
. Vypotajte
rchlos v
0
tak, aby hmotn bod po odptan sa pod uhlom
dopadol do stredu krunice. Pasvne odpory s zanedban.
Vsledok:
pre
3
1
cos =
3
2
1
rg v = , ) 3 2 (
0
= rg v .

m
F

f
v
0
v
1
m


r

r
v
min
B
A
124
Prklad 3. 11
Hmotn bod je vymrten stlaenou pruinou. Pruina m tuhos k a je stlaen o dku l.
Pri rieen uvaujte pasvne odpory.
Vsledok:
g
v
h
D
2
2
=
1
2
2glf l
m
k
v
B
= ,
1 0
2
2 f gl v v
B C
=
2
2
2
2
2
2
2
2
2 2
4 1
2 3
2
4 1
1
2
2
f
f
gRf e
f
f
gR v v
f
C D
+
+

=



Prklad 3. 12

Vypotajte, z akej minimlnej vky
h m by spusten hmotn bod s
hmotnosou m, aby sa dostal do
polohy definovanej bodom A. Pasvne
odpory zanedbajte
Vsledok:
r h
2
5
min
=

Prklad 3.13

Mal guka o hmotnosti m = 0,5 kg je upevnen na
kruhov obru s polomerom R = 0,15 m, ktor rotuje okolo
zvislej osi kontantnou uhlovou rchlosou = 6 rad/s.
Vypotajte:
a) uhol , ktorm je uren poloha guliky pri rotcii,
b) silu, ktorou psob obru na guku.
Vsledok:
o
53 , 65 =
84 , 11 = F N








A B C
D
E
m
k
l

l
0
f
1 f
2
R
h

R


h
r
m
A
125
Prklad 3.14

Guka o hmotnosti m = 3 kg sa ke po rme,
ktor m tvar paraboly
2
8
2
x
y = .
Vekos rchlosti pohybu guky, ke vzdialenos
l = 2 m, je v = 5 m/s a tangencilna zloka
zrchlenia je a = 3 m/s
2
.
Vypotajte:
a - vekos tangencilnej zloky sily,
b - vekos normlovej zloky sily,
ktorou psob rm na guku.
Vsledok:
32 , 17 =
t
F N
46 , 6 =
n
F N

3.2 DYNAMIKA SSTAVY HMOTNCH BODOV
Kinematick veliiny i-teho HB
polohov vektor

iT T i
r r r + = , (3.8)
vektor rchlosti

iT T i
v v v + = , (3.9)
vektor zrchlenia

iT T i
a a a + = . (3.10)
Pohybov rovnice SHB
posuvn pohyb SHB

=
=
n
i
E
i T
m
1
F a , (3.11)
rotan pohyb SHB

=
=
n
i
E
i iT T
I
1
F r . (3.12)
Hybnos SHB

T
n
i
iT i
m m v v H = =

=1
. (3.13)
h
0
l
0
l

m

y

x

126
Moment hybnosti SHB


+ = =
n
i=
Ti i iT T T
n
i=
i i i
m m m
1 1
v r v r v r L . (3.14)
Kinetick energia SHB

2
T
2
T
1
2 2
T
2
1
2
1
2
1
2
1
v v v I m m m E
n
i=
iT i k
+ = + =

, (3.15)
kde
T
r - polohov vektor aiska,

T
v - vektor rchlosti aiska,

T
a - vektor zrchlenia aiska,
- uhlov zrchlenie SHB vzhadom na aisko,

iT
r - polohov vektor psobiska i-tej sily vzhadom na aisko,

iT iT
r v = - vektor rchlosti i-teho hmotnho bodu vzhadom na aisko,

iT iT iT
v r a + = - vektor zrchlenia i-teho hmotnho bodu vzhadom na aisko,

E
i
F - i-ta vonkajia akn sila,

=
n
i=
iT i T
m I
1
2
r - moment zotrvanosti SHB.
Prklad 3.15
Bremen o hmotnosti m
1
a m
2
s vzjomne spojen dokonale ohybnm lanom, ktor je veden
cez kladku, ktorej polomer a hmotnos je mon zanedba. Faktor mykovho trenia medzi
bremenom m
1
a naklonenou rovinou ( = 30) je f = 0,15; hmotnos bremena je m
1
= 25 kg.
Na zaiatku v ase t
0
= 0 s je sstava v pokoji v naznaenej polohe (h = 3 m). Po uvonen
sstavy naraz bremeno m
2
na vodorovn povrch za as t = 3 s. Vypotajte zrchlenie a
2
,
hmotnos bremena m
2
a silu v lane poas pohybu.



loha bude rieen pomocou pohybovch rovnc, ktor s pre jednotliv hmotn body m
1
a
m
2
zostaven pomocou metdy zrchujcich sl:
- pohybov rovnica pre m
1
= sin
1 1 1
G F F a m
T
, (a)
- pohybov rovnica pre m
2
F G a m =
2 2 2
. (b)
Pre hmotn bod m
1
zrove plat rovnovna rovnica v smere kolmom na naklonen rovinu
m
1
m
2


h

f

m
1
m
2


h

F

F
T
F
N
G
1
a
1
F

G
2
a
2
127
0 cos
1
= G F
N
= cos
1
g m F
N
. (c)
Treciu silu medzi hmotnm bodom m
1
a naklonenou rovinou mono vyjadri pomocou
= = cos
1
g fm fF F
N T
. (d)
Pre zrchlenia oboch hmotnch bodov plat m
1
a m
2

a a a = =
2 1
. (e)
Po dosaden rovnc (c)(e) do (a)(b) s pohybov rovnce v tvare

.
), sin cos (
2 2
1 1
F g m a m
f g m F a m
=
+ =
(f)
Eliminciou sily v lane F je po prave vekos zrchlenia pohybu bodov

2 1
1 2
2 1
) sin cos (
m m
f g m g m
a a a
+
+
= = = ,
ktor je kontantn.
Pohyb bodu m
2
na drhe h je charakterizovan nasledovne:

dt
dv
a =

=
t v
adt dv
0 0
at v = ,

dt
ds
v =

= =
t t h
atdt vdt ds
0 0 0

2
2
1
at h =
2
2
t
h
a = .
Pre hmotnos hmotnho bodu m
2
po vyjadren z (f) dostaneme

a g
f g a
m m

+ +
=
) sin cos (
1 2
,
a pre silu v lane
) (
2
a g m F = .
Po dosaden s hodnoty poadovanch velin:

N. 18 , 171
kg, 723 , 18
, ms 667 , 0
2
-2
2
=
=
=
F
m
a












128
Prklad 3.16

Do balistickho kyvadla pozostvajceho
z nehmotnho tuhho zvesu dky l a
zbernho lapaa s hmotnosou m
2
naraz
projektil s hmotnosou m
2
vystrelen v
horizontlnom smere rchlosou v
1
.
Projektil po nraze zostane v zbernom
lapai. Balistick kyvadlo sa po nraze
projektilu vychli o uhol .
Vypotajte rchlos projektilu.

Projektil a zbern lapa tvoria sstavu hmotnch bodov. Vzhadom na to, e v okamihu
nrazu projektilu do lapaa nepsob v horizontlmom smere iadna vonkajia sila, je mon
kontatova, e hybnos sstavy pred nrazom a v okamihu nrazu je rovnak, t.j. plat
v v ) (
2 1 1 1
m m m + = , (a)
resp.
v m m v m ) (
2 1 1 1
+ =
1
2 1
1
v
m m
m
v
+
= , (b)
kde v je spolon rchlos projektilu a lapaa v okamihu spojenia.
Poas pohybu spojenho projektilu a lapaa z polohy definovanej okamihom spojenia po ich
maximlne vychlenie psobia na sstavu hmotnch bodov iba konzervatvne sily. Z toho
dvodu mon pre alie rieenie lohy poui zkon zachovania mechanickej energie, t.j.
kontanta = + = +
pB kB pA kA
E E E E , (c)
po dosaden
) cos 1 ( ) ( 0 0 ) (
2
1
2 1
2
2 1
+ + = + + gl m m v m m , (d)
je spolon rchlos vyjadren v tvare
) cos 1 ( 2 = gl v . (e)
Rchlos projektilu je pomocou vzahov (b) a (e)
) cos 1 ( 2
1
2 1
1

+
= gl
m
m m
v .






v
1
m
2
l


m
1
0

A

B

129
Prklad 3.17
Dva hmotn body m
1
a m
2
vzjomne spojen lanom s umiestnen na ramene a spolu s
ramenom rotuj okolo vertiklnej osi kontantnou uhlovou rchlosou . Vzdialenos bodu
m
1
od osi rotcie je r
1
a vzdialenos bodu m
2
od osi rotcie je r
2
. Ak je trenie medzi rmom
a hmotnmi bodmi zanedban, vypotajte:
a silu v lane spjajcom hmotn body,
b silu psobiacu od hmotnho bodu m
2
na doraz.


Vzhadom na to, e drha obidvoch hmotnch bodov pri rotcii je krunica, ich vsledn
zrchlenie m dve zloky normlov a tangencilnu. V prpade, e sstava rotuje
kontantnou uhlovou rchlosou , potom uhlov zrchlenie = 0 a potom aj tangencilne
zloky zrchlenia 0
1 1
= = r a
t
, 0
2 2
= = r a
t
.
Pre kad hmotn bod je zostaven pohybov rovnica pre normlov smer, t.j. plat

.
,
d n
n
F F a m
F a m
=
=
2 2
1 1
(a)
Normlov zloky zrchlen jednotlivch bodov je mon vyjadri v tvare

1
2
1
1
r
v
a
n
= , resp.
2
2
2
2
r
v
a
n
= . (b)
Pre obvodov rchlosti v
1
a v
2
hmotnch bodov m
1
a m
2
s pre prpad pohybu bodov po
krunici

1 1
r v = , resp.
2 2
r v = . (c)
Po dosaden (c) a (b) do pohybovch rovnc (a)

.
,
d
F F r m
F r m
=
=
2
2 2
2
1 1
(d)
Z tchto dvoch rovnc s vyjadren:
- sila v lane
2
1 1
= r m F ,
- sila na doraz ) (
2 2 1 1
2
r m r m F
d
= .



m
2


r
1
r
2
G
2
G
1
m
1
F

F

F
d
a
n1
a
n2
m
2
m
1


r
1
r
2
130
Prklad 3.18
Dva hmotn body m
1
a m
2
vzjomne spojen lanom (vi. prklad 3.17), s umiestnen na
ramene a spolu s ramenom rotuj okolo vertiklnej osi kontantnou uhlovou rchlosou .
Vzdialenos bodu m
1
od osi rotcie je r
1
a vzdialenos bodu m
2
od osi rotcie je r
2
. Faktor
mykovho trenia medzi hmotnm bodom m
1
a ramenom je f
1
a medzi hmotnm bodom m
2
a ramenom f
2
. Vypotajte:
a - silu v lane spjajcom hmotn body,
b - silu psobiacu od hmotnho bodu m
2
na doraz.
Vsledok:
a) ) (
1
2
1 1
gf r m F = ,
b) g f m f m r m r m F
d
) ( ) (
2 2 1 1
2
2 2 1 1
+ = .

Prklad 3.19


Balk s hmotnosou m
z
je hoden rchlosou
v
z
na ploinu dopravnho vozka, ktorho
hmotnos je m
v
. Vozk je v okamihu dopadu
balka v pokoji, t.j. jeho rchlos je v
v
= 0.
Vypotajte spolon rchlos vozka s
balkom.
Vsledok:
v z
z z
m m
v m
v
+
=

Prklad 3.20

Dva hmotn body m
1
a m
2
s pripojen na lano
poda obrzka. V zaiatonom stave je hmotn
bod m
1
v polohe urenej bodom A
0
. Zo
zaiatonej polohy s uvonen a nsledne sa
obidva hmotn body pohybuj a do zastavenia.
Vypotajte drhu hmotnho bodu m
1
. Definujte
podmienku platnosti formulovanho problmu.
Vsledok:
2
1
2
2
2 1
1
4
4
m m
l m m
s

= - platn pre
2 1
2m m <










m
v
m
z
v = ?
m
v
m
z
v
z
v
v
= 0
m
1
m
2
s
1
l
0
l
0
s
2
A
0
131
Prklad 3.21
Na nehmotnom ramene rotujcom okolo
zvislej osi sa nachdza hmotn blok m, ktor
je pomocou lana vinceho sa cez nehmotn
kladku spojen s bremenom m
0
. V zaiatonom
stave (t = 0 s) sa hmotn blok m nachdza vo
vzdialenosti l
0
a rameno je roztoen na
kontantn uhlov rchlos
0
. Medzi
ramenom a hnotnm blokom je faktor
mykovho trenia f. Uvonenm sa hmotn
blok m postupne pribliuje k osi rotcie.
Vyjadrite asov zvislos uhlovej rchlosti
(t) rotcie ramena.

Vsledok:

2
2
0
0
0
2
0
0
) ( 2
) (

=
gt
m m
fm m
l
l
t

Prklad 3.22
Dva hmotn bloky s hmotnosami m
1
a m
1
, umiestnen na ploine, s spojen lanom cez
sstavu nehmotnch kladiek. Jeden koniec lana je pevne uchyten na rm a na druhom
konci lana psob ahov sila. Medzi blokmi a ploinou, po ktorej sa bloky mkaj, s
faktory mykovho trenia f
1
a f
2
.
Urte, akmi zrchleniami sa pohybuj hmotn bloky.

Vsledok:
1
1
1
2
gf
m
F
a =
2
2
2
2
gf
m
F
a =



m
1
m
2
F

f
1
f
2
f
l
0
m

m
0


x(t)

132
3.3 GEOMETRIA HMT
Veobecn defincia momentov zotrvanosti a devianch momentov
- pre SHB

=
n
i=
i i
r m I
1
2
, (3.16)
- pre tuh teleso

=
) (
2
d
m
m r I . (3.17)
Momenty zotrvanosti SHB, resp. tuhho telesa k sradnicovm osiam

+ =
n
i=
i i i x
z y m I
1
2 2
) (

+ =
) (
2 2
d ) (
m
x
m z y I ,

+ =
n
i=
i i i y
x z m I
1
2 2
) (

+ =
) (
2 2
d ) (
m
y
m x z I , (3.18)

+ =
n
i=
i i i z
y x m I
1
2 2
) (

+ =
) (
2 2
d ) (
m
z
m y x I .
Momenty zotrvanosti SHB, resp. tuhho telesa k sradnicovm rovinm

=
n
1 = i
2
i i
z m I
xy

=
) (
2
d
m
xy
m z I ,

=
n
i=
i i yz
x m I
1
2

=
) (
2
d
m
yz
m x I , (3.19)

=
n
i=
i i zx
y m I
1
2

=
) (
2
d
m
zx
m y I .
Momenty zotrvanosti SHB, resp. tuhho telesa k stredu sradnicovho systmu

+ + =
n
i=
i i i i P
z y x m I
1
2 2 2
) ( ,

+ + =
) (
2 2 2
d ) (
m
P
m z y x I . (3.20)
Devian momenty SHB, resp. tuhho telesa

=
n
i=
i i i xy
m y x D
1

=
) (
d
m
xy
m xy D ,

=
n
i=
i i i yz
m z y D
1

=
) (
d
m
yz
m yz D , (3.21)

=
n
i=
i i i zx
m x z D
1

=
) (
d
m
zx
m zx D .

133
Transforman vzahy

zx xy x
I I I + = ,
xy yz y
I I I + = ,
yz zx z
I I I + = ,

z xy p
I I I + = ,
x yz p
I I I + = ,
y zx p
I I I + = (3.22)

yz xy zx p
I I I I + + = ,
2
z y x
p
I I I
I
+ +
= .
Steinerova veta - momenty zotrvanosti, devian momenty k rovnobene posunutm osiam
, ) ( 2 2
2 2
m e e m z e m y e I I
z y T z T y x x
+ + =


, ) ( 2 2
2 2
m e e m z e m x e I I
z x T z T x y y
+ + =


, ) ( 2 2
2 2
m e e m y e m x e I I
y x T y T x z z
+ + =


, 2
2
m e m z e I I
z T z xy y x
+ =


, 2
2
m e m x e I I
x T x yz z y
+ =

(3.23)
, 2
2
m e m y e I I
y T y zx x z
+ =


, ) ( 2 2 2
2 2 2
m e e e m z e m y e m x e I I
z y x T z T y T x P P
+ + + =


, 2 2 m e e m x e m y e D D
y x T y T x xy y x
+ =


, 2 2 m e e m y e m z e D D
z y T z T y yz z y
+ =


. m e e m x e m z e D D
z x T z T x zx x z
+ =

2 2

Moment zotrvanosti ku veobecnej osi prechdzajcej stredom sradnicovho systmu

. cos cos cos cos cos cos
cos cos cos

+ + =
zx yz xy
z y x o
D D D
I I I I
2 2 2
2 2 2
(3.24)
Veobecn vyjadrenie momentu zotrvanosti a devianho momentu tvaru zloenho
z m jednoduchch telies

=
m
i=
i
I I
1
,

=
m
i=
i
D D
1
.
z
z
y
x
x
y
0
0
e
x
e
z
e
y
x

y

z

x
z
y
T [x
T
, y
T
, z
T
]
dm
134
Model Momenty zotrvanosti

) (
3
1
2
2 2 1
2
1
l l l l m I
y
+ =

2
2
1
mr I
x
= ) 3 (
12
2 2
r h
m
I I
z y
+ = =
2
4
1
mr I I
zx xy
= =
2
12
1
mh I
yz
=
) 6 (
12
2 2
r h
m
I
T
+ =
) (
12
2 2
c b
m
I
x
+ =
) (
12
2 2
c a
m
I
y
+ =
) (
12
2 2
b a
m
I
z
+ =

2
12
c
m
I
xy
=

2
12
a
m
I
yz
=

2
12
b
m
I
zx
=

) (
12
2 2 2
c b a
m
I
T
+ + =

) 2 (
20
2 2
h r
m
I I
y x
+ = =
2
10
3
mr I
z
=
2
20
3
mr I I
zx yz
= =
2
10
1
mh I
xy
=
) 3 (
10
2 2
0
h r
m
I + =

2
5
2
mr I I I
z y x
= = =
2
5
1
mr I I I
zx yz xy
= = =
2
5
3
mr I
T
=
h
r
x
y
z
0
x
y
z
r
m
T
h
x
y
z
m
b
a
c
T
x
y
z
l
1
l
2
m
x
y
z
m
r
T
135

Prklad 3.23
Vypotajte moment zotrvanosti oceovej siastky vzhadom na os o. Hustota ocele je
Fe
.








Moment zotrvanosti siastky vzhadom k osi o je dan stom momentov zotrvanosti
jednotlivch ast, z ktorch je siastka zloen, t.j.

o o o o o
I I I I I
4 3 2 1
+ + + = , (a)
- moment zotrvanosti telesa 1:

2
1
2
1 1 1
2
1
h m r m I
o
+ = , (b)

1
2
1 1 1
l r V m
Fe Fe
= = ,
- moment zotrvanosti telesa 2:

2
2 2 2
2
1
r m I
o
= , (c)

2
2
2 2 2
l r V m
Fe Fe
= = ,
- moment zotrvanosti telesa 3:
( )
2
2
1
3
2
1
2
0 3 3
2 12
1

+ + = h
h
m h b m I
o
, (d)
t b h V m
Fe Fe 0 1 3 3
= = ,
- moment zotrvanosti telesa 4:
( )
2
2
0
4
2
0
2
0 4 4
2 12
1

+ + = h
h
m h b m I
o
, (e)
) (
0 0 0 4 4
t t b h V m
Fe Fe
= = .
b
0
h
0
h
2
t
0
t

o

C
b
0
h
1
h
2
t

o

C
r
2
l
2
o

C
r
1
l
1
o

h

C
r
1
r
2
h

l
2
l
1
b
0
h
0
h
1
h
2
t
0
t

o

136
Po dosaden vzahov (b)-(d) do rovnice (a)

]. [
] [
) (
) (
) ( ) (
) (
) (
0 2 2
2
0
2
0 0 0 0
1 2 2
2
1
2
0 0 1
2
1
2
1 2
4
2 1
4
1
12 4
12
12 4
12
2
h h h h b t t b h
h h h h b t b h
h l r l r l r I
Fe
Fe
Fe o
+ +

+
+ +

+ +

=


Prklad 3.24
Vypotajte moment zotrvanosti trojuholnkovej dosky vzhadom na os o.


Veobecn vzah pre vpoet momentu zotrvanosti k osi o je vyjadren v tvare

+ = =
) (
2 2
) (
m
x o
dm z y I I . (a)
Z trojuholnkovej dosky je vybrat element, ktorho hmotnos je
dxdydz dm = . (b)
Po dosaden za dm do rovnice (a) je moment zotrvanosti vyjadren ako trojn integrl


+ =
b x y t
o
dxdydz z x y I
0
) (
0 0
2 2
] )) ( [( . (c)
Vzhadom na geometrick tvar dosky je sradnica y funkciou x, t.j. z podobnosti
trojuholnkov vyplva
x
b
h
x y = ) ( . (d)
Postupnou integrciou rovnice (c) je vypotan moment zotrvanosti dosky k osi o

,
3
3

3
)) ( ( ] )) ( [(
0
3
3
3
3
0
) (
0
3
2
0
) (
0 0
2 2

+ =

+ = + =
b
b x y b x y t
o
dx x
b
h t
x
h
th
dxdy
t
x y t dxdydz z x y I

[ ], 2
6
2 2
t h
m
I
o
+ = kde
2
bht
m = je hmotnos dosky.
y
x
dx
dy
b
h
t
o
137

Prklad 3.25

Vypotajte moment zotrvanosti anuloidu s hmotnosou
m, polomerom R vzhadom na zvisl os. Prierez m
polomer r.
Vsledok:

+ =
2 2
4
3
r R m I
o



Prklad 3.26

Vypotajte moment zotrvanosti telesa vzhadom na
os o. Mern hmotnos ocele je
Fe
.
Vsledok:

+

= ) 5 (
) (
5
10
1 2
4
1
2 0
0
5
2
5
2 4
2
h h r
r r
h r r
h r I
Fe
o



Prklad 3.27

Vypotajte momenty zotrvanosti polgule s polomerom
R a hmotnosou m vzhadom k osiam prechdzajcim
aiskom.
Vsledok:
2
320
83
mR I I
y x
= = ,
2
5
2
mR I
z
=

Prklad 3.28
Vypotajte moment zotrvanosti telesa vzhadom
k osi 0 zloenho z dvoch po dke homognnych
ty kontantnch prierezov s hmotnosami m
1
a
m
2
.
Vsledok:

+ + + =
2
0
2 2
1 2
2
2 2
2
1 1 0
2 12
1
3
1
l
l
l m l m l m I .
m
1
m
2
l
1
l
2
l
0
0

x

R

y

z

T

o
r
0
r
1
r
2
h
0
h

h
1
h
2
o

138
3.4 DYNAMIKA TUHHO TELESA
Hybnos tuhho telesa
posuvn pohyb
T
mv H = ,
rotan pohyb
To T
m m r v H = = (aisko T mimo osi rotcie o), (3.25)
veobecn pohyb
T
mv H = .
Moment hybnosti tuhho telesa
posuvn pohyb
T T
mv r L = ,
rotan pohyb L
o o
I = , (3.26)
veobecn pohyb v r L
T T T
I m + = .
Kinetick energia tuhho telesa
posuvn pohyb
2
2
1
T k
mv E = ,
rotan pohyb
2
2
1
=
o k
I E , (3.27)
veobecn pohyb
2 2
2
1
2
1
+ =
T T k
I mv E .
Pohybov rovnice tuhho telesa
posuvn pohyb

=
n
i=
E
i T
m
1
F a , (3.28)
,

=
n
i=
E
i iT
1
F r 0 (rovnovna momentov rovnica) (3.29)
rotan pohyb

=
n
i=
E
i
1
F 0 , (rovnovna silov rovnica) (3.30)

=
i
i o o
I M , (3.31)
veobecn pohyb

=
=
n
i
E
i T
m
1
F a , (3.32)

=
=
n
i
E
i iT T
I
1
F r , (3.33)
kde m je hmotnos telesa,
I
T
- moment zotrvanosti telesa k aisku,
I
o
- moment zotrvanosti telesa k osi rotcie,
r
T
- polohov vektor aiska,
v
T
- vektor rchlosti aiska,
a
T
- vektor zrchlenia aiska,
- vektor uhlovej rchlosti telesa,
- vektor uhlovho zrchlenia telesa,

E
i
F - vektor externho silovho inku.
139
Prklad 3.29
Posuvn dvere o hmotnosti m a rozmeroch l
0
h s zavesen na vodorovnom nosnku. Na
pravej strane (A) s dvere zavesen na kladke so zanedbatenmi odpormi a na avej strane
(B) s zavesen pomocou mykovho zvesu s faktorom mykovho trenia f. V lane
pripojenom ku kladke s dvere ahan kontantnou silou F, ktor zane psobi v ase t = 0 s,
kedy s dvere v pokoji. Urte, akm zrchlenm a
Tx
sa bud dvere pohybovat a za ak as
sa dvere presun tak, aby uzatvorili otvor rky l
0
.


V prpade psobenia sily F dvere konaj posuvn pohyb. Zrchlenie dver
Tx
a a as t, za
ktor dvere uzatvoria otvor rky l
0
, je mon uri z pohybovej rovnice. Pohybov rovnica
pre horizontlny pohyb dver je zostaven pomocou metdy uvoovania, t.j.

A T
T F ma
x
= . (a)
Zrove platia pre dvere rovnice rovnovhy
0 =

i
iy
F 0
B A
= + G N N , (b)
0
T
=

i
i
M 0 ) ( ) (
A 0 0 0 B 2 A 1
= + + + + + T h h F h r h N l N l . (c)
Vzhadom na to, e tri rovnice (a-c) obsahuj tyri neznme -
A B A T
, , , T N N a
x
, je potrebn
z dvodu rieitenosti sstavy rovnc formulova aliu doplnkov rovnicu zaloen na
vzjomnej zvislosti trecej (T
A
) a normlovej sily (N
A
), t.j.

A A
fN T = . (d)
Z rovnice (b) je vyjadren sila N
B

A B
N G N = . (e)
Dosadenm (d) a (e) do (c) s vypotan normlov sily v bodoch A a B

) (
) (
0 2 1
0 0 2
A
h h f l l
F r h h mg l
N
+ +
+ +
= ,

) (
) ( )) ( (
0 2 1
0 0 0 1
B
h h f l l
F r h h mg h h f l
N
+ +
+ + + +
= .
l
1
l
2
h
0
r
0
T

l
0
B

F
A
N
B
N
A
T
A
G

a
Tx
h

h

l
1
l
2
h
0
r
0
T

l
0
A

B

l
0
m

h

h

140
Z pohybovej rovnice (a) po dosaden za T
A
je vyjadren zrchlenie

) (
) (
0 2 1
2 0 2 1
T
h h f l l
mg fl F fr l l
a
x
+ +
+ +
= . (f)
Vzhadom na to, e vetky leny na pravej strane rovnice s kontanty, je zrejm, e aj
zrchlenie
x
a
T
mus by kontantn. Z toho dvodu pre uvaovan podmienky konaj dvere
rovnomerne zrchlen pohyb. Zrchlenie dver mono vyjadri v tvare

dt
dv
a
x
x
T
T
= ,
odkia po separcii premennch a nslednej integrcii v prslunch hraniciach je vyjadren
rchlos posuvnho pohybu dver
dt a dv
x x T T
=

=
t
x
v
x
dt a dv
x
0
T
0
T
T
t a v
x x T T
= .
Pre rchlos veobecne plat

dt
dx
v
x
=
T


= =
t
x
t
x
l
tdt a dt v dx
0
T
0
T
0
0

2
2
T 0
t
a l
x
= .
Z poslednej rovnice je vyjadren t

x
a
l
t
T
0
2
=
a po dosaden za
x
a
T
je as potrebn na uzatvorenie otvoru rky l
0
vyjadren vzahom

+ +
+ +
=
mg fl F fr l l
h h f l l
l t
2 0 2 1
0 2 1
0
) (
) (
2 .

Prklad 3.30
Teleso s hmotnosou m je z kudovej polohy ahan silou F na naklonenej rovine (obr.A)
so sklonom . Faktor mykovho trenia medzi telesom a naklonenou rovinou je f.
Vypotajte zrchlenie, akm sa bude teleso pohybova, asov zvislos drhy telesa a
vzbov reakcie v prpade, ke bude an sila F kontantn.

Obr.A

Obr.B
141
Teleso vykonva priamoiary posuvn pohyb. Zostavenie prslunch rovnc je podmienen
uvonenm telesa (obr. B). Teleso vykonva pohyb iba v smere naklonej roviny a z toho
dvodu bude zostaven iba jedna pohybov rovnica pre smer naklonenej roviny. alie dve
rovnice - smer kolmo na naklonen rovinu a rotcia okolo aiska bud rovnovne.
Pohybov rovnica pre smer naklonenej roviny (zostaven metdou zrchujcich sl):
= sin G T T F ma
B A
. (a)
Rovnovne rovnice:
- smer kolmo na naklonen rovinu

=
i
i
F 0: 0 cos = + G N N
B A
, (b)
- momentov rovnovna rovnica vzhadom na aisko

=
i
Ti
M 0 : 0 ) (
2 1 0
= + +
B A T B A
T T h N l N l Fh . (c)
Je potrebn uvies vzahy pre mykov sily
B A
T T , v bodoch A a B

A A
fN T = ,
B B
fN T = , (d)
kde
B A
N N , s normlov sily v bodoch A a B.
Z rovnc (b)-(d) s vyjadren normlov sily v bodoch A a B, t.j.

) (
cos ) (
2 1
0 1
l l
F h mg fh l
N
T
A
+
+ +
= , (e)

) (
cos ) (
2 1
0 2
l l
F h mg fh l
N
T
B
+

= . (f)
Po dosaden (d) a (e) do pohybovej rovnice (a) je vekos zrchlenia vyjadren v tvare
) cos (sin + = f g
m
F
a . (g)
Vzhadom na to, e vetky leny pravej strany rovnice (f) s kontanty, aj zrchlenie pohybu
telesa po naklonenej rovine bude kontantn, a teda mono kontatova, e teleso kon
rovnomerne zrchlen pohyb.
Rchlos pohybu telesa je vyjadren nasledovnm postupom:

dt
dv
a = adt dv =

=
t v
adt v d
0 0
at v = .
Pre drhu telesa po naklonenej rovine potom plat:

dt
ds
v = atdt vdt ds = =

=
t s
atdt s d
0 0

2
2
t
a s = .
asov zvislos drhy je definovan vzahom

2
2
) cos (sin
t
f g
m
F
s

+ = .
142
Prklad 3.31
Na vozku, ktor sa pohybuje zrchlenm a
u
, je umiestnen ty, ktor je v bode A spojen s
lonou plochou kbom a v bode B je opret. Ty m hmotnos m a je po celej dke l
homognna. Vypotajte:
a - minimlnu vekos zrchlenia a
u
, pri ktorom nastane strata kontaktu medzi tyou a
vozkom v bode B,
b - vekos sily, ktorou je zaaovan ap v bode A.

Obr. A

Obr. B

Pre ty, ktor je uloen na vozku, s (poda vpotovho modelu na obr. B) zostaven
nasledovn rovnice:
0 =

i
xi
F 0 =
B Du x
N F A ,
0 =

i
yi
F 0 = G A
y
, (a)
0 =

i
Ai
M 0 cos
2 2
= + h N
l
G
h
F
B Du
,
kde

=
l
h
arcsin .
V rovniciach (a) je pouit dAlembertova zotrvan sila F
Du
, ktor psob na ty a je
spsoben unavm pohybov vozka. Pre vektorov vyjadrenie unavej zotrvanej sily F
Du


u Du
ma F = ,
priom pre jej vekos plat

u Du
ma F = .
Kontakt v bode B medzi tyou a vozkom nastane vtedy, ke bude plati
0
B
N .
Na zklade tohto kontatovania, ak v tretej rovnici (a) polome 0 =
B
N , pre minimlnu
vekos zrchlenia vozka plat
cos
h
l
g a
u
.

m,I
O
l
2
l
2
T

A

N
B
G

A
x
A
y
a
u
B

h
l
h
m
a
u
A
B
143
Pre vekos sl psobiacich na ap v smere sradnicovch os potom
= cos
h
l
mg A
x
,
mg A
y
= ,
resp vsledn silov inok na ap
1 cos
2
2
2 2
+

= + =
h
l
mg A A A
y x
.

Prklad 3.32
Hnac moment elektromotora je definovan zvislosou ) 1 (
0
=
m h
k M M , kde n = 2
je uhlov rchlos, n s otky elektromotora, M
0
a k
m
s kontanty elektromotora.
Elektromotor m v celom rozsahu otok kontantn straty vyjadren kontantnm
odporovm momentom
r
M . Moment zotrvanosti rotora elektromotora je
m
I .
Vypotajte:
a - asov zvislos otok ) (t n n = ,
b - maximlne otky rotora elektromotora a as, za ktor ich rotor dosiahne.



V rmci tejto lohy je rieen problm dynamiky rotanho pohybu telesa. Vchodiskom pre
rieenie je zostavenie pohybovej rovnice

r h m
M M I = . (a)
Pre vyjadrenie zrchlenia je pouit vzah

dt
d
=
a po dosaden do rovnice (a) dostaneme

r m m
M k M
dt
d
I =

) 1 (
0
,
resp.

m
m
m
r
I
k M
I
M M
dt
d
0 0
. (b)

I
m
M
h
M
r


144
Po separcii premennch je
dt
I
k M
I
M M
d
m
m
m
r
=


0 0

a nslednou integrciou v prslunch hraniciach


=


t
m
m
m
r
dt
I
k M
I
M M
d
0 0
0 0
,
je zskan asov zvislos uhlovej rchlosti

= =
t
I
k M
m
r
m
m
e
k M
M M
t
0
1 ) (
0
0
. (c)
asov zvislos otok elektromotora je vyjadren zo vzahu n = 2 , t.j. plat

=
t
I
k M
m
r
m
m
e
k M
M M t
t n
0
1
2 2
) (
) (
0
0
. (d)
Maximlne otky elektromotora s vypotan njdenm extrmu funkcie ) (t n
0
) (
=
dt
t dn
. (e)
Pre as
max
t , pri ktorom bude ma elektromotor maximlne otky, mus poda rovnice (e)
splni podmienku
0
max
0
=
t
I
k M
m
m
e . (f)
Rovnica (f) je splnen iba pre prpad, ke

max
t . (g)
0
0,0
n
max
O
t

k
y

r
o
t
o
r
a


n
(
t
)


as t
Obr. A

To znamen, e rotor elektromotora
nedosiahne maximlne otky v relnom
asovom horizonte, ale iba sa k nim
asymtoticky pribliuje (vi. obr. A).
Asymtotick hodnota maximlnych
otok, ku ktorm sa blia otky
rotora, je vyjadren z rovnice (d) po
dosaden za
max
t z rovnice (g), t.j.

m
r
k M
M M
n
0
0
max
2

= .

145
Prklad 3.33
Lietadlo s hmotnosou m pristva pristvacou rchlosou v
0
na dve koles, ktor s pri
pristvan v pokoji. Koles maj polomer r a moment zotrvanosti I. Faktor mykovho
trenia medzi kolesami a pristvacou plochou je f. Pasvne odpory v loiskch kolies s
zanedban. Vypotajte, za ak as sa koles prestan mka a zan sa odvaova.


Pri zanedban vztlakovej sily je koleso lietadla pri pristvan na pristvaciu plochu pritlan
silou, ktor sa rovn polovinej hodnote gravitanej sily psobiacej na lietadlo, t.j.

N k k
F g m
m
G
G
F =

+ = + =
2 2
1
.
V okamihu dotyku kolesa s pristvacou plochou zane na koleso psobi trecia sila
g m
m
f fF F
k N T

+ = =
2
. (a)
Pohybov rovnica kolesa liedadla pre jeho rotan pohyb m potom tvar
r F I
T k
= , (b)
z ktorej pre uhlov zrchlenie plat
r
I
fg
m
m
r
I
F
k
k
k
T

+ = =
2
. (c)
Kee veliiny nachdzajce sa na pravej strane rovnice s kontantn, bude kontantn sj
uhlov zrchlenie . Pre asov zvislos uhlovej rchlosti rotcie kolesa potom plat
rt
I
fg
m
m
t
k
k

+ = =
2
. (d)
Koleso sa po pristvacej drhe prestane mka, ak zane plati vzah pre odvaovanie kolesa

r
v
0
= . (e)
Platnos rovnice (e) nastane vtedy, ak sa dotykov body na obvode kolesa a body na pristvacej
drhe nebud voi sebe premykova, t.j. vznikaj podmienky pre odvaovanie kolesa a
dotykov body s vlastne okamit stredy otania.
Z rovnice (d) je po dosaden rovnice (e) vyjadren as, kedy prestane mkanie kolies po
pristvacej drhe

) 2 (
2
2
0
k
k
m m fgr
I
v t
+
= .
m
k
, I
k
r
F
1
F
N
F
T
v
0


146
Prklad 3.34
Homognny kruhov kot je ovinut nehmotnm lankom. Jeden koniec lanka je upevnen
na valcovej asti kota a druh koniec je pripojen na nehybn zklad. Kot sa po
uvonen zane pohybova. Vypotajte zrchlenie aiska kota, uhlov zrchlenie kota a
silu, ktor pri pohybe kota vznika v lanku. Pasvne odpory zanedbajte.

Obr. A

Obr. B


Obr. C
Kot sa po uvonen zane pohybova a kon veobecn pohyb, t.j. posuvn pohyb smerom
dolu a zrove rotan pohyb okolo aiska. Pohybov rovnice, z ktorch je mon
poadovan veliiny vypota, mu by zostaven dvoma spsobmi:
- metda zrchujcich sl (poda obr. B)
N G ma = , (a)
Nr I
T
= , (b)
- dAlembertovou metdou (poda obr. C)
0 F F = +

i
i D
0 =
D
F N G , (c)
0 M M = +

i
Ti DT
0 = rN M
D
, (d)
kde a F m
D
= - vektor dAlembertovej zotrvanej sily - vekos je ma F
D
= ,

T D
I = M - vektor dAlembertovho zotrvanho momentu - vekos je =
T D
I M .
Po dosaden za F
D
a M
D
do rovnc (c) a (d) s zskan tie ist pohybov rovnice, ako s
rovnice zskan metdou zrchujcich sl, t.j.
N G ma = ,
Nr I
T
= .
Vzhadom na to, e mme zostaven iba dve pohybov rovnice a v tchto rovniciach s tri
neznme (a, , N), ktor mme vypota, je potrebn zostavi aliu doplnkov rovnicu
medzi uvedenmi neznmymi, t.j.
r a = . (e)
Rieenm sstavy rovnc (a), (b) a (e) s vyjadren poadovan veliiny
g a
3
2
= ,
r
g
3
2
= ,
3
mg
N = .

m, I
T
r
T
N
G
F
D
M
D
OSO

m, I
T
r
T
N
G
a

OSO
m, I
r
T
147
Prklad 3.35
Rotane symetrick teleso s hmotnosou m je spusten po naklonenej rovine s uhlom
sklonu . Teleso sa val na polomere r. Drhu valenia l teleso prejde za as t. Vypotajte
moment zotrvanosti I
T
telesa vzhadom na os symetrie.

Teleso pri valen vykonva veobecn pohyb a pre rieenie lohy s zostaven rovnice:
- pohybov rovnica pre pohyb aiska telesa v smere naklonenej roviny

T T
F G ma = sin , (a)
- pohybov rovnica pre rotciu telesa okolo aiska
r F I
T T
= , (b)
- rovnica rovnovhy telesa v smere kolmo na naklonen rovinu

=
i
i
F 0 0 cos = G F
N
. (c)
V uvedench troch rovniciach je p neznmych velin:
a
T
, , F
T
, F
N
, I
T
.
Doplnkov kinematick rovnica, ktor je pouit pre rieenie:
r a
T
= . (d)
Z rovnice (d) je vyjadren uhlov zrchlenie

r
a
T
=
a z rovnice (b) je vyjadren trecia sila

T
T T
T
a
r
I
r
I
F
2
=

= . (e)
Po dosaden (e) do pohybovej rovnice (a) je vyjadren zrchlenie aiska

2
sin
r
I
m
mg
a
T
T
+

= . (f)

l

v
T0
= 0

0
= 0
t
r
R
T
T
T
a
T


v
T

t = 0
F
N
F
T

G

148
Vetky veliiny na pravej strane rovnice (f) s kotantn a z toho vyplva, e aj zrchlenie
aiska je kontantn. Na zklade toho teleso kon rovnomerne zrchlen pohyb.
Rchlos pohybu aiska telesa:

dt
dv
a
T
T
= dt a dv
T T
=

=
t
T T
v
dt a v d
T
0 0
t a v
T T
= .
Drha aiska telesa po naklonenej rovine:

dt
ds
v
T
T
= tdt a dt v ds
T T T
= =

=
t
T T
l
tdt a s d
0 0

2
2
t
T
a l = .
Po dosaden do poslednej rovnice za a
T
z rovnice (f)

2 2
2
2 sin
t
T
I mr
g mr
l

+

= ,
odkia pre moment zotrvanosti telesa plat

= 1
2
sin
2
2
l
gt
mr I
T
.

Prklad 3.36
Pre ojnicu kukovho mechanizmu, ktor m hmotnos m a je zobrazen na obrzku A,
vypotajte moment zotrvanosti k aisku.

Obr. A

Obr. B

Obr. C
T

l
T
l

A

B

m

G

N
A
N
B
r

T

l
T
l

A

m


T

G


r

T

l
T
l

r

149
Ojnica kukovho mechanizmu je tvarovo komplikovan, prpadne nehomognne teleso.
Vzhadom na to je vemi oban vypota momenty zotrvanosti pomocou zkladnch
defininch vzahov. Pre teles tohto typu je mon vypota ich moment zotrvanosti
nepriamo. Podstatou rieenia je tzv. fyziklne kyvadlo.
Ojnica je kbovo uchyten v bode A (obr. C) a po vychlen o uhol a nslednom uvonen
bude kona rotan pohyb okolo bodu A. Jej pohybov rovnicu mono napsa metdou
zrchujcich sl v tvare
+ = sin ) ( r l G I
T A
, (a)
kde = & & je uhlov zrchlenie a
A
I je moment zotrvanosti ojnice na bod A.
Po prave m rovnica (a) tvar
0 sin ) ( = + + r l mg I
T A
& & 0 sin
) (
=
+
+
A
T
I
r l mg
& & . (b)
Rovnica (b) je vzhadom na sin nelinerna diferencilna rovnica. Pre mal uhly
o
5 <
plat sin (vo vzahu je v radinoch). Na zklade tohto predpokladu je diferencilna
rovnica linearizovan a m tvar
0
) (
=
+
+
A
T
I
r l mg
& & 0
2
0
= + & & .
Pre vlastn kruhov frekvenciu
0
oscilanho pohybu ojnice plat

0
0
2 ) (
T I
r l mg
A
T

=
+
= , (c)
kde T
0
je perida oscilanho pohybu ojnice.
Uveden postup je mon aplikova na experimentlne urovanie momentu zotrvanosti
telies, ktor s tvarovo komplikovan a nehomognne. Na vpoet momentu zotrvanosti
A
I
zo vzahu (c) je potrebn zmera as trvania kyvu ojnice, t.j. jednu peridu.
Na vyjadrenie momentu zotrvanosti
A
I potom plat

2
2
0
4
) (

+ =
T
r l mg I
T A
. (d)
Veliiny - m, r, T
0
vystupujce v rovnici (d) s bu zadan, alebo sa daj zmera. Poloha
aiska
T
l , ak nie je zadan, sa taktie d zisti experimentlne poda schmy na obr. B.
Ojnica je na jednej strane podopret v pevnom bode A a druh koniec ojnice je podopret
v bode B, ktor je umiestnen na vhach, pomocou ktorch je odmeran hmotnos m
B
v
podopret bod B. Na zklade schmy (obr. B) je mon zostavi rovnovne rovnice
0 =

i
yi
F 0 = + G N N
B A
,
0 =

i
Ai
M 0 ) ( ) ( = + + G r l N r l
T B
,
kde g m N
B B
= a m
B
je hodnota zmeran na vhach.
150
Z druhej z rovnovnych rovnc pre polohu aiska
T
l vyplva
r
m
m
r l l
B
T
+ = ) ( .
Po dosaden za
T
l do rovnice (d) je vyjadren moment zotrvanosti na bod A

2
2
0
4
+ =
T
r l g m I
B A
) ( .
Pre vpoet momentu zotrvanosti vzhadom na aisko T je pouit Steinerova veta.
Moment zotrvanosti k bodu A, ktor je posunut o
T
l vzhadom na aisko, je vyjadren
pomocou aiska v tvare

2
T T A
ml I I + = .
Pre moment zotrvanosti ojnice vzhadom na aisko plat

2
2
2
0 2
4
) (
T B T A T
ml
T
r l g m ml I I

+ = = ,
resp.

2
2
2
0
) (
4
) (

+ = r
m
m
r l m
T
r l g m I
B
B T
.

Prklad 3.37

Ty kontantnho prierezu s rovnomernm rozloenm
hmotnosti po dke m hmotnos m = 2 kg a dku
l = 35 cm. V bode A je upevnen idelnym rotanm
kbom a v bode B lanom. Vypotajte zrchlenie
aiska tye a vekos reakcie v bode A v okamihu
po preseknut lana.
Vsledok:
a
T
= 7,36 ms
-2
R
A
= 4,9 N

Prklad 3.38

Homognna hmotn ty, ktorej hmotnos je m = 3,63 kg
a dka l = 0,92 m, m konce pripojen k nehmotnm
objmkam v bodoch A a B. Kzanie objmok po tyiach je
uvaovan bez trenia. Ak je ty uvonen zo zaiatonej
polohy definovanej na obrzku ( = 30), vypotajte:
a - uhlov zrchlenie v okamihu po uvonen,
b - reakcie v bodoch A a B.
Vsledok:
= 8,05 rad.s
-2
, R
A
= 11,56 N, R
B
= 28,91 N

m
l
0

B
A
A
B
l
m
151
Prklad 3.39
Po povrchu valcovej plochy s polomerom R sa val
gua s hmotnosou m. Zaiaton rchlos stredu gule
je v
0
. Vypotajte uhol, pri ktorom sa gua odpta od
povrchu valcovej plochy.
Vsledok:

+
+ =
) ( 10
7
1
17
10
arccos
2
0
r R g
v


Prklad 3.40
Bubon s momentom zotrvanosti I a polomerom R sa
ota otkami n
0
. V ase t = 0 s je na bubon
kontantnou silou F pritlaen eus brzdy. Medzi
eusou brzdy a bubnom je faktor trenia f.
Vypotajte, koko otok vykon bubon a do
zastavenia.
Vsledok:
Poet otok do zastavenia
fFR
In
2
0

=

Prklad 3.41

Vozk s kruhovou drhou o polomere R sa pohybuje
po vodorovnej rovine kontantnm zrchlenm a
u
. Na
kruhovej drhe sa nachdza guka s polomerom r,
hmotnosou m a momentom zotrvanosti I. V ase
t = 0 s sa guoka nachdzala v najniej polohe (bod
A). Vypotajte relatvnu rchlos stredu guky, ak
sa valec dostane do polohy definovanej bodom B.
Vsledok:
2
2
2
mr I
gR R a
mr v
u
B
+

= pre g a
u






r
a
u
B
A
F

R

f
I


v
0
R


0

r

m, I
0

0
152
3.5 DYNAMIKA SSTAVY TUHCH TELIES
Metda uvoovania
Uvonenm jednotlivch lenov sstavy tuhch telies a zostavenm pohybovch (resp.
rovnovnych) rovnc pre tieto teles je zskan sstava tzv. vlastnch pohybovch rovnc vo
veobecnom tvare
0 ) , , , , ( = t f
j j j Ai j
q q q F
& & &
, pre j = 1 n, (3.34)
kde n je poet stupov vonosti sstavy telies a
Ai
F ,(i = 1 m) s akn-pracovn sily
psobiace na sstavu telies. Rieenm tejto sstavy rovnc pre zadan zaiaton podmienky
je pohyb sstavy tuhch telies plne popsan.
Metda redukcie
Vlastn pohybov rovnicu sstavy tuhch telies s 1 vonosti (po redukcii)
) (
d
) ( d
2
1
) (
2
q Q q
q
q m
q q m = +

& & & ,



(3.35)
kde m
*
(q) - zoveobecnen (redukovan) hmotnos,
Q(q) - zoveobecnen (redukovan) sila,
q, & q , && q - zoveobecnen sradnica, zoveobecnen rchlos, zoveobecnen zrchlenie.
Redukcia hmotnosti
Rovnos kinetickej energie pvodnej sstavy telies a kinetickej energie redukovanej sstavy

=
n
i=
ki red k
E E
1
,
. (3.36)
Redukcia silovch inkov
Zoveobecnen (redukovan) sila je uren na zklade rovnosti okamitho vkonu pracovnch
sl psobiacich na pvodn sstavu telies a vkonu hadanej redukovanej sily

=
F
n
j=
Pj red P
P P
1
,
, (3.37)
kde n
F
- poet pracovnch sl v pvodnej sstave, n - poet telies v sstave.
Poda toho, na ak objekt je vykonan redukcia, hovorme o redukcii na:
len konajci rotan pohyb
, ) (
2
1
1
2
,
= =
n
i=
ki red red k
E I E &

= =
F
n
j=
Pj red red P
P P
1
,
) ( & M , (3.38)
len konajci posuvn pohyb
, ) (
2
1
1
2
,
= =
n
i=
ki red red k
E x x m E &

= =
F
n
j=
Pj red red P
P x P
1
,
) ( x F
&
, (3.39)
na bod (bod niektorho telesa sstavy)
, ) (
2
1
1
2
,
= =
n
i=
ki A A red red k
E x x m E &

= =
F
n
j=
Pj A A red red P
P x P
1
,
) ( x F
&
. (3.40)
153
Prklad 3.42
Prevodovka pozostva z dvoch ozubench kolies, ktor tvoria eln ozuben skolie. Koles
maj rozostupov krunice R
1
, R
2
a momenty zotrvanosti I
1
, I
2
. Na koleso 1 psob hnac moment
M
1
a na koleso 2 zan moment M
2
. Vypotajte vekos sily medzi zubami v zbere.





lohu rieime metdou uvoovania telies, t.j. uvonme jednotliv ozuben koles s tm, e
s pri ich uvonen uvaovan silov inky odstrnench telies v mieste vzieb. Vzhadom
na to, e v tomto prpade je poadovan iba vpoet sily medzi zubami, nie je potrebn
uvoova jednotliv ozuben koles v loiskch.
Pohybov rovnice jednotlivch telies s zostaven metdou zrchujcich sl, t.j.

1 21 1 1 1
R F M I = , (a)

2 12 2 2 2
R F M I + = . (b)
Uveden rovnice obsahuj tyri neznme, t.j.
12 21 2 1
, , , F F . Neznme veliiny nie je mon
vypota vzhadom na to, e s k dispozcii iba dve rovnice (a) a (b). Z tohto dvodu je
potrebn sformulova dodaton rovnice medzi uvedenmi tyrmi neznmymi.
Prv doplnkov rovnica vyplva zo zkona akcie a reakcie, t.j. pre vekosti sl medzi zubami
ozubench kolies plat
F F F = =
12 21
. (c)
Druh dodatkov rovnica zase vyplva zo vzjomnho pohybu telies, t.j. uhly natoenia,
uhlov rchlosti a uhlov zrchlenia s definovan

2 2 1 1
R R = ,
2 2 1 1
R R = ,
2 2 1 1
R R = . (d)
Zvislos uhlovho zrchlenie telesa 2 od uhlovho zrchlenia telesa 1

2
1
1 2
R
R
= . (e)
R
2

2
M
2
A
F
12
I
2
A
R
2
M
2
M
1

2
R
1
I
1
I
2
A
R
2
M
2
M
1

2
R
1
I
1
I
2
R
1
A
M
1

1 F
21
I
1
154
Pouitm rovnc (c) a (e) v pohybovch rovniciach (a) a (b) dostaneme

1 1 1 1
FR M I = , (f)

2 2
2
1
1 2
FR M
R
R
I + = . (g)
Po vyjadren uhlovho zrchlenia ozubenho kolesa 1 z rovnice (f)

1
1 1
1
I
FR M
= ,
jeho dosadenm do rovnice (g) po nslednej prave je silov inok medzi zubami ozubench
kolies vyjadren vzahom

2
1 2
2
2 1
1 1 2 2 2 1
R I R I
M R I M R I
F
+
+
= .

Prklad 3.43
Prvesn dvojkolesov vozk, na ktorom je naloen debna, je ahan za automobilom,
ktor zrchuje a pohybuje sa zrchlenm a. Hmotnos prvesnho vozka je m
1
. Hmotnos
debny je m
2
. Medzi debnou a povrchom lonej plochy vozka je uvaovan trenie, ktor je
pecifikovan faktor mykovho trenia f. Vetky zaktovan rozmerov parametre s
zadan. Vypotajte vekos sily v pripojen vozka za automobil (silu v bode C) a silu
psobiacu na npravu prvesnho vozka.

Obr. A

Obr. B

Obr. C

m
1
a

T
1
b
1
+ b
2
l
0
l

h
0
h
1
A

B

C

T
A
N
A
l
1
N
B
T
B
F
Cy
F
Cx
O

F
Oy
G
1
b
0
m
2
a

T
2
b
2
b
1
l
0
h
0
h
2
h
1
A

B

C

O

b
0
l

N
A
T
A
T
B
N
B
b
0
m
2
m
1
a

T
2
T
1
b
2 b
1
l
0
l

h
0
h
2
h
1
A

B

C

l
1
b
0
155
loha bude rieen pouitm metdy uvoovania. Postupne bud uvonen teles - debna
(teleso 2) a prvesn vozk (teleso 1). Nsledne s pre uvonen teles zostaven pohybov a
rovnovne rovnice.
Rovnice pre teleso 1:
- pohybov rovnica v horizontlnom smere

B A Cx
T T F a m =
1
, (a1)
- rovnovna rovnica vo vertiklnom smere
0
1
=
O Cy B A
F F N N G , (a2)
- rovnovna rovnica pre rotciu okolo aiska

. 0 ) (
) ( ) (
0 1 0 1
0 1 0 1 0 0 2 1
= + +
+ + + +
Cx Cy O
B A
F h F l l l F l
N b l l N l l b b b
(a3)
Rovnice pre teleso 2:
- pohybov rovnica v horizontlnom smere

B A
T T a m + =
2
, (b1)
- rovnovna rovnica vo vertiklnom smere
0
2
= + +
B A
N N G , (b2)
- rovnovna rovnica pre rotciu okolo aiska
0 ) (
2 1 2
= + +
A B B A
N b N b T T h . (b3)
Pre vekos trecch sl v bodoch A a B plat

A A
fN T = (c2)

B B
fN T = . (c1)
Z rieenia rovnc (b1-b3) a (c) vyplva pre vekosti normlovch sl v bodoch A a B

2 1
2 1
2
b b
ah gb
m N
A
+
+
= ,

2 1
2 2
2
b b
ah gb
m N
B
+

= .
Pouitm sl
A
N a
B
N v prvch troch rovniciach (a1-a3) po nslednch pravch dostaneme
vekosti sl v pripojen prvesnho vozka k anmu automobilu:
a m m F
Cx
) (
2 1
+ = ,

) )( (
) (
2 1 0
2 1 0 0 2 1 1
2
0
2 0
2
0
1 0
1
b b l l
b l l b b b b
g m
l l
h h
a m
l l
gl ah
m F
Cy
+ +
+ +

+
+
+
+

= .
Sila psobiaca na npravu prvesnho vozka

Cy O
F a m m F + + = ) (
2 1
.
156
Prklad 3.44
Na navjac bubon s polomerom r
b
, hmotnosou m
b
a momentom zotrvanosti I
b
je pomocou
nehmotnho lana pripojen bremeno s hmotnosou m. Bubon je pohan momnetom M.
Vypotajte uhlov zrchlenie navjacieho bubna, zrchlenie a bremena m, ahov silu N
v lane, vertiklnu A
y
a horizontlnu A
x
silu na loisk bubna (bod A).



loha je rieen metdou uvoovania, t.j. kad teleso je uvonen a pre kad teleso s
zostaven prslun rovnovne alebo pohybov rovnice.
Rovnice pre navjac bubon:
- rovnovna rovnica v horizontlnom smere
0 =
x
A , (a)
- rovnovna rovnica vo vertiklnom smere
0 = N G A
b y
, (b)
- pohybov rovnica pre rotciu okolo aiska (bod A)
.
b b
Nr M I = (c)
Rovnice pre zdvhan bremeno m:
- pohybov rovnica pre zvisl smer
G N ma = . (d)
Z rovnice (a) priamo vyplva, e loisko bubna nie je zaaovan v horizontlnom smere.
Vzhadom na to, e v alch troch rovniciach s 4 neznme, je zostaven doplnkov rovnica
r a = . (e)
Vzjomnm rieenm sstavy rovnc (b-e) dostaneme:
- uhlov zrchlenie bubna

2
mr I
mrg M
+

= ,
r
b
m
b
A

m

M

r
b
m
b
O

m

M

N

N

G

G
b
A
y
A
x


a

157
- zrchlenie bremena
r
mr I
mrg M
a
2
+

= ,
- silu v lane

2
mr I
Ig Mr
m N
+
+
= ,
- sily v loisku navjacieho bubna
0 =
x
A
2
mr I
Ig Mr
m g m A
b y
+
+
+ = .

Prklad 3.45
Vypotajte silu v lane medzi telesami 2 a 3 zdvhacieho zariadenia, ktor je pohan
krtiacim momentom M posobiacom na teleso 1. Vetky oznaen a zaktovan rozmerov
a hmotnostn parametre s zadan.




Na vpoet sily v lane bud postaova iba pohybov rovnice, ktor bud zostaven metdou
uvoovania a bud doplnen prslunmi kinematickmi rovnicami.
Na zskanie rieenia mus by splnen podmienka - poet zostavench rovnc (pohybov +
doplnkov) sa mus rovna potu neznmych v tchto rovniciach. Vzhadom na poadovan
veliinu nie je potrebn zostavi rovnovne rovnice pre jednotliv teles.
Pohybov rovnice pre jednotliv teles:
- teleso 1 (rotan pohyb):

1 1 1 1
r S M I = ,
- teleso 2 (veobecn pohyb):

2 2 2 1 2 2
r S r S I = ,
m
3

r
2
, m
2
, I
2

M
m
1
, I
1
, r
1

S
1
S
1
S
2
S
3
G
2

2
a
2
a
3
G
3
S
3
m
3

r
2
, m
2
, I
2

M
m
1
, I
1
, r
1

158

2 3 2 1 2 2
G S S S a m + = ,
- teleso 3 (posuvn pohyb):

3 3 3 3
G S a m = .
Uvedenm postupom boli zostaven 4 rovnice, ktor obsahuj 7 neznmych velin:

3 2 1 3 2 2 1
, , , , , , S S S a a .
Aby bolo mon vypota uveden neznme, je potrebn zostavi alie 3 doplujce
kinematick rovnice, ktor bud vyjadrova zvislosti medzi vzjomnmi pohybmi telies.
Doplnkov kinematick rovnice:

3 2
a a = ,

2 2 2
r a = ,

2 2 2 1 1
2 2 a r r = = .
Z rieenia sstavy 7 linernych algebraickch rovnc (4 pohybov rovnice + 3 doplujce
kinematick rovnice) je vypotan sila
3
S , ktor psob v lane medzi telesami 2 a 3

2
2 1
2
1 2
2
2
2
1 3 2
2
2 1
2
1 2
2
2 1
3 3
4 ) (
) 4 ( 2
r I r I r r m m
g r I r I r Mr
m S
+ + +
+ +
= .

Prklad 3.46
Zdvhacie zariadenie je pohan momentom M na telese 1. Metdou uvoovania zostavte
vetky pohybov a rovnovne rovnice. Zrove zostavte vetky doplnkov rovnice
potrebn pre zabezpeenie rieitenosti lohy. Vetky oznaen a zaktovan rozmerov a
hmotnostn parametre s zadan.






Pre kad teleso s zostaven zodpovedajce rovnovne a pohybov rovnice, ktor s
zostaven pomocou metdy uvoovania:
m
4

a
4
G
4
S
4
m
3
, I
3
, r
3
S
2
S
3
S
4
G
3

3
a
3
r
2

m
3
, I
3
, r
3

R
2

R
0

m
4

m
2
, I
2

M
m
1
, I
1
, r
1

m
2
, I
2

r
2

R
2

R
0

S
1

2
S
2
B
y
S
3
B
x
G
2
M
m
1
, I
1
, r
1
S
1
G
1
A
x

1
A
y
159
Teleso 1:

=
i
ix
F 0 0 =
x
A - rovnovna rovnica v smere osi x,

=
i
iy
F 0 0
1 1
= + G S A
y
- rovnovna rovnica v smere osi y,

1 1 1 1
r S M I = - pohybov rovnica pre rotciu telesa.
Teleso 2:

=
i
ix
F 0 0 =
x
B - rovnovna rovnica v smere osi x,

=
i
iy
F 0 0
3 2 1 2
= S S S G B
y
- rovnovna rovnica v smere osi y,

2 3 2 2 0 1 2 2
R S r S R S I + = - pohybov rovnica pre rotciu telesa.
Teleso 3:

4 3 3 2 3 3
S G S S a m + = - pohybov rovnica pre posuv v smere osi y,

3 2 3 3 3 3
r S r S I = - pohybov rovnica pre rotciu telesa.
Teleso 4:

4 4 4 4
G S a m = - pohybov rovnica pre posuv v smere osi y.
Prv rovnice pre rovnovhy telies 1 a 2 v smere osi x nie je potrebn uvaova v systme
rovnc, ktor je potrebn riei. Taktie silov inky A
x
a B
x
nie je potrebn uvaova ako
neznme silov inky.

Pre vzjomn pohyby telies plat:
- dka oblkov
2 1
s s =
2 0 1 1
= R r ,
po derivcii
0 2 0 1 1
v R r = =
2 0 1 1
= R r .

- vekos posuvnej rchlosti v
3
a zrchlenia a
3

2 2
2 2 2 2 2 20
3

=

=
r R v v
v
2
2 2 2 2
3

=
r R
a ,
- vekos uhlovej rchlosti
3
a zrchlenia
3

3
2 2 2 2
3
2 20
3
2 2 r
r R
r
v v +
=
+
=
3
2 2 2 2
3
2r
r R +
= ,
- vekos rchlosti v
4
a zrchlenia a
4

3 4
v v =
3 4
a a = .
Rovnovne, pohybov a doplnkov rovnice tvoria sstavu 11 linernych algebraickch
rovnc s 11 neznmymi, ktor mono zapsa v maticovom tvare, t.j. plat
r
3

v
20

v
2

v
3

3

m
4

v
4

R
0

r
1

2
s
1
s
2

v
0

160



0
0
0
0
0
0
1 1 0 0 0 0 0 0 0 0 0
0 0 2 0 0 0 0 0 0 0
0 2 0 0 0 0 0 0 0 0
0 0 0 0 0 0 0 0 0
0 0 0 0 1 0 0 0 0 0
0 0 0 0 0 0 0 0
0 0 0 0 1 1 1 0 0 0
0 0 0 0 0 0 0
0 0 0 0 0 0 1 1 1 1 0
0 0 0 0 0 0 0 0 0
0 0 0 0 0 0 0 0 1 0 1
4
3
2
1
4
3
3
2
1
4
3
2
1
3 0 2
0 2
0 1
4
3 3 2
2
2 2 2 0
1 1
G
G
G
M
G
a
a
S
S
S
S
B
A
r R r
R r
R r
m
I r r
m
I R r R
I r
y
y
,
resp. v skrtenom zpise
b Ax = ,
kde A - matica koeficientov,
x - vektor neznmych,
b - vektor neznmych.
Sstavu linernych algebraickch rovnc je mon riei pomocou Gaussovej eleminanej
metdy alebo pouitm inverznej matice. Rieenie sstavy linernych algebraickch rovnc
b Ax = pomocou inverznej matice:
b A x
1
= , priom plat E A A =
1
,
kde
1
A je inverzn matica k matici A a E je jednotkov matica.

Prklad 3.47
Na hnaciu kladku 4 zdvhacieho zariadenia zane v ase t = 0 s psobi kontantn hnac
moment M, ktorm sa cez sstavu kladiek zdvha bremeno m
1
. Vypotajte rchlos
bremena m
1
v zvislos od jeho vkovej polohy x
1
. Zaiaton podmienky s uvaovan
v tvare: t = 0 s: x
1
(0) = 0, v
1
(0) = 0.




Redukovan systm

m
1
R
3
r
3
I
3
R
4
, I
4
M

2
v
1
x
1

R
2
, I
2
m
r1
x
1
, v
1
, a
1

F
r1
(x
1
)
161
loha bude rieen metdou redukcie hmotnch a silovch parametrov. Veobecn pohybov
rovnica pre redukovan sstavu m tvar
) (
d
) ( d
2
1
) (
2
q Q q
q
q m
q q m = +

& & & . (a)


Vzhadom na to, e lohou je vypota rchlos v
1
, redukcia bude vykonan na teleso 1, t.j.
parametre v rovnici (a) s

1
x q = ,
1
v q = & ,
1
a q = & & ,
) ( ) (
1 1
*
x m q m
r
= , ) ( ) (
1 1
x F q Q
r
= .
Po dosaden do veobecnej pohybovej rovnice dostaneme
) (
d
) ( d
2
1
) (
1 1
2
1
1
1 1
1 1 1
x F v
x
x m
a x m
r
r
r
= + .
Redukcia hmotnosti sstavy telies na teleso 1:
Redukcia hmotnosti je zaloen na zklade rovnosti kinetickej energie redukovanej sstavy a
kinetickej energie pvodnej sstavy telies, t.j. plat

=
=
4
1
, 1 ,
i
i k r k
E E ,

2
4 4
2
3 3
2
2 2
2
1 1
2
1 1 1
2
1
2
1
2
1
2
1
) (
2
1
+ + + = I I I v m v x m
r
,
odkia

2
1
4
4
2
1
3
3
2
1
2
2
2
1
1
1 1 1
) (

=
v
I
v
I
v
I
v
v
m x m
r
. (b)
Podiely rchlost v ztvorkch v rovnici (b) s vyjadren zo vzjomnch pohybov telies:

2 2 1
R v =
2 1
2
1
R v
=

,

3 3 2 2
r R =
3 1
3
1
r v
=

, (c)

4 4 3 3
R R =
4 3
3
1
4
R r
R
v
=

.
Po dosaden tchto podielov do rovnice (b) je redukovan hmotnos na teleso 1

2
4 3
3
4
2
3
3
2
2
2 1 1 1
1 1
) (

+ =
R r
R
I
r
I
R
I m x m
r
. (d)
Redukcia silovch inkov sstavy telies na teleso 1:
Redukcia silovch inkov je zaloen na zklade rovnosti vkonu pracovnch sl
redukovanej sstavy a vkonu pracovnch sl pvodnej sstavy telies, t.j. plat

=
=
F
n
j
j p r p
P P
1
, 1 ,
. (e)
162
V rmci silovch inkov psobiacich na pvodn sstavu mono za pracovn sily
povaova hnac moment M a gravitan silu psobiacu na teleso 1. Rovnica (e) m potom
nasledujci tvar

1 1 4 1 1 1
) ( v G M v F + = x
r
,
resp. po vyjadren skalrnych sinov

1
1
1
1
4
1 1
) (
v
v
G
v
M x F
r

= . (f)
Po dosaden pomerov rchlost z rovnice (c) do rovnice (f) pre redukovan silu bude plati

1
4 3
3
1 1
) ( G
R r
R
M x F
r
= . (g)
Redukovan hmotnos
1 r
m a redukovan sila
1 r
F nie s zvisl od sradnice x
1
, t.j. plat

1 1 1 r r
F a m =
a po dosaden za
1 r
m a
1 r
F dostaneme

1
4 3
3
1
2
4 3
3
4
2
3
3
2
2
2 1
1 1
G
R r
R
M a
R r
R
I
r
I
R
I m =

+ .
Pre zrchlenie telesa 1 potom plat

2
4 3
3
4
2
3
3
2
2
2 1
1
4 3
3
1
1 1

=
R r
R
I
r
I
R
I m
G
R r
R
M
a .
Vetky leny na prvej strane s kontanty aj zrchlenie a
1
je kontantn. Z toho vyplva, e
sstava redukovan na teleso 1 vykonva rovnomerne zrchlen pohyb. Zvislos rchlosti v
1

od vkovej polohy x
1
je mon vyjadri nasledujcim postupom

1
1
1
1
1 1
1
1 1 1
1
dx
dv
v
dx
dv
dt
dx
dx
dx
dt
dv
dt
dv
a = = = = ,
odkia po separcii premennch a integrcii v prslunch hraniciach

1 1 1 1
dx a dv v =

=
1 1
0
1 1
0
1 1
x v
dx a dv v
1 1
0
1 1
2
1
1
2
x a dx a
v
x
= =

,
resp.

2
4 3
3
4
2
3
3
2
2
2 1
1
4 3
3
1
1 1 1
1 1
2
2

= =
R r
R
I
r
I
R
I m
G
R r
R
M x
x a v .

163
Prklad 3.48
Mechanizmus plantovej prevodovky sa rozbieha vplyvom kontantnho hnacieho momentu
M, ktor psob na una - teleso 1. Zkladn ozuben koleso s rozstupovm polomerom
R je nehybn. Una (1) m hmotnos m
1
rovnomerne rozloen po dke a moment
zotrvanosti je I
1
. Satelit (2) m polomer rozstupovej krunice r, hmotnos m
2
a moment
zotrvanosti I
2
. Zostavte pohybov rovnicu mechanizmu plantovej prevodovky metdou
redukcie na teleso 1. Vypotajte uhlov zrchlenie unaa (1) pre uveden spsob zaaenia.


Sstava bude rieen metdou redukcie, priom veobecn pohybov rovnica pre redukovan
sstavu m tvar
) (
d
) ( d
2
1
) (
2
q Q q
q
q m
q q m = +

& & & . (a)


Redukcia bude vykonan na teleso 1, ktor kon rotan pohyb a vpotov model
redukovanej sstavy je na nasledujcom obrzku.
Model redukovanej sstavy na teleso 1 Veliiny v rovnici (a):

1
= q
1
= q&
1
= q& &
) ( ) (
1 1
*
=
r
I q m
) ( ) (
1 1
=
r
M q Q
Po dosaden do veobecnej pohybovej rovnice dostaneme pohybov rovnicu pre redukciu na
teleso konajce rotan pohyb
) (
d
) ( d
2
1
) (
1 1
2
1
1
1 1
1 1 1
=

+
r
r
r
M
I
I . (b)
Redukcia hmotnosti sstavy telies na teleso 1:
Redukcia hmotnosti je zaloen na zklade rovnosti kinetickej energie redukovanej sstavy a
kinetickej energie pvodnej sstavy telies, t.j. plat
R

r

I
2
, m
2
I
1
, m
1

R

M

G
2
G
1

1
v
2
v
T1
T

2
M
r1
1

1
I
r1

1

164

=
=
2
1
, 1 ,
i
i k r k
E E .
Po dosaden dostaneme

2
2 2
2
2 2
2
1 1
2
1 1 1
2
1
2
1
2
1
) (
2
1
v m I I I
r
+ + = ,
odkia

2
1
2
2
2
1
2
2
2
1
1
1 1 1
) (

=
v
m I I I
r
. (c)
Podiely rchlost v ztvorkch v rovnici (c) vyjadren zo vzjomnch pohybov telies:

2 2 1
) ( v r r R = = +
r
r R +
=

1
2
,
) (
1 2
r R v + = r R
v
+ =

1
2
,

2
1 1
r R
v
T
+
=
2
1
1
r R v
T
+
=

.
Po dosaden tchto podielov rchlost do rovnice (c) je po prave redukovan moment
zotrvanosti sstavy na teleso 1

2
2
2
2 1 1
) ( r R m
r
r R
I I I
r
+ +

+
+ = . (d)
Redukcia silovch inkov sstavy telies na teleso 1:
Redukcia silovch inkov je realizovan pomocou rovnosti vkonu pracovnch sl
redukovanej sstavy a vkonu pracovnch sl pvodnej sstavy telies, t.j. plat

=
=
F
n
j
j p r p
P P
1
, 1 ,
. (e)
Na pvodn sstavu psobia nasledovn pracovn silov inky:
- hnac moment M,
- gravitan sila G
1
- psobiaca v aisku telesa 1,
- gravitan sila G
2
- psobiaca v aisku telesa 2.
Rovnica (e) m potom nasledujci tvar

2 2 1 1 1 1 1 1
) (
T T r
v G v G M M + + = .
Po vyjadren skalrnych sinov
) cos( ) cos( ) (
1 2 2 1 1 1 1 1 1 1
+ + = v G v G M M
T r

a nslednej prave je redukovan moment vyjadren v tvare

1
1
2
2 1
1
1
1
1
1
1 1
cos cos ) (

=
v
G
v
G M M
T
r
.
165
Po dosaden pomerov rchlost z rovnice (c) do rovnice (f) pre redukovan silu bude plati

1 2
1
1 1
cos ) (
2
) ( +

+ = r R g m
m
M M
r
. (f)
Redukovan moment zotrvanosti
1 r
I nie je zvisl od sradnice
1
, t.j. plat
) (
1 1 1 1
=
r r
M I ,
resp. po dosaden za
1 r
I a ) (
1 1

r
M je zskan pohybov rovnica sstavy redukovanej na teleso 1

1 2
1
1
2
2
2
2 1
cos ) (
2
) ( +

+ =

+ +

+
+ r R g m
m
M r R m
r
r R
I I .
Pre uhlov zrchlenie telesa 1, po vyjadren z tejto pohybovej rovnice potom plat

+ +

+
+
+

+
=
2
2
2
2 1
1 2
1
1
) (
cos ) (
2
r R m
r
r R
I I
r R g m
m
M
.

Prklad 3.49

Na bubon s polomerom r = 0,15 m je navinut lano v
ktorom psob ahov sila F = 250 N. S bubnom je
pevne spojen kladka s polomerom R = 0,25 m, na
ktorej je navinut lano, na konci ktorho je pripojen
bremeno s hmotnosou m
1
= 10 kg. Moment zotrvanosti
sstavy kladka + bubon je I
2
= 0,512 kgm
2
.
Pomocou metdy uvoovania vypotajte:
a - uhlov zrchlenie telesa 2,
b - silu F
1
v lane, na ktorom vis bremeno m
1
.
Vsledok:

2
= 11,412 rad.s
-2
F
1
= 126,63 N

Prklad 3.50

Vypotajte zrchlenie a
1
telesa m
1
a silu S
1
v
lane medzi telesami 1 a 2, ak sstavu telies v
uritom okamihu uvonme. Rozmerov a
hmotnostn parametre s zadan.
Vsledok:
) )( (
) ( ) (
3 3 3 3
2
1 1 2
2
3 3 3 3 3 3 3 3 1 2
2 1
R r m I r m I
R m I fm R r m I m
gr a
+

=
) )( (
) ( ) (
3 3 3 3
2
1 1 2
2
2 3 3
2
3 3 3 3 3 3 2
1 1
R r m I r m I
r m I R fm R r m I I
g m S
+
+ +
=
r
2
, I
2

R
3

r
3

I
3

m
1

f
m
1
r

R

F

I
2
166
Prklad 3.51
Metdou redukcie vypotajte zrchlenie a
5

pri zdvhan telesa 5. Dan hnac moment
M
h
psob na teleso 1. Rozmerov a
hmotnostn parametre sstavy s zadan.
Vsledok:
2
2
3 3
2
2
2
2
1
1
2
4
2 1
5
5
4 1
2 1
5
r
I I
r
I
r
I
r
r r
m
g m
r r
r r
M
a
h
+
+

+
+

+
=

Prklad 3.52
Ty AB na obrzku rotuje okolo zvislej osi uhlovou rchlosou = 15 rad/s. V bode C je
kbovo pripojen thla ty, ktor m kontantn prierez po celej dke a jej hmotnos
m
1
= 2 kg. Na druh koniec (bod E) thlej tye je pripojen gua s polomerom r a
hmotnosou m
1
= 4 kg. Pomocou lana je thla ty dran v zvislej polohe.
Vypotajte:
a - silu v lane,
b - silu v kbe C.
Vsledok:
a - sila v lane = 781,4 N
b - 51 , 115 =
x
C N
62 , 268 =
y
C
4 , 292 = C N





l
1
l
2
h
2
h
1
r

A

C

D

E


B

m
5
I
4
r
4
r
2
r
1
I
2
, m
2

I
3
I
1
M
h
167
3.6 ANALYTICK DYNAMIKA
Princp virtulnych prc
Virtulny pohyb - myslen mon pohyb sstavy okolo polohy, ktor vo veobecnom
okamihu me sstava nadobudn a ktor dovouj vzby. Virtulny pohyb sa odohrva
nezvisle od asu, t.j. virtulny pohyb je uvaovan ako izochrnny.
Veobencn formulcia princpu virtulnych prc v dynamike - mechanick sstava sa
pohybuje tak, e set virtulnych prc pracovnch a zotrvanch sl je v kadom okamihu
rovn nule
0 ) (
,
= + =

i
i Di
p
i D p
A r F F , (3.41)
kde
D p
A
,
- virtulna prca,

p
i
F - vonkajie pracovn sily,

Di
F - dAlembertove zotrvan sily,

i
r - virtulne posunutie.
Poznmka: Virtulna prca vzbovch sl
v
i
F je v kadom okamihu rovn nule, t.j.
0 =

i
i
v
i
r F . (3.42)
Lagrangeove rovnice druhho druhu
Sstavy s jednm stupom vonosti
Q
q
E
q
E
q
E
dt
d p
k k
=

&
, (3.43)
kde
k
E - kinetick energia sstavy,

p
E - potencilna energia sstavy,
q - zoveobecnen sradnica,
q& - zoveobecnen rchlos
Q - zoveobecnen sila,
Sstavy s n stupami vonosti

,
,
1
1 1 1
n
n
p
n
k
n
k
p
k k
Q
q
E
q
E
q
E
dt
d
Q
q
E
q
E
q
E
dt
d
=

&
M
&
(3.44)
kde
j
q - zoveobecnen sradnica pohybu j-teho telesa,

j
q& - zoveobecnen rchlos pohybu j-teho telesa,

j
Q - zoveobecnen sila psobiaca na j-te teleso.
168
Zoveobecnen sila - vyjadruje rovnos virtulnych prc pracovnch sl sstavy a virtulnej
hadanej zoveobecnenej sily, t.j. plat




+

=
i
j
i p
i
i
j
i p
zi
j
i p
yi
j
i p
xi j
q
M
q
z
F
q
y
F
q
x
F Q , (3.45)
kde
p
zi
p
yi
p
xi
F F F , , , resp.
p
i
M - zloky pracovnch sl, resp. momentov.
V prpade, e s vetky pracovn sily iba potencilne, t.j. plat

q
E
Q
p

= ,

(3.46)
maj Lagrangeove rovnice druhho druhu tvar
0 =

q
L
q
L
dt
d
&
, (3.47)
kde
p k
E E L = - kinetick potencil.
V prpade sstavy, v ktorej s zabudovan kontrukn prvky s disipatvnymi (tlmiacimi)
vlastnosami, je mon poui Lagrangeove rovnice v tvare
Q
q
E
q
E
q
E
q
E
dt
d p
d k k
=

& &
, (3.48)
kde
2
2
1
q b E
d
& = je disipatvna funkcia a b je sinite linerneho viskzneho tlmenia.

Prklad 3.53
Pomocou pouitia princpu virtulnych prc zostavte pohybov rovnicu homognnej tye s
hmotnosou m a dkou 2l, ktor je ahan lanom. Hmotnos lana, kladky a pasvne odpory
zanedbajte.



Medzi pracovn sily v tejto sstave patria - sila F, gravitan sila G a zotrvan moment M
D
.
Poda princpu virtulnych prc mus veobecne plati
0 ) (
,
= + =

i
i Di
p
i D p
A r F F . (a)
l
l
T
m, I
h
F


..

.
G
y

M
D
l
l
T
m, I
h
F
169
Pre dan sstavu potom poda princpu virtulnych prc plat
0 =
D
M y G F , (b)
kde = & & I M
D
je vekos dAlembertovho zotrvanho momentu.
Za zoveobecnen sradnicu je povaovan uhol . Na zklade toho mono z geometrie
zostavi nasledovn zvislosti
- poloha aiska vzhadom na horizontlnu os
= sin l y = ) cos (l y , (c)
- zmena dky lana je uren pomocou cosnusovej vety

+ =
2
cos 4 4
2 2 2
hl l h ,
resp.
+ = sin 4 4
2 2 2
hl l h

= cos
2hl
. (d)
Po dosaden (c) a (d) do rovnice (b) a nslednej prave
0 cos cos
2
=

& & I mgl


hl
F .
Tto rovnica mus by splnen pre ubovon , a preto vraz v ztvorke mus by rovn
nule, t.j. plat
0 cos
sin 4 4
2
cos
2 2
=
+
+
hl l h
hl
F mgl I & & ,
o je hadan pohybov rovnica.

Prklad 3.54
Zdvhacie zariadenie je pohan kontantnm momentom M. Vypotajte zrchlenie a
1

bremena m
1
. lohu riete aplikovanm princpu virtulnych prc, ako aj Lagrangeovch
rovnc II. druhu.


I
2
m
1
R
3
r
2
R
2
I
3
M

2
G
1

2
v
1
y
1
I
2
m
1
R
3
r
2
R
2
I
3
M

170
Zostavenie pohybovej rovnice pomocou princpu virtulnych prc
Poda princpu virtulnych prc mus veobecne plati
0 ) (
,
= + =

i
i Di
p
i D p
A r F F . (a1)
Medzi pracovn silov inky
P
i
F v tejto sstave patria - hnac moment M, gravitan sila G
1
,
zotrvan sila F
D1
a zotrvan momenty M
D2
a M
D3
.
Pre dan sstavu potom poda princpu virtulnych prc plat
0
1 1 3 3 2 2 1 1 3
= y F M M y G M
D D D
, (b1)
kde
2 2 2
= & & I M
D
- vekos zotrvanho momentu psobiaceho na teleso 2, (c1)

3 3 3
= & & I M
D
- vekos zotrvanho momentu psobiaceho na teleso 3, (d1)

1 1 1
y m F
D
& & = - vekos zotrvanej sily psobiacej na teleso 1, (e1)
g m G
1 1
= - vekos gravitanej sily psobiacej na teleso 1. (f1)
Za zoveobecnen sradnicu je povaovan posunutie
1
y . Na zklade tejto zoveobecnenej
sradnice mono zo vzjomnch pohybov telies skmanej sstavy zostavi nasledovn
kinematick zvislosti:
- vyjadrenie natoenia ) (
2
telesa 2 vzhadom na posunutie ) (
1
y telesa 1

2 2 1
= r y
1
2
2
1
y
r
=
1
2
2
1
y
r
= , (g1)
- vyjadrenie natoenia ) (
3
telesa 3 vzhadom na posunutie ) (
1
y telesa 1

3 3 2 2
= R R
1
2 3
2
2
3
2
3
y
r R
R
R
R
= =
1
2 3
2
3
y
r R
R
= . (h1)
Zrchlenia
2
& & a
3
& & s vyjadren pomocou
1
y& & nasedovne:

1
2
2
1
y
r
& & & & = , (i1)

1
2 3
2
3
y
r R
R
& & & & = . (j1)
Po dosaden vzahov (d1-k1) do rovnice (b1) a nslednej prave je vyjadrenie rovnice pre
princp virtulnych prc
0
1
1 1 1
2
2 3
2
3
2
2
2 1 1
2 3
2
=

y y m
r R
R
I
r
I g m
r R
R
M & & .
Na zklade tejto rovnice m pohybov rovnica tvar

1 1
2 3
2
1 1
2
2 3
2
3
2
2
2
1
g m
r R
R
M y m
r R
R
I
r
I =

& & . (k1)


171
Zostavenie pohybovej rovnice pomocou Lagrangeovch rovnc II. druhu
Veobecn vyjadrenie Lagrangeovch rovnc II. druhu je
Q
q
E
q
E
q
E
dt
d p
k k
=

&
. (a2)
Pre rieen sstavu plat:

1
y , (
1
y& ,
1
y& & ) - zoveobecnen sradnica (rchlos, zrchlenie).
Vyjadrenie Lagrangeovch rovnc II. druhu m potom tvar
Q
y
E
y
E
y
E
dt
d p
k k
=

1 1 1
&
. (b2)
Jednotliv leny v rovnici (a2) s vyjadren nasledovne:
- kinetick energia sstavy

2
3 3
2
2 2
2
1 1
2
1
2
1
2
1
+ + = & & & I I y m E
k
, (c2)
- potencilna energia sstavy

1 1
gy m E
p
= , (d2)
- zoveobecnen sila

1
3
y
M Q


= . (e2)
Zvislosti rchlost telies 2 a 3 od rchlosti
1
y&

1
2
2
1
y
r
& & = ,
1
2 3
2
3
y
r R
R
& & = . (f1)
Po dosaden prslunch vzahov:
- kinetick energia sstavy

2
1
2
2 3
2
3
2
2
2 1
1
2
1
y
r R
R
I
r
I m E
k
&

+ = ,
- potencilna energia sstavy

1 1
gy m E
p
= ,
- zoveobecnen sila

2 3
2
r R
R
M Q = .
Po dosaden do (b2) je zskan pohybov rovnica

1 1
2 3
2
1 1
2
2 3
2
3
2
2
2
1
g m
r R
R
M y m
r R
R
I
r
I =

& & ,
ktor m rovnak tvar ako rovnica zskan pomocou princpu virtulnych prc.
172
3.7 LINERNE KMITANIE
Sstava s jednm stupom vonosti


Vpotov model Schma vpotovho modelu
Pohybov rovnica jednohmotnej sstavy
) ( ) ( ) ( t u b t ku t F kx x b x m & & & & + + = + + , (3.49)
kde m - hmotnos,
k - tuhos pruiny,
b - sinite linerneho tlmenia (tlmi),
F(t) - budiaca sila,
u(t) - budiaca vchylka zkladu,
x(t) - vchylka jednohmotnej sstavy.
Rozdelenie kmitania sstavy s jednm stupom vonosti
0 ) ( = t F 0 ) ( = t u - von kmitanie,
0 ) ( t F 0 ) ( = t u - vynten kmitanie so silovm budenm,
0 ) ( = t F 0 ) ( t u - vynten kmitanie s kinematickm budenm,
0 ) ( t F 0 ) ( t u - vynten kmitanie s kombinovanm budenm,
0 = b - netlmen kmitanie,
0 b - tlmen kmitanie.
Von netlmen kmitanie
0 = + kx x m& & , resp. 0
2
0
= + x x& & . (3.50)
Charakteristick rovnica
0
2
0
2
= + , korene
0 2 , 1
= i . (3.51)
Rieenie
) sin( ) sin( ) cos(
1 0 1 0 1 0 1
+ = + = t C t B t A x . (3.52)
Von tlmen kmitanie
0 = + + kx x b x m & & & , resp. 0 2
2
0
= + + x x x & & & . (3.53)
F(t)
u(t)
m
b
k
m
b
k
F(t)
x,
.
x,
..
x,
u(t)
173
Charakteristick rovnica
0 2
2
0
2
= + + . (3.54)
Korene charakteristickej rovnice

2
0
2
2 , 1
= . (3.55)
Rieenie pre
0
< (t.j.
d
i i = =
2 2
0 2 , 1
)
[ ] ) sin( ) sin( ) cos(
2 2 2 2
+ = + =

t e C t B t A e x
d
t
d d
t
, (3.56)
kde m b 2 = - pomern tlmenie, m k =
0
- vlastn kruhov frekvencia netlmenho
kmitania,
d
- vlastn kruhov frekvencia tlmenho kmitania.
Vynten netlmen kmitanie s harmonickou budiacou silou ) sin( ) (
0
t F t F =
) sin( ) (
0
t F t F kx x m = = + & & , (3.57)
kde
0
F - amplitda budiacej sily, - kruhov frekvencia budiacej sily.
Rieenie
) sin(
) (
) sin( ) cos(
2 2
0
0
0 1 0 1
t
m
F
t B t A x x x
p h


+ + = + = . (3.58)
Rezonancia nastva v prpade:
0
= .
Vynten tlmen kmitanie s harmonickou budiacou silou ) sin( ) (
0
t F t F =
) sin( ) (
0
t F t F kx x b x m = = + + & & & . (3.59)
Rieenie

[ ]
.
2
arctan sin
4 ) (

) sin( ) cos(
2 2
0
2 2 2 2 2
0
0
0 2 0 2




+
+
+ = + =

t
m
F
t B t A e x x x
t
p h
(3.60)
Rezonancia nastva v prpade:
2
0
2 1 = .
Vynten tlmen kmitanie s budiacou silou ) (t F - veobecn asov funkcia.
) (t F kx x b x m = + + & & & . (3.61)
Rieenie

[ ]
. )) ( sin( ) (
1

) sin( ) cos(
0
) (
1 1

+
+ = + =


t
d
t
d
d d
t
p h
d t e F
m
t B t A e x x x
(3.62)
Kontanty A
i
, B
i
; resp.
2 2
i i i
B A C + = , ) arctan(
i i i
B A = s integran kontanty (i = 1,2),
ktor je mon uri zo zaiatonch podmienok (napr.
0 0
) 0 ( ; ) 0 ( v x x x = = & ).
174
Tuhosti prunch elementov
Model Ekvivalentn tuhos

=
=
n
i
i
eq
k
k
1
1
1

=
=
n
i
i eq
k k
1


Pozdne kmitajca ty
l
ES
k
eq
=
E - Youngov modul prunosti v ahu
S - prieny prierez tye

Torzne kmitajca ty
l
GJ
k
eq
=
G - modul prunosti v myku
J - polrny moment prierezovej plochy


3
4
64nR
Gd
k
eq
=
G - modul prunosti v myku
d - priemer drtu pruiny
R - polomer vinutia pruiny
n - poet zvitov

3
3
l
EJ
k
eq
=
J - kvadratick moment prierezovej plochy


2 2
) ( 3
b a
b a EJ
k
eq
+
=

) 4 3 (
) ( 12
2 3
3
b a b a
b a EJ
k
eq
+
+
=

3 3
3
) ( 3
b a
b a EJ
k
eq
+
=


a
m
b
a
m
b
a
m
b
l
m
R
d
I
l
m
l
k
n
m

k
1
k
1 k
n
m

175
Prklad 3.55
Vypotajte dobu kyvu homognnej guky s hmotnosou m a polomerom r, ktor sa
pohybuje po kruhovej drhe s polomerom R. Predpoklad sa, e sa guka po kruhovej
drhe odvauje, t.j. nedochdza k jej premykovaniu.



Dobu kyvu je mon vypota z pohybovej rovnice guky. Pohybov rovnica me by
zostaven rznymi spsobmi metda uvoovania, metda redukcie a Lagrangeove rovnice
II. druhu.
Na zostavenie pohybovej rovnice bud na rieenie tohto prkladu pouit Lagrangeove
rovnice II. druhu. Zkladn vchodiskov rovnica m tvar
0 =

q
L
q
L
dt
d
&
, (a)
kde - kinetick energia
2 2
2
1
2
1
T T k
mv I E + = ,
- potencilna energia ) cos 1 )( ( = r R mg E
p
,
- kinetick potencil ) cos 1 )( (
2
1
2
1
2 2
+ = = r R mg mv I E E L
T T p k
.
Moment zotrvanosti guky vzhadom na aisko

2
5
2
mr I
T
= .
Ako zoveobecnen sradnica me by pouit sradnica . Lagrangeova rovnica m po
zabudovan zoveobecnenej sradnice tvar
0 =


L L
dt
d
&
. (b)
Vzjomn vzah medzi uhlami a je zaloen na rovnosti dky drhy, ktor vznikne pri
odvaovan, t.j.
= r R =
r
R

R
r
m



v
T
R
r
m


176
a pre vzahy medzi rchlosami plat

T T
r r v = =
& & ) ( r R r r v
T
= = = & & ,

= = & &
r
r R
.
Po dosaden do kinetickho potencilu dostaneme
) cos 1 )( ( ) (
2
1
2 2
2


= r R mg r R m
r
r R
I L
T
& . (c)
Vpoet jednotlivch derivci pre rovnicu (b)

&
&
2
2
) ( r R m
r
r R
I
L
T
,

& &
&
2
2
) ( r R m
r
r R
I
L
dt
d
T
, (d)
=

sin ) ( r R mg
L
.
Po dosaden m pohybov rovnica tvar
0 sin ) ( ) (
2
2
= +


r R mg r R m
r
r R
I
T
& & . (e)
Vzhadom na predpoklad, e pre mal uhly , t.j.
o
5 < plat sin (pre v radinoch).
Linearizovan pohybov rovnica m potom tvar
0 ) ( ) (
2
2
= +


r R mg r R m
r
r R
I
T
& & ,
resp. po prave
0
) )( (
2
2
=
+
+
r R mr I
mgr
T
& & ,
resp.
0
2
0
= + & & . (f)
Vlastn kruhov frekvencia

) )( (
2
2
0
r R mr I
mgr
T
+
=
a doba kyvu je vypotan zo vzahu

2
2
0
0
) )( (
2
2
mgr
r R mr I
T
T
+
=

= .
177
Prklad 3.56
Jednohmotn sstava s hmotnosou m = 2 kg a jednm stupom vonosti je k zkladu
pripojen pomocou troch prun k
1
= 2 kN/m, k
2
= 3 kN/m, k
3
= 2 kN/m. Zaiaton
podmienky s: t = 0 s, x(0) = x
0
= 0,005 m, 25 , 0 ) 0 (
0
= = v x& m/s.
Vypotajte:
a - vlastn kruhov frekvenciu sstavy,
b - peridu vlastnho kmitania,
c - asov zvislos vchylky x(t) hmoty m.

Obr. A

Obr. B

Pre vpotov model (obr.B) je metdou zrchujcich sl zostaven pohybov rovnica

3 2 1 k k k
F F F x m = & & , (a)
kde pre sily v pruinch plat
x k F
k 1 1
= , x k F
k 2 2
= , x k F
k 3 3
= .
Po dosaden do pohybovej rovnice
0 ) (
3 2 1
= + + + x k k k x m& & , (b)
resp.
0 = + x k x m
eq
& & , resp. 0
2
0
= + x x& & . (c)
Vlastn kruhov frekvencia
16 , 59 2
3 2 1
0 0
=
+ +
= = =
m
k k k
m
k
f
eq
rad/s.
Vlastn frekvencia
42 , 9
2
0
0
=

= f Hz.
Perida vlastnho kmitania
106 , 0
2 1
0 0
0
=

= =
f
T s.
Rieenm diferencilnej rovnice (c) je
) sin( ) sin( ) cos( ) (
1 0 1 0 1 0 1
+ = + = t C t B t A t x , (d)
priom rchlos pohybu hmoty m je
) cos( ) cos( ) sin( ) (
1 0 0 1 0 0 1 0 0 1
+ = + = t C t B t A t x& .
F
k1
F
k2
F
k3
x, x, x

. ..
178
Integran kontanty v rovnici s uren zo zadanch zaiatonch podmienok, t.j.

1 0
) 0 ( A x x = =
0 1
x A = ,

0 1 0
) 0 ( = = B v x&
0
0
1

=
v
B .
Po dosaden za integran kontanty do (d) dostaneme

+ =

+ =
0
0 0
0
2
0
2
0 2
0 0
0
0
0 0
v
x
t
v
x t
v
t x t x arctg sin ) sin( ) cos( ) ( ,
resp.
) 8691 , 0 16 , 59 sin( 006547 , 0 ) ( + = t t x .
asov priebeh amplitdy je na nasledujcom obrzku.
0,0 0,1 0,2 0,3 0,4 0,5
-0,008
-0,004
0,000
0,004
0,008
a
m
p
l
i
t

d
a

C
1



[
m
]
as [s]

Prklad 3.57
Jednohmotn sstava s hmotnosou m a jednm stupom vonosti je spojen so systmom
prun troch prun k
1
, k
2
, k
3
poda obr. A. Vypotajte vlastn kruhov frekvenciu sstavy,
peridu vlastnho kmitania sstavy.

Obr. A


Obr. B

Pre vpotov model sstavy (obr.B) je pomocou metdy zrchujcich sl zostaven
pohybov rovnica

3 2 k k
F F x m = & & (a)
F
k1
x
A
, x
A
, x
A
. ..
F
k2
A

k
3
k
1
k
2
A

F
k3
F
k2
x, x, x

. ..
179
a pre bod A plat rovnovha sl psobiacich v pruinch

1 2 k k
F F = , (b)
kde pre sily v pruinch plat

A k
x k F
1 1
= , ) (
2 2 A k
x x k F = , x k F
k 3 3
= . (c)
Po dosaden za
1 k
F a
2 k
F do rovnice

A A
x k x x k
1 2
) ( = x
k k
k
x
A
2 1
2
+
= . (d)
Po dosaden z (c) a (d) do pohybovej rovnice (a) dostaneme
0
3
2 1
2
2
= +

+
+ x k x
k k
k
x k x m& & , (b)
resp. po prave
0
) (
2 1
3 2 3 2 1
=
+
+ +
+ x
k k
k k k k k
x m& & 0 = + x k x m
eq
& & 0
2
0
= + x x& & , (c)
kde
2 1
3 2 3 2 1
1 1
3 2 , 1 3
) ( 1
2 1
k k
k k k k k
k k k k
k k
eq
+
+ +
=
+
+ = + = .
Vlastn kruhov frekvencia sstavy

) (
) (
2 1
3 2 3 2 1
0
k k m
k k k k k
m
k
eq
+
+ +
= = .

Prklad 3.58
Vypotajte vlastn kruhov frekvenciu systmu. Hmotnosti kladiek a pasvne odpory s
zanedban.

Obr. A

Obr. B

Obr. C

Pre vpotov model sstavy (obr.B) je pohybov rovnica

2 1
2
k k
F F x m = & & , (a)
x
2x
OSO
m
x, x, x

. ..
F
k1
F
k1
F
k2
m
k
2
x, x, x

. ..
k
1
180
kde pre sily v pruinch plat
x k x k F
k 1 1 1 1
= = , x k x k F
k 2 2 2 2
2 = = . (b)
Po dosaden za
1 k
F a
2 k
F do pohybovej rovnice (a)
0 ) 4 (
2 1
= + + x k k x m& & 0 = + x k x m
eq
& & , (c)
resp. 0
2
0
= + x x& & . (d)
Vlastn kruhov frekvencia sstavy

m
k k
m
k
eq
2 1
0
4 +
= = .

Prklad 3.59
Vypotajte dobu kmitu mechanickho systmu na obrzku. Teleso sa val po vodorovnej
ploche. Pohybov rovnicu zostavte:
a - metdou uvoovania,
b - metdou redukcie,
c - Lagrangeovmi rovnicami II. druhu.

Obr. A

Obr. B

Obr. C

Obr. D

Metda uvoovania
Pohybov rovnica pre vpotov model sstavy (obr. B):
- posuvn pohyb telesa (aisko je v bode A)

T k k A
F F F x m =
2 1
& & , (a1)
- rotan pohyb telesa okolo aiska

T k k A
RF rF rF I + + =
2 1
& & , (b1)
kde F
T
je trecia sila.
m
rA
..

x
A
, x
A
, x
A

.

F
rA
k
rA
m
rA
..

x
A
, x
A
, x
A

.

F
k1
I, m

r
R
F
k2
x
1
x
A
x
2
A


..

.

x
A
.

x
A
..

F
T
G

k
1
k
2
I, m

R
r
181
Sily v pruinch

1 1 1
x k F
k
= ,
2 2 2
x k F
k
= . (c1)
Vzahy medzi posuvmi
1
x ,
2
x a
A
x

r R
x
R
x
r R
x
A

= =
+
2 1

A
x
R
r R
x
+
=
1
,
A
x
R
r R
x

=
2
. (d1)
Vzah medzi zrchleniami
A
x& & a & &
R x
A
= & & & & . (e1)
Po dosaden (c) a (d) do rovnc (a) a (b) a ich nslednej prave dostaneme pohybov rovnicu
0
) ( ) (
2
2
2
2
1
2
=

+ +
+

+
A A
x
R
r R k r R k
x
R
I
m & & . (f1)
Metda redukcie
Redukcia sstavy (obr. A) bude uroben na bod A (obr. D), priom veobecn pohybov
rovnica sstavy po redukcii
) (
d
) ( d
2
1
) (
2
q Q q
q
q m
q q m = +

& & & . (a2)


Parametre v rovnici (a2) s

A
x q = ,
A
x q & & = ,
A
x q & & & & = ,
) ( ) (
*
A rA
x m q m = , ) ( ) (
A rA
x F q Q = .
Po dosaden do veobecnej pohybovej rovnice dostaneme
) (
d
) ( d
2
1
) (
2 1
A rA A
A
A r
A A rA
x F x
x
x m
x x m = + & & & . (b2)
Redukcia hmotnosti na bod A:
Pre redukciu hmotnosti na bod A plat

=
i
i k rA k
E E
, ,
.
Po dosaden dostaneme

2 2 2
2
1
2
1
) (
2
1
A A A rA
x m I x x m & & & + = m
x
I m
A
rA
+


=
2
&
&
,
odkia po dosaden za R x
A
= & & dostaneme redukovan hmotnos
m
R
I
m
rA
+ =
2
. (c2)
Redukcia silovch inkov na bod A:
Pre redukciu silovch inkov na bod A plat

=
=
F
n
j
j p rA p
P P
1
, ,
. (d2)
182
Rovnica (c2) m po dosaden tvar

2 2 1 1
) ( x F x F x F
& & &
+ =
k k A A rA
x .
Po vyjadren skalrnych sinov

2 2 1 1
) ( x F x F x x F
k k A A rA
& & & =
A
k
A
k A rA
x
x
F
x
x
F x F
&
&
&
&
2
2
1
1
) ( = .
Po dosaden vzahov (c1) a (d1) a po prave m redukovan sila tvar

R
r R
x k
R
r R
x k x k F
A rA rA

+
= =
2 2 1 1
,
resp. tuhos redukovanej pruiny

2
2
2
1

+
=
R
r R
k
R
r R
k k
rA
. (e2)
Po dosaden (c2) a (d2) do pohybovej rovnice (b2) je pohybov rovnica redukovanho
systmu (obr. C)
0
) ( ) (
2
2
2
2
1
2
=

+ +
+

+
A A
x
R
r R k r R k
x
R
I
m & & . (f2)
Lagrangeove rovnice II. druhu
Na zostavenie pohybovej rovnice sstavy s pouit Lagrangeove rovnice II. druhu v tvare
0 =

q
L
q
L
dt
d
&
, (a3)
kde - kinetick energia
2 2
2
1
2
1
A k
x m I E & & + = ,
- potencilna energia
2
2 2
2
1 1
2
1
2
1
x k x k E
p
+ = ,
- kinetick potencil
2
2 2
2
1 1
2 2
2
1
2
1
2
1
2
1
x k x k x m I E E L
A p k
+ = = & & .
Ako zoveobecnen sradnica je pouit sradnica
A
x . Lagrangeove rovnice maj potom tvar
0 =

A A
x
L
x
L
dt
d
&
.
Pouitm vzahov (d1) je kinetick potencil vyjadren v tvare

2
2
2
2
2
1 2
2
) ( ) (
2
1
2
1
A A
x
R
r R k r R k
x m
R
I
L

+ +

+ = & (b3)
a po jeho dosaden do rovnice (a3) m pohybov rovnica sstavy tvar
0
) ( ) (
2
2
2
2
1
2
=

+ +
+

+
A A
x
R
r R k r R k
x
R
I
m & & . (c3)
183
Z pohybovej rovnice, ktor bola zostaven rznymi spsobmi (f1, f2, c3), je po prave
0
) ( ) (
2
2
2
2
1
=

+
+ +
+
A A
x
I mR
r R k r R k
x& & ,
vyjadren vlastn kruhov frekvencia v tvare

I mR
r R k r R k
+
+ +
=
2
2
2
2
1
0
) ( ) (
.

Prklad 3.60
Kmitajca sstava m jeden stupe vonosti a je zloen z kladky s polomerom R = 0,5 m,
momentom zotrvanosti I = 0,08 kgm
2
, na ktor je pevne pripojen homognna ty s
kontantnm prierezom, dky l = 0,8 m a hmotnosti m
0
= 0,9 kg. Na sstavu s pripojen
dve pruiny k
1
= 5,5 kN/m a k
2
= 6 kN/m a tlmi, ktorho sinite linerneho tlmenia je
b = 12 Ns/m. Zaiaton podmienky s: t = 0 s, (0) =
0
= 0 rad, 2 ) 0 (
0
= = & rad/s.
Vypotajte:
a - vlastn kruhov frekvenciu netlmenej a tlmenej sstavy,
b - peridu vlastnho kmitania netlmenej a tlmenej sstavy,
c - asov zvislos natoenia (t) systmu.

Obr. A Obr. B

Pre kmitajci systm s jednm stupum vonosti (obr. A), ktor kon rotan pohyb, je
pohybov rovnica zostaven pomocou metdy zrchujcich sl

b k k
F R l RF F R l I ) ( ) (
2 1 0
+ + = & & , (a)
kde - sily v pruinch s
1 1 1
x k F
k
= ,
2 2 2
x k F
k
= ,
- sila v tlmii
1
x b F
b
& = ,
- moment zotrvanosti systmu vzhadom na jeho os rotcie

p
I I I + =
0
,
kde I - moment zotrvanosti kladky vzhadom na os rotcie systmu,
I
p
- moment zotrvanosti pky k osi rotcie sytmu.
184
l + R
R
r
dr
m
0
dm
Vpoet momentu zotrvanosti pky k osi rotcie
systmu:

+
= =
R l
R m
p
dr r
l
m
dm r I
2 0
) (
2
,
3
) (
3 3
0
R R l
l
m
I
p
+
= ,
resp.
777 , 0 ) 3 3 (
3
2 2 0
= + + = R lR l
m
I
p
[kgm
2
] (b)

Vyjadrenie posunut
1
x a
2
x v zvislosti od natoenia systmu

R
x
R l
x
2 1
sin =
+
= ,
kde pre hodnoty uhlov
o
5 < plat
sin + = ) (
1
R l x , = R x
2
, kde je v radinoch. (c)
Pre rchlos bodu, v ktorom je pripojen tlmi, plat
+ = & & ) (
1
R l x . (d)
Sily v pruinch a v tlmii potom mono vyjadri v tvare
+ = ) (
1 1
R l k F
k
, = R k F
k 2 2
, + = & ) ( R l b F
b
(e)
a po ich dosaden do pohybovej rovnice a jej nslednej prave dostaneme
[ ] [ ] 0 ) ( ) ( ) 3 3 (
3
2
2
2
1
2 2 2 0
= + + + + +

+ + + R k R l k R l b R lR l
m
I & & & ,
resp. 0 2
2
0
= + + & & & , (f)
- pomern tlm:
832 , 11
)] 3 3 ( 3 [ 2
) ( 3
2 2
0
2
=
+ + +
+
=
R lR l m I
R l b
s
-1

- vlastn kruhov frekvencia netlmenho kmitania:
23 , 112
) 3 3 ( 3
] ) ( [ 3
2 2
0
2
2
2
1
0
=
+ + +
+ +
=
R lR l m I
R k R l k
rad/s,
- vlastn kruhov frekvencia tlmenho kmitania:
61 , 111
2 2
0
= =
d
rad/s,
- perida vlastnho kmitania netlmenej sstavy:
=

=
0
0
2
T 0,0560 s,
185
- perida vlastnho kmitania tlmenej sstavy:
=

=
d
d
T
2
0,0563 s.
Rieenie diferencilnej rovnice (f) m tvar

+
+ =

0 0
0
2
0 0 2
0
&
&
d
d
d
t
t e t arctg sin ) ( .
asov priebeh natoenia vone kmitajceho systmu je na nasledujcom obrzku.
0,0 0,1 0,2 0,3
-0,03
-0,02
-0,01
0,00
0,01
0,02
0,03
a
m
p
l
i
t

d
a




[
r
a
d
]
as [s]


Prklad 3.61
Vypotajte dobu kmitu mechanickho systmu pozostvajceho z dvoch ozubench kolies
s momentami zotrvanosti I
1
a I
2
, hmotnosami m
1
a m
2
s rovnakm polomerom R
rozstupovej krunice. Na prav ozuben koleso je pevne pripojen homognna ty s
hmotnosou m
p
a dkou l a kontantnm prierezom po celej dke.



Na zostavenie pohybovej rovnice sstavy je pouit Lagrangeova rovnica II. druhu v tvare pre
sstavu s jednm stupom vonosti
I
1
, m
1

2
m
p
l

I
2
, m
2

R

R

T

I
1
, m
1



m
p
l

I
2
, m
2

R
R

186
0 =

q
L
q
L
dt
d
&
, (a)
kde - kinetick energia sstavy

2 2
2 2
2
1 1
2
1
2
1
2
1
p p k
I I I E + + = & & & ,
- potencilna energia sstavy - nulov potencilna hladina pre sstavu je uvaovan
v rovni najniej polohy aiska T rotujcej tye
) cos (
p p p
l
g m E = 1
2
,
- kinetick potencil
) cos 1 (
2 2
1
2
1
2
1
2 2
2 2
2
1 1 p p p p p k
l
g m I I I E E L + + = = & & & , (b)
kde
2
3
1
l m I
p p
= - moment zotrvanosti vzhadom na os rotcie tye.
Ak je ako zoveobecnen sradnica pouit uhol natoenia
2
telesa 2, potom
0
2 2
=


L L
dt
d
&
. (c)
Kinematick vzahy platn medzi jednotlivmi telesami
R R
2 1
=
2 1
=
2 1
= & &
2 1
= & & & & (d)
a zrove plat

2
=
p

2
= & &
p

2
= & & & &
p
. (e)
Pouitm vzahov (e,d) je kinetick potencil vyjadren v tvare
) cos 1 (
2 2
) ( ) (
2
1
2 2 1
2
2 2 1
+ + + =
l
g m
l
g m m I I I L
p p
&
a po jeho dosaden do rovnice (c) m pohybov rovnica sstavy tvar
0 sin
2
) (
2
2
2 2 1
= + + +
l
g m I I I
p p
& . (f)
Rovnica (f) je vzhadom na druh len ) (sin
2
nelinerna. Uveden diferencilnu rovnicu je
mon linearizova s ohadom na predpoklad, e pre mal uhly
2
, t.j.
o
5
2
< plat
2 2
sin (pre
2
v radinoch). Linearizovan pohybov rovnica m potom tvar
0
2
) (
2
2
2 2 1
= + + +
l
g m I I I
p p
& ,
odkia meme po prave psa
0
) ( 2
2
2 1
2
2
=
+ +
+
p
p
I I I
gl m
& ,
priom vlastn kruhov frekvenciu je mon vypota zo vzahu

) ( 2
2 1
0
p
p
I I I
gl m
+ +
= ,
187
doba kmitu tohto systmu je potom

gl m
I I I
T
p
p
) ( 2
2
2 2 1
0
0
+ +
=

= .

Prklad 3.62
Rotujca homognna ty s hmotnosou m a dkou l
1
+ l
2
, ktor m na vonom konci
pripojen hmotn bod s hmotnosou m
0
, je uloen na sstave pruina-tlmi (k-b). Na
hmotn bod psob harmonick budiaca sila ) sin(
0
t F F = , kde je kruhov frekvencia
budiacej sily a
0
F je amplitda budiacej sily. Jednotliv parametre s:
m = 1,0 kg m
0
= 0,3 kg k = 24,0 kN/m b = 60,0 Ns/m
l
1
= 0,4 m l
2
= 0,3 m F
0
= 35,0 N = 20,0 rad/s
Zaiaton podmienky: t = 0 s (0) =
0
= 0,01 rad
= =
0
) 0 ( & & 2,0 rad/s
Vypotajte:
a - vlastn kruhov frekvenciu netlmenej a tlmenej sstavy,
b - peridu vlastnho kmitania netlmenej a tlmenej sstavy,
c - asov zvislos natoenia (t) tye,
d - rezonann frekvenciu,
e - zostrojte amplitdovo-frekvenn charakteristiku a fzovo-frekvenn charakteristiku.

Obr. A

Obr. B

Kmitajci systm m jeden stupe vonosti (obr. A) a vykonva rotan pohyb. Pohybov
rovnica zostaven pomocou metdy zrchujcich sl
) sin( ) (
0 2 1 1 1 0
t F l l F l F l I
b k
+ + = & & , (a)
kde - sila v pruine
1
kx F
k
= , (b)
- sila v tlmii
1
x b F
b
& = , (c)
- moment zotrvanosti systmu vzhadom na jeho os rotcie

2
2 1 0
2
2 1 0
2
2 1 1 0
) (
3
) ( ) (
3
1
l l m
m
l l m l l m I I I +

+ = + + + = + = , (d)
kde I - moment zotrvanosti tye vzhadom na os rotcie systmu,
I
1
- moment zotrvanosti hmotnho bodu vzhadom na os rotcie sytmu.
I, m

l
1
l
2
m
0
F

F
k
F
b
x
1
x
2


0

.


..

x
1
.

x
2
.

k

b

I, m

l
1
l
2
m
0
F

188
Vzahy medzi posunutiami
1
x ,
2
x a natoenm tye s definovan nasledovne
=
1 1
l x = & &
1 1
l x , (e)
+ = ) (
2 1 2
l l x + = & & ) (
2 1 2
l l x . (f)
Po dosaden rovnc (b-f) do pohybovej rovnice (a) m pohybov rovnica tvar
) sin( ) ( ) (
3
0 2 1
2
1
2
1
2
2 1 0
t F l l kl bl l l m
m
+ = + + +

+ & & & ,


resp.
) sin(
) )( 3 (
3
2
2 1 0
0 2
0
t
l l m m
F

+ +
= + + & & & , (g)
kde
2
2 1 0
2
1
) )( 3 ( 2
3
l l m m
bl
+ +
= - pomern tlm,

2
2 1 0
2
1
0
) )( 3 (
3
l l m m
kl
+ +
= - vlastn kruhov frekvencia pre netlmen sstavu,

2 2
0
=
d
- vlastn kruhov frekvencia pre tlmen sstavu.
Rieenie diferencilnej rovnice (g)
Homognna as rieenia diferencilnej rovnice
0 2
2
0
= + +
h h h
& & & ,
rieenie m tvar
[ ]
h d h
t
h
t C e t + =

sin ) ( .
Partikulrna as rieenia diferencilnej rovnice
) sin(
) )( 3 (
3
2
2 1 0
0 2
0
t
l l m m
F
p p p

+ +
= + + & & & ,
rieenie m tvar




+
+ +
=
2 2
0
2 2 2 2 2
0
2 1 0
0
2
arctan sin
4 ) (
) )( 3 (
3
) ( t
l l m m
F
t
p
.
Celkov rieenie diferencilnej rovnice (g)
) ( ) ( ) ( t t t
p h c
+ = .
Integran kontanty
h h
C , s uren pomocou zaiatonch podmienok.
Jednotliv asov priebehy s zobrazen na nasledovnch obrzkoch (obr. 1

-

obr. 3). Z obr. 1,
resp. z obr. 3 je zreten, e homognna as rieenia s narastajcim asom postupne zanik a
dominantnou sa stva iba partikulrna as rieenia, t.j. as spsoben budiacim silovm
inkom.
189
0,0 0,2 0,4 0,6 0,8 1,0
-0,04
-0,02
0,00
0,02
0,04

h


[
r
a
d
]
as [s]

Obr. 1 asov zvislos natoenia tye (homognne rieenie) pri vonom kmitan
0,0 0,2 0,4 0,6 0,8 1,0
-0,008
-0,004
0,000
0,004
0,008

p


[
r
a
d
]
as [s]

Obr. 2 asov zvislos natoenia tye (partikulrne rieenie) pri vyntenom kmitan
s budiacou silou ) sin(
0
t F F =
0,0 0,2 0,4 0,6 0,8 1,0
-0,04
-0,02
0,00
0,02
0,04

c


[
r
a
d
]
as [s]

Obr. 3 asov zvislos natoenia tye (homognne + partikulrne rieenie) pri vyntenom
kmitan s budiacou silou ) sin(
0
t F F =
190
Vplyvom zmeny budiacej frekvencie sa bude meni amplitda partikulrnej asti rieenia

2 2 2 2 2
0
2 1 0
0
4 ) (
) )( 3 (
3
) (
+
+ +
=
l l m m
F
C
p
,

0 50 100 150 200 250
0,00
0,05
0,10
0,15
a
m
p
l
i
t

d
a


[
r
a
d
]
frekvencia [Hz]
b = 0,0 Ns/m
b = 10,0 Ns/m
b = 20,0 Ns/m
b = 40,0 Ns/m
b = 60,0 Ns/m

Obr. 4 Amplitdovo-frekvenn charakteristika ) sin(
0
t F F =


Prklad 3.63

Vypotajte dobu kmitu mechanickho systmu
pozostvajceho z dvoch ozubench kolies s
momentami zotrvanosti I
1
a I
2
, hmotnosami s
rovnakm polomerom R rozstupovej krunice.
Koles s na polomere r navzjom spojen
pruinou s tuhosou k.
Vsledok:
kr
I I
T
4
2
1 1
0
+
=

Prklad 3.64
Vypotajte vlastn kruhov frekvenciu
0

homognnej tye s kontantnm prierezom po
celej dke, ktor kon rotan pohyb.
Hmotnos tye je m a jej dka je l
1
+ l
2
. Ty je
pripojen na sstavu dvoch prun k
1
a k
2
.
Vsledok:
2
2 1
2
2 1 2
2
1 1
0
) (
) (
3
l l m
l l k l k
+
+ +
=

k
1
k
2
m

l
1
l
2
I
1



I
2
k

R
R

r

r

191
Prklad 3.65
Vypotajte vlastn kruhov frekvenciu mechanickho systmu pozostvajceho z vozka
na dvoch kolesch a hmotnosami m
1
= m
2
= m
3
= m. Koles maj rovnak polomery r
1
= r
2

= r a rovnak momenty zotrvanosti I
1
= I
2
= I = 2
2
mr . Vozk je pripojen k zkladu
dvoma pruinami radenmi za sebou, ktor maj rovnak tuhosti k
1
= k
2
= k.

Vsledok:
m
k
8
0
=





































r
1
, m
1
, I
1
k
1 k
2
r
2
, m
2
, I
2
m
3
192
Zoznam bibliografickch odkazov

[1] BEER, F. P., JOHNSTON, E. R. Vector Mechanics for Engineers : Statics and
Dynamics. 5
th
ed. New York : McGraw-Hill, Inc., 1988, 1026 s. ISBN 0-07-079923-7
[2] BRT, V. Pruka kinematiky s pklady. Praha: SNTL/ALFA, 1976, 273 s. 04-203-76
[3] BRT, V., BROUSIL, J. Dynamika. Praha: ES VUT, 1983, 306 s.
[4] JANINA, J., PEKREK, F. Mechanika II: Kinematika. Bratislava: ALFA/SNTL, 1987,
331 s. 063-556-87 MIK
[5] KRATOCHVL, C., MALENOVSK, E. Mechanika pevnch a tuhch tles : Sbrka loh
z dynamiky. Brno : Akademick nakladatelstv CERM, 2006. 164 s. ISBN 80-214-3228-4
[6] MERIAM, J. L., KRAIGE L. G. Engineering Mechanics : Statics. 4
th
ed. New York:
John Wiley & Sons, Inc., 1998, 524 s. ISBN 0-471-24164-4
[7] MUDRIK, J. a i. Mechanika tuhch telies. Bratislava: Vydavatestvo STU, 1999, 272 s.
ISBN 80-227-1181-0
[8] PEKTORYS, K. a i. Pehled uit matematiky I. 7. vyd. Praha: Prometeus, 2000, 720 s.
ISBN 80-7196-180-9
[9] PEKTORYS, K. a i. Pehled uit matematiky II. 7. vyd. Praha: Prometeus, 2000, 874 s.
ISBN 80-7196-181-7
[10] TIMOSHENKO, S. P.,YOUNG, D. H., WEAVER, W. .
: , 1985, 472 s.
[11] ZHOREC, O..MICHALEK, M., IARAN, S. Dynamika : Zbierka prkladov. Bratislava:
ALFA, 1991, 287 s. ISBN 80-05-00718-3
[12] IARAN, S. Technick mechanika. Statika. Bratislava: Vydavatestvo STU, 2003,
357 s. ISBN 80-227-1863-7















193
Obsah

vod
3
1 STATIKA
4
1.1 Sila a jej inky na hmotn objekt 14
1.2 Statick ekvivalencia silovch sstav 14
1.3 aisko hmotnch objektov 20
1.4 Rovnovha hmotnch objektov 29
1.5 Rovnovha rovinnch sstav telies 43
1.6 Rovinn prtov sstavy 53
1.6.1 Analytick rieenie prtovej sstavy - stynkov metda 54
1.6.2 Analytick rieenie prtovej sstavy - priesen metda 56
1.7 Pasvne odpory 59
1.7.1 mykov trenie pri posuvnom pohybe 59
1.7.2 mykov trenie rotujcich telies 60
1.7.3 Psov trenie 62
1.7.4 Valiv odpor 63
1.7.5 Sstavy telies s uvaovanm pasvnych odporov 65

2 KINEMATIKA
69
2.1 Kinematika bodu 69
2.1.1 Pohyb bodu v kartezinskej sradnicovej sstave 69
2.1.2 Pohyb bodu vo valcovej sradnicovej sstave 70
2.1.3 Pohyb bodu v prirodzenej sradnicovej sstave 71
2.2 Kinematika tuhho telesa 82
2.2.1 Posuvn pohyb telesa v rovine 82
2.2.2 Rotan pohyb telesa 83
2.2.3 Veobecn pohyb telesa v rovine 85
2.2.4 Okamit stred otania 85
2.3 Rchlos a zrchlenie pri sasnch pohyboch 96
194
2.4 Analytick rieenie mechanizmov 102
2.4.1 Trigonometrick metda 102
2.4.2 Vektorov metda 102
2.4.3 Metda komplexnch sel 103

3 DYNAMIKA
112
3.1 Dynamika hmotnho bodu 112
3.2 Dynamika sstavy hmotnch bodov 125
3.3 Geometria hmt 132
3.4 Dynamika tuhho telesa 138
3.5 Dynamika sstavy tuhch telies 152
3.6 Zklady analytickej dynamiky 167

3.7 Linerne kmitanie 172
Zoznam bibliografickch odkazov
192
Obsah
193
















EDCIA VYSOKOKOLSKCH SKRPT




























Autori: Ing. Milan Na, CSc.
Ing. Eva Labaov, PhD.


Nzov: MECHANIKA TUHCH TELIES. Nvody na cvienia.
Miesto vydania: Trnava
Vydavate: AlumniPress
Rok vydania: 2008
Vydanie: prv
Rozsah : 194 strn
Edin slo: 2/AP/2008

ISBN 978-80-8096-050-6
EAN 9788080960506




zverejnen na https://is.stuba.sk

You might also like